JAC Class 11 Geography Important Questions Chapter 13 महासागरीय जल

Jharkhand Board JAC Class 11 Geography Important Questions Chapter 13 महासागरीय जल Important Questions and Answers.

JAC Board Class 11 Geography Important Questions Chapter 13 महासागरीय जल

बहु-विकल्पी प्रश्न (Multiple Choice Questions)

दिए गए चार वैकल्पिक उत्तरों में से सही उत्तर चुनिए
1. निम्नलिखित में से कौन-सा भाग महासागरीय तल का सबसे अधिक क्षेत्र घेरता है?
(A) महाद्वीपीय निमग्न तट
(B) महाद्वीपीय ढाल
(C) महासागरीय मैदान
(D) गर्त।
उत्तर:
(C) महासागरीय मैदान।

2. पृथ्वी पर जलमण्डल का विस्तार लगभग कितना है?
(A)-36 करोड़ वर्ग कि०मी०
(B) 105 करोड़ वर्ग कि०मी०
(C) 10 लाख वर्ग कि०मी०
(D) 500 लाख वर्ग कि०मी०।
उत्तर:
(A) 36 करोड़ वर्ग कि०मी०।

3. महाद्वीपीय निमग्न तट की औसत गहराई कितनी है?
(A) 100 फैदम
(C) 300 मीटर
(B) 500 फुट
(D) 400 मीटर
उत्तर:
(A) 100 फैदम।

4. महाद्वीपीय निमग्न तट का विस्तार किस महासागर में सबसे अधिक है?
(A) अन्ध महासागर
(B) प्रशान्त महासागर
(C) हिन्द महासागर
(D) हिम महासागर।
उत्तर:
(A) अन्ध महासागर।

5. संसार में सबसे गहरा महासागरीय गर्त है
(A) प्यूरिटो रिको
(B) मेरियाना
(C) सुण्डा
(D) ऊटाकाया।
उत्तर:
(B) मेरियाना।

JAC Class 11 Geography Important Questions Chapter 13 महासागरीय जल

6. निम्नलिखित में से कौन-सी खाई प्रशान्त महासागर में स्थित नहीं है?
(A) अल्यूशियन खाई
(B) प्यूरिटो रिको खाई
(C) मिंडानाओ खाई
(D) मेरियाना खाई।
उत्तर:
(B) प्यूरिटो रिको खाई।

7. निम्नलिखित में से कौन-सा सागर अन्ध महासागर का सीमान्त सागर नहीं है?
(A) तसमान सागर
(B) उत्तरी सागर
(C) कैरेबियन सागर
(D) बाल्टिक सागर।
उत्तर:
(A) तसमान सागर।

8. एल्बेट्रोस पठार कौन-से महासागर या महाद्वीप में स्थित है:
(A) हिन्द महासागर
(B) प्रशान्त महासागर
(C) अफ्रीका महाद्वीप
(D) एशिया महाद्वीप
उत्तर:
(B) प्रशान्त महासागर

अति लघु उत्तरीय प्रश्न (Very Short Answer Type Questions)

प्रश्न 1.
महासागरों के ताप प्राप्ति के स्रोत क्या हैं?
उत्तर:

  1. सौर विकिरण।
  2. समुद्र के रेडियो एक्टिव पदार्थ।
  3. जलवाष्प की ऊष्मा।
  4. वायुमण्डल द्वारा संवहन क्रिया।
  5. रासायनिक पदार्थों द्वारा उत्पन्न ताप।

प्रश्न 2.
सागरीय जल के तापमान का वार्षिक अन्तर किन घटकों पर निर्भर होता है?
उत्तर:

  1. विभिन्न गहराइयों पर तापमान में विभिन्नता है।
  2. ताप चालन का प्रभाव।
  3. संवहनी धाराओं का प्रभाव।
  4. जलराशियों का पार्श्व विस्थापन।

प्रश्न 3.
महासागरों में लवण का निर्माण कैसे होता है?
उत्तर:
सागरों में नमक की विशाल मात्रा पाई जाती है। यदि सागरों के समस्त लवण को समतल धरातल पर फैला दिया जाए तो इसकी 150 मीटर मोटी पर्त बन जाएगी। समुद्र की लवणता का मुख्य स्रोत नदियां हैं जो घुले हुए पदार्थ के रूप में लवण समुद्र तट तक पहुँचाती हैं। पवनें तथा लहरें ज्वालामुखी समुद्री जल की लवणता में वृद्धि करते हैं । परन्तु इस लवणता का मुख्य स्रोत समुद्र स्वयं ही है जहां प्रारम्भ से ही लवण पदार्थ विद्यमान हैं।

JAC Class 11 Geography Important Questions Chapter 13 महासागरीय जल

प्रश्न 4.
महासागरों के क्षैतिज तथा ऊर्ध्वाधर तापमान वितरण को कौन-से कारक नियन्त्रित करते हैं?
उत्तर:
महासागरों के जल का तापमान निम्नलिखित कारकों द्वारा नियन्त्रित होता है:

  1. सूर्यातप की तीव्रता और अवधि।
  2. जल का खारापन, घनत्व तथा वाष्पीकरण।
  3. ऊष्ण तथा शीत वायु धाराओं का बहना।
  4. जल मग्न कटकों की स्थिति।
  5. समुद्र की स्थिति और आकार।

प्रश्न 5.
समुद्री जल में पाए जाने वाले पांच प्रसिद्ध लवण तथा उनकी मात्रा बताओ।
उत्तर:

  1. सोडियम क्लोराइड – 77.7 प्रतिशत
  2. मैग्नीशियम क्लोराइड – 10.9 प्रतिशत
  3. मैग्नेशियम सल्फेट – 4.7 प्रतिशत
  4. कैल्शियम सल्फेट – 3.6 प्रतिशत
  5. पोटेशियम सल्फेट – 2.5 प्रतिशत

प्रश्न 6.
महासागरीय जल की लवणता किन तत्त्वों पर प्रभाव डालती है?
उत्तर:

  1. तापमान
  2. घनत्व
  3. सूर्यातप का अवशोषण
  4. वाष्पीकरण
  5. आर्द्रता
  6. मछलियों के जीवन

प्रश्न 7.
महासागरीय जल की लवणता के विभिन्न स्त्रोत बताओ।
उत्तर:

  1. नदियां
  2. महासागरीय लहरें
  3. ज्वालामुखी
  4. रासायनिक क्रियाएं
  5. महासागरों में पहले ही लवण का होना।

प्रश्न 8.
भूमध्यरेखा के निकट महासागरों में लवणता कम क्यों है?
उत्तर:

  1. सारा साल भारी वर्षा (200 सें० मी० से अधिक )
  2. सारा साल उच्च सापेक्षिक आर्द्रता (80%)
  3. मेघाच्छनन आकाश
  4. डोलड्रम क्षेत्र शांत वायु
  5. अमेजन तथा जायरे जैसी बड़ी-बड़ी नदियों के स्वच्छन्द जल के कारण।

प्रश्न 9.
घिरे हुए तीन सागरों के नाम तथा लवणता बताओ।
उत्तर:

  1. ग्रेट साल्ट झील (संयुक्त राज्य) – 220 लवणता प्रति हज़ार
  2. मृत सागर ( जार्डन) – 240 लवणता प्रति हज़ार
  3. वान झील (तुर्की) – 330 लवणता प्रति हज़ार।

प्रश्न 10.
ज्वारीय ऊर्जा किसे कहते हैं?
उत्तर:
ज्वार-भाटा महासागरों में ऊर्जा के प्रमुख स्रोत हैं। ज्वार भाटे के समय जल के ऊपर उठने तथा नीचे गिरने से ज्वारीय ऊर्जा उत्पन्न होती है। ज्वार भाटे की गति से जैनरेटर चलाने का कार्य किया जा सकता है। सोवियत संघ, जापान तथा फ्रांस में ज्वारीय शक्ति उत्पन्न की जाती है।

प्रश्न 11.
भू-तापीय ऊर्जा से क्या अभिप्राय है?
उत्तर:
पृथ्वी के भूगर्भ में गर्म जल तथा ज्वालामुखियों द्वारा उत्पन्न ऊर्जा को भू-तापीय ऊर्जा कहते हैं। गर्म पानी के झरने तथा गीजर इसके प्रमुख स्रोत हैं। भू-तापीय ऊर्जा न्यूजीलैंड, संयुक्त राज्य तथा मैक्सिको में उत्पन्न की जाती है।

JAC Class 11 Geography Important Questions Chapter 13 महासागरीय जल

प्रश्न 12.
रचना के आधार पर महाद्वीपीय मग्न तट के कितने प्रकार हैं?
उत्तर:
तीन प्रकार: नदियों से निर्मित, हिमानीकृत, प्रवाल भित्ति निर्मित।

लघु उत्तरीय प्रश्न (Short Answer Type Questions )

प्रश्न 1.
महासागरीय तली को कितने मुख्य विभागों में बांटा जाता है?
उत्तर:
महासागरीय तली को चार मुख्य विभागों में बांटा जाता है:

  1. महाद्वीपीय मग्न तट (Continental shelf)
  2. महाद्वीपीय ढाल (Continental slope )
  3. महासागरीय मैदान (Deep sea plain )
  4. महासागरीय गर्त (Ocean deeps)।

प्रश्न 2.
महासागरीय नितलों पर पाए जाने वाली सबसे अधिक सामान्य आकृतियों के नाम लिखो।
उत्तर:
महासागरीय तली पर महाद्वीपीय मग्न तट, महाद्वीपीय ढाल, महासागरीय मैदान तथा गर्त के अतिरिक्त निम्नलिखित लक्षण पाए जाते हैं

  1. उद्रेख (Ridges)
  2. पहाड़ियाँ (Hills)
  3. टीले (Sea mounts)
  4. खाइयां (Trenches)
  5. palm fufaui (Coral reefs) |
  6. निमग्न द्वीप (Guyots)
  7. कैनियन ( Canyons)

प्रश्न 3.
महासागरीय खाइयों तथा गर्तों को विवर्तनिक उत्पत्ति वाला क्यों समझा जाता है?
उत्तर:
महासागरों में लम्बे, गहरे तथा संकरे खड्ड को महासागरीय गर्त कहा जाता है। इनकी उत्पत्ति भूतल पर दरारें पड़ने तथा मोड़ पड़ने की हलचल के कारण हुई है। ये अधिकतर उन क्षेत्रों में पाए जाते हैं जहां भूकम्प आते हैं तथा ज्वालामुखी स्थित हों। ये अधिकतर वलित पर्वतों तथा द्वीपीय चापों के समानान्तर स्थित होते हैं। इसलिए इनका सम्बन्ध भूगर्भिक हलचलों से है।

प्रश्न 4.
महाद्वीपीय मग्नतट किसे कहते हैं?
उत्तर:
महाद्वीपीय मग्नतट (Continental Shelf):
यह महाद्वीपों के चारों ओर मंद ढाल वाला जलमग्न धरातल है। वास्तव में यह महाद्वीपीय खंड का ही जलमग्न किनारा है जो 150 से 200 मीटर तक गहरा होता है।

प्रश्न 5.
महासागरीय नितल की गहराई कैसे मापी जाती है?
उत्तर:
महासागरीय नितल की गहराई गम्भीरता मापी यन्त्र (Sonic Depth Recorder) से मापी जाती है। इस यन्त्र से ध्वनि तरंगें (Sound waves) महासागरीय नितल से प्रति ध्वनि (Echo) के रूप में वापस आती हैं। इनकी गति व समय से गहराई ज्ञात की जाती है।

प्रश्न 6.
संसार में सबसे गहरा स्थान कौन-सा है?
उत्तर:
संसार में सबसे गहरा स्थान प्रशांत महासागर में गुआम द्वीपमाला के समीप मेरिआना गर्त (Mariana Trench) है। इसकी गहराई 11022 मीटर है। यदि एवरेस्ट पर्वत को इस गर्त में डुबो दिया जाए तो इसकी चोटी समुद्री जल सतह से 2 कि०मी० नीचे रहेगी।

प्रश्न 7.
हिन्द महासागर को आधा महासागर क्यों कहते हैं?
उत्तर:
अन्ध महासागर तथा प्रशान्त महासागर उत्तर-दक्षिण दोनों ओर खुले हैं। ये भूमध्य रेखा के दोनों ओर समान रूप से फैले हुए हैं। परन्तु हिन्द महासागर उत्तर की ओर बन्द है। एशिया महाद्वीप इस के विस्तार को रोकता है। एक प्रकार से इसका विस्तार अधिकतर दक्षिण की ओर ही है। इसलिए इसे आधा महासागर कहते हैं।

JAC Class 11 Geography Important Questions Chapter 13 महासागरीय जल

प्रश्न 8.
जलमण्डल से क्या अभिप्राय है?
उत्तर:
पृथ्वी के तल के जल से डूबे हुए भाग को जल मण्डल (Hydrosphere) कहते हैं। यह धरालत पर लगभग 361,059,000 वर्ग किलोमीटर में फैले हुए हैं जो पृथ्वी के धरातल के कुल क्षेत्र का 71% भाग है। उत्तरी गोलार्द्ध का 61% भाग तथा दक्षिणी गोलार्द्ध का 81% भाग महासागरों से घिरा हुआ है। उत्तरी गोलार्द्ध की अपेक्षा दक्षिणी गोलार्द्ध में जल का विस्तार अधिक है इसलिए इसे (Water Hemisphere) भी कहते हैं।

प्रश्न 9.
प्रति ध्रुवीय स्थिति से क्या अभिप्राय है?
उत्तर:
धरती पर जल और स्थल का वितरण प्रति ध्रुवीय (Antipodal) है। महाद्वीप और महासागर एक-दूसरे के विपरीत स्थित हैं। यह संयोजन इस प्रकार है कि जल और स्थल एक-दूसरे से एक व्यास के विपरीत कोनों पर (Diametrically Opposite) स्थित हैं, जैसे आर्कटिक महासागर अण्टार्कटिक महाद्वीप के विपरीत स्थित है। यूरोप तथा अफ्रीका प्रशान्त महासागर के विपरीत स्थित हैं। उत्तरी अमेरिका हिन्द महासागर के विपरीत स्थित है।

प्रश्न 10.
गम्भीर समुद्री उद्रेख (Submarine Ridge) किसे कहते हैं?
उत्तर:
महासागरीय तल पर ऊंचे उठे हुए भागों को गम्भीर उद्रेख कहते हैं। यह प्रायः 60 हज़ार किलोमीटर लम्बे और 100 किलोमीटर चौड़े हो सकते हैं। इनकी विश्वव्यापी स्थिति किसी भूमण्डलीय हलचल का संकेत देती है । प्रायः यह महासागरों के मध्य में या धरती पर पाई जाती है। इनकी रचना के कई कारण हैं:

  1. दरारों के साथ बॅसाल्ट का फैलना।
  2. संवाहिक धाराओं द्वारा भूपटल का ऊंचा उठना तथा नीचे धंसना।

प्रश्न 11.
अन्त: समुद्री कैनियन की विशेषताओं एवं निर्माण को स्पष्ट करें।
उत्तर:
महासागरीय निमग्न तट तथा ढाल पर तंग, गहरी तथा ‘V’ आकार की घाटियों को कैनियन कहा जाता है। ये घाटियां विश्व के सभी तटों पर नदियों के मुहानों पर पाई जाती हैं। जैसे- हडसन, सिन्ध, गंगा, कांगो नदी । यह कैनियन नदी द्वारा अपरदन तथा सागरीय अपरदन से बनी है।

कैनियनों के प्रकार (Types of Canyons) ये घाटियां मुख्य रूप से तीन प्रकार की होती हैं:

  1. वे कैनियन जो छोटे, गार्ज के रूप में महाद्वीपीय मग्न तट से शुरू होकर ढाल पर काफ़ी गहराई तक विस्तृत होते हैं। जैसे न्यू इंग्लैण्ड तट पर ओशनोग्राफर ( oceanographer) कैनियन।
  2. वे कैनियन जो नदियों के मुहानों से शुरू होकर केवल मग्न तट तक ही पाए जाते हैं। जैसे – मिसीसिपी तथा सिन्धु नदी के कैनियन। हडसन कैनियन।
  3. वे कैनियन जो तट व ढाल पर काफ़ी कटे-फटे होते हैं, जैसे- दक्षिणी कैलीफोर्निया के तट पर बेरिंग केनियन तथा जेम चुंग कैनियन।

प्रश्न 12.
मनुष्य के लिए महासागरों के महत्त्व का वर्णन करो।
उत्तर:
महासागरों का महत्त्व – महासागर कई प्रकार से, प्रत्यक्ष तथा परोक्ष रूप से मनुष्य के लिए उपयोगी हैं।

  1. महासागर जलवायु पर व्यापक प्रभाव डालते हैं।
  2. समुद्री धाराएं तापमान और आर्द्रता के वितरण को प्रभावित करती हैं।
  3. महासागर बढ़ती हुई जनसंख्या के लिए खाद्य पदार्थों का सुलभ तथा अनन्त भण्डार हैं।
  4. महासागर संसार के भोजन का 10% भाग मछलियों द्वारा प्रदान करते हैं।
  5. महासागरों में अनेक समुद्री जन्तु, तेल, चमड़ा, सरे आदि उपयोगी वस्तुएं प्राप्त होती हैं।
  6. महासागरों में कम गहरे भागों में खनिज तेल तथा प्राकृतिक गैस के भण्डार प्राप्त हुए हैं।
  7. महासागर में अनेक उपयोगी खनिजों के भण्डार हैं। जैसे – मैंगनीज़, मोनोजाइट, लोहा, टिन, सोना आदि।
  8. महासागरों में ज्वार-भाटे की तरंगों से ज्वारीय शक्ति उत्पन्न की जाती है।
  9. महासागर यातायात तथा परिवहन के सब से महत्त्वपूर्ण, सस्ते तथा प्राकृतिक साधन हैं।
  10. महासागरों से आवश्यक लाभ प्राप्त करने के लिए इन्हें प्रदूषण से मुक्त रखना आवश्यक है ।

प्रश्न 13.
मानव के लिए महासागरों के विभिन्न प्रत्यक्ष और परोक्ष उपयोग क्या-क्या हैं?
उत्तर:

  • प्रत्यक्ष उपयोग
    1. महासागर मछली जैसे खाद्य पदार्थ का असीमित भण्डार हैं।
    2. महासागर खनिजों का भण्डार हैं।
    3. महासागर यातायात तथा परिवहन मार्ग हैं।
  • अप्रत्यक्ष उपयोग
    1. महासागर जलवायु को नियन्त्रित करते हैं।
    2. महासागर ज्वारीय ऊर्जा तथा भूतापीय ऊर्जा के भण्डार हैं।

प्रश्न 14.
महासागरों को पृथ्वी पर जलवायु के महान् नियन्त्रक क्यों कहते हैं?
अथवा
महासागर जलवायु को किस प्रकार प्रभावित करते हैं?
उत्तर:
महासागर जलवायु पर व्यापक प्रभाव डालने के कारण प्रमुख नियन्त्रक कहे जाते हैं।

  1. महासागर धरातल पर तापमान तथा आर्द्रता पर प्रभाव डालते हैं।
  2. महासागर सौर ऊर्जा को संचय करते हैं
  3. महासागरों में ऊर्जा के अवशोषण और निष्कर्षण की विशाल क्षमता है।
  4. समुद्र तट पर तापांतर बहुत कम होता है।
  5. महासागरीय धाराएं तटीय क्षेत्रों के तापमान को सम करने में मदद करती हैं।

JAC Class 11 Geography Important Questions Chapter 13 महासागरीय जल

प्रश्न 15.
अन्तर्राष्ट्रीय व्यापार के लिए महासागर कैसे वरदान सिद्ध हुए हैं?
उत्तर:
महासागर यातायात तथा परिवहन के सबसे महत्त्वपूर्ण प्राकृतिक साधन हैं। महासागर सबसे सस्ता यातायात साधन हैं। महासागरों में कोई सड़कों या मार्गों का निर्माण नहीं होता । महासागर सभी महाद्वीपों को आपस में जोड़कर एक अन्तर्राष्ट्रीय मार्ग की रचना करते हैं।

प्रश्न 16.
महाद्वीपीय मग्न तट तथा महाद्वीपीय ढाल में अन्तर स्पष्ट करो।
उत्तर:

महाद्वीपीय मग्न तट (Continental Shelf) महाद्वीपीय ढाल (Continental Slope)
(1) महाद्वीपों के चारों ओर जल-मग्न चबूतरों को महाद्वीपीय मग्न तट कहते हैं। (1) महाद्वीपीय मग्न तट से महासागरों की ओर के ढाल को महाद्वीपीय ढाल कहते हैं।
(2) इसकी औसत गहराई 200 मीटर (100 फैदम) होती है। (2) इसकी औसत गहराई 200 मीटर से 3000 मीटर तक होती है।
(3) समस्त महासागरों के $7.5$ प्रतिशत भाग पर इसका विस्तार है। (3) समस्त महासागरों के $8.5$ प्रतिशत भाग पर इसका विस्तार है।
(4) इसका औसत ढाल 10 से कम है। (4) इसका औसत ढाल 2° से 5° है।
(5) मछली क्षेत्रों तथा पेट्रोलियम के कारण इसका आर्थिक महत्त्व है। (5) इस पर कई समुद्री कैनियन स्थित हैं।

निबन्धात्मक प्रश्न (Essay Type Questions)

प्रश्न 1.
महासागरों की तली के उच्चावच के सामान्य लक्षणों का वर्णन करो।
उत्तर:
महासागरों की गहराई तथा उच्चावच में काफ़ी विभिन्नता है जिसे उच्चता दर्शी वक्र (Hypsographic Curve) से दिखाया जाता है। बनावट तथा गहराई के अनुसार सागरीय तल को चार भागों में बांटा जाता है
1. महाद्वीपीय मग्न तट (Continental Shelf): महाद्वीपों के चारों ओर सागरीय तट का वह भाग जो 150 से 200 मीटर तक गहरा होता है, महाद्वीपीय चबूतरा कहलाता है। वास्तव में यह महाद्वीप का ही भाग होता है जो जलमग्न होता है। विस्तार समस्त महासागरों के 7.5 प्रतिशत भाग ( 260 लाख वर्ग किलोमीटर) पर महाद्वीपीय मग्न तट का विस्तार है। इसका सबसे अधिक विस्तार अन्धमहासागर ( 13.3%) है। इसकी औसत चौड़ाई 70 किलोमीटर तथा गहराई 72 फैदम होती है। आर्कटिक सागर के तट पर इसका विस्तार 1000 कि० मी० से भी अधिक है। पर्वत श्रेणियों वाले तटों पर संकरे महाद्वीपीय मग्न तट पाए जाते हैं। इस तट का औसत ढाल 1° कोण होता है। भारत के पूर्वी तट पर चौड़ा मग्न तट मिलता है।

मग्न तटों की उत्पत्ति – मग्न तटों के निर्माण सम्बन्धी विचार इस प्रकार हैं:

  1. कुछ विद्वानों के अनुसार मग्न तट वास्तव में स्थल का बढ़ा हुआ रूप है। समुद्र तल के ऊपर उठने से या स्थल भाग के नीचे धंस जाने से मग्न तट की रचना होती है।
  2. सागरीय अपरदन से इन चबूतरों का निर्माण होता है।
  3. नदियों, लहरों, वायु आदि द्वारा तलछट के निक्षेप से मग्न तट चबूतरों (Submarine Terrace) का निर्माण होता है।

महत्त्व:
महाद्वीपीय मग्न तट मनुष्य के लिए काफ़ी उपयोगी हैं। इन प्रदेशों में मछलियों के भण्डार हैं। यहां तेल व गैस उत्पादन होता है। यहां बजरी व बालू के विशाल भण्डार पाए जाते हैं। यहां समुद्री जीवों तथा वनस्पति की अधिकता होती है।

2. महाद्वीपीय ढाल (Continental Slope ):
महाद्वीपीय मग्न तट के बाहर का ढाल जो महासागर की ओर तीव्र गति से नीचे उतरता है, महाद्वीपीय ढाल कहलाता है। वास्तव में महाद्वीप ढाल के किनारे पर ही समाप्त होते हैं। इसकी गहराई 3660 मीटर तक है। इसका कुल विस्तार 8.5% क्षेत्र (310 लाख वर्ग कि० मी०) पर है । इसका सब से अधिक विस्तार अन्धमहासागर में 12.4% क्षेत्र पर है। इसकी ढाल का औसत कोण 4° है, परन्तु स्पेन के निकट यह कोण 36° है। यह ढाल वास्तव में गहरे समुद्रों तथा महाद्वीपों के स्तर को पृथक् करती है। जहां महाद्वीपीय ढाल का अन्त होता है, वहां मन्द ढाल को महाद्वीपीय उत्थान कहते हैं।
JAC Class 11 Geography Important Questions Chapter 13 महासागरीय जल 1

3. महासागरीय मैदान (Deep Sea plain ):
महाद्वीपीय ढाल के पश्चात् समुद्र में चौड़े तथा समतल क्षेत्र को महासागरीय मैदान कहते हैं। इसकी औसत गहराई 3000 से 6000 मीटर तक है। कुल महासागरों में इसका लगभग 40% विस्तार है। इसका सबसे अधिक विस्तार प्रशान्त महासागर में 80.3% है। इसे नितल मैदान (Abyssal plain) भी कहते हैं। इन मैदानों का ढाल 1/100 से भी कम होता है। इन मैदानों पर कई भू-आकृतियां पाई जाती हैं, जैसे – समुद्री उद्रेख (Ridges ) द्वीप, समुद्री टीले (Sea mounts), निमग्न द्वीप (Guyots) आदि। इन मैदानों पर स्थलज तथा उथले जल से उत्पन्न तलछट पाए जाते हैं। नितल मैदान पर जलमग्न कटक पाए जाते हैं जो महासागरों के मध्य क्षेत्र में हैं। इन कटकों की लम्बाई 75000 कि० मी० है। ये मन्द ढाल वाले चौड़े पठार के समान हैं।

4. महासागरीय गर्त (Ocean Deeps):
ये समुद्र तल पर गहरे गड्ढे होते हैं। इनका विस्तार बहुत कम होता है। कुल महासागरों के 7% भाग में महासागरीय गर्त पाए जाते हैं। लम्बे चौड़े खड्ड को गर्त (Trough) कहते हैं जबकि लम्बे, गहरे तथा संकरे खड्ड को खाई (Trench) कहा जाता है। इनकी औसत गहराई 5500 मीटर है। संसार में 57 प्रसिद्ध गड्ढे हैं जिनमें अन्धमहासागर में 19, प्रशान्त महासागर में 32 तथा हिन्द महासागर में 6 हैं। संसार में सब से अधिक गहरा गर्त (Mariana Trench) प्रशान्त महासागर में है जिसकी गहराई 11022 मीटर है। इन गर्तों की मुख्य विशेषताएं निम्नलिखित हैं:

(क) ये महासागरों के किनारों पर पाये जाते हैं।
(ख) ये भूकम्पीय तथा ज्वालामुखी क्षेत्रों से सम्बन्धित हैं।
(ग) ये गर्त द्वीपीय चापों के सहारे मिलते हैं।

5. अन्य विशेष आकृतियां:
महासागरों में उच्चावच में विविधता के कारण कई भू-आकृतियां, जैसे- द्वीप (Islands), उद्रेख (Ridge), अटोल (Attol), समुद्री कैनियन (Canyon ) भी पाई जाती हैं।

JAC Class 11 Geography Important Questions Chapter 13 महासागरीय जल

प्रश्न 2.
महासागरों में मिलने वाले छुद्र उच्चावच्च के लक्षण बताओ।
उत्तर:
छुद्र उच्चावच्च के लक्षण (Minor Relief Features): ऊपर दिए गए महासागरीय सतह के बड़े उच्चावच्चों के अतिरिक्त कुछ छोटी लेकिन महत्त्वपूर्ण आकृतियां महासागरों के विभिन्न भागों में बहुतायत पायी जाती हैं।

1. मध्य महासागरीय कटक (Mid Oceanic Ridge ):
एक मध्य महासागरीय कटक पर्वतों के दो श्रृंखलाओं का बना होता है, जिसके बीच बहुत बड़ा गड्ढा होता है। इन पर्वत श्रृंखलाओं के शिखर की ऊंचाई 2,500 मीटर तक हो सकती है तथा इनमें से कुछ समुद्र की सतह तक भी पहुंच सकती हैं, उदाहरण के लिए, आईसलैंड, जो कि मध्य अटलांटिक कटक का एक भाग है।

2. समुद्री पर्वत (Sea Mount ):
यह नुकीले शिखरों वाला एक पर्वत है, जो कि समुद्री सतह से ऊपर की ओर उठा होता है, किंतु महासागरों के ऊपरी स्तर तक नहीं पहुंच पाता है। समुद्री पर्वत ज्वालामुखी के द्वारा उत्पन्न होते हैं। ये 3,000 से 4,500 मीटर लम्बे हो सकते हैं। उदाहरणस्वरूप, इम्पेरर समुद्री पर्वत, जो कि प्रशान्त महासागर में हवाई द्वीपसमूहों का विस्तार है।

3. अंत: समुद्री दरे (कैनियन) (Canyons ):
कोलोरैडो नदी के ग्रैंड कैनियन की तरह ये गहरी घाटियां होती हैं। ये कभी-कभी महादेशीय छज्जों एवं महादेशीय ढालों को एक तरफ से दूसरे तरफ काटते हुए पाए जाते हैं जो कि प्रायः बड़ी नदियों के मुहाने से विस्तृत होते हैं। विश्व का सबसे महत्त्वपूर्ण दर्रा हडसन दर्रा है।

4. निमग्न द्वीप (Guyots):
यह चपटे शिखर वाला समुद्री पर्वत है। ये लगातार हो रहे अवतलन के प्रमाणों को दर्शाते हैं, जिसके कारण धीरे-धीरे इन डूबे हुए पर्वतों का शिकर चपटा होता जाता है। प्रशान्त महासागर में अनुमानतः 10,000 से अधिक समुद्री पर्वत एवं निमग्न द्वीप उपस्थित हैं।

5. प्रवाल द्वीप (Atoll ):
ये कटिबन्धीय महासागरों में पाए जाने वाले छोटे आकार के द्वीप हैं, न जहां एक गड्ढे को चारों तरफ से मूंगे की चट्टानें रहती हैं। ये समुद्र के एक भाग हो सकते हैं या कभी-कभी ये साफ, खारे या बहुत अधिक जल को चारों तरफ से रहते हैं।

प्रश्न 3.
विभिन्न सागरों में लवणता की मात्रा को प्रभावित करने वाले तत्त्वों का वर्णन करो।
उत्तर:
समुद्र का जल सदा खारा होता है। सागरीय जल में औसत लवणता 35 प्रति हज़ार है। परन्तु भिन्न-भिन्न सागरों में लवणता की मात्रा भिन्न-भिन्न होती है। खारेपन की भिन्नता निम्नलिखित तत्त्वों पर निर्भर करती है

1. स्वच्छ जल की पूर्ति ( Supply of Fresh Water ):
स्वच्छ जल की अधिकता से सागरीय जल में लवणता कम हो जाती है। भूमध्य रेखीय खण्ड में अधिक वर्षा से सागरीय जल में कम लवणता पाई जाती है। बड़ी-बड़ी नदियों के मुहानों के निकट लवणता कम होती है। ध्रुवीय प्रदेशों में हिम के पिघलने से स्वच्छ जल प्राप्त होता रहता है जिससे लवणता कम हो जाती है।

2. वाष्पीकरण की मात्रा तथा गति (Rapidity and Amount of Evaporation ):
अधिक तापक्रम, वायु की तीव्र गति तथा शुष्कता के कारण वाष्पीकरण की क्रिया अधिक होती है जिससे सागरीय जल में लवणता बढ़ जाती है। इसी कारण कर्क रेखा तथा मकर रेखा के आस-पास लवणता अधिक होती है।

3. सागरीय जल की मिश्रण क्रिया – सागरीय जल, ज्वार भाटा, लहरों तथा धाराओं के कारण एक स्थान से दूसरे स्थान तक बहता रहता है, जल के इस मिश्रण से स्थानिक रूप में लवणता बढ़ जाती है या कम हो जाती है।

4. प्रचलित पवनें (Prevailing Winds):
गर्म तथा शुष्क पवनों के कारण वाष्पीकरण अधिक होता है। उच्च वायु दबाव पेटियों में नीचे उतरती पवनों के कारण वाष्पीकरण अधिक होता है जो लवणता में वृद्धि कर देता है।

5. धारायें (Currents):
धाराएं खुले सागरों में एक भाग से दूसरे भाग तक जल ले जाती हैं। गर्म धाराएं लवणता को बढ़ा देती हैं तथा ठण्डी धाराएं लवणता को कम करती हैं। इस प्रकार महासागरों में तापमान, घनत्व तथा लवणता में सम्बन्ध है। तापमान तथा घनत्व में परिवर्तन से लवणता में परिवर्तन होता है।

JAC Class 11 Geography Important Questions Chapter 13 महासागरीय जल

प्रश्न 4.
महासागरों में तापमान के वितरण को नियन्त्रित करने वाले घटकों का वर्णन करो।
उत्तर:
सागरीय जल ताप का एक उत्तम संचालक है। इसी कारण जल, स्थल की अपेक्षा देर से गर्म होता है तथा देर से ठण्डा होता है। सागरीय जल का तापमान सभी स्थानों पर एक समान नहीं होता। सागरीय जल के तापमान का वितरण निम्नलिखित घटकों पर निर्भर करता है:

1. भूमध्य रेखा से दूरी:
सागरीय जल की ऊपरी सतह का तापमान अक्षांश के साथ हटता रहता है। भूमध्य रेखा पर यह तापमान 27° C के लगभग रहता है। 40° अक्षांश पर सागरीय जल का तापमान 14° C पाया जाता है तथा 70° अक्षांश पर 5°C। शून्य डिग्री सेल्सियस समताप रेखा ध्रुवीय क्षेत्रों के गिर्द वृत्त बनाती है। प्रति 1° अक्षांश पर 0.5°C तापमान घटता है।

2. प्रचलित पवनें:
स्थायी पवनें समुद्र जल की ऊपरी परत को हटाती रहती हैं तथा नीचे से ठण्डा जल आ जाता है। इस उत्स्रवण (Up welling of Water) की क्रिया से तापमान कम हो जाता है। इसके विपरीत समुद्र से स्थल की ओर आने वाली पवनें गर्म जल इकट्ठा कर के तापमान को बढ़ा देती हैं।

3. महासागरीय धाराएं:
महासागरीय धाराएं तापमान में समानता लाने का प्रयत्न करती हैं। गर्म धाराएं ठण्डे प्रदेशों में तापमान को बढ़ा देती हैं। उष्ण गल्फस्ट्रीम के कारण ही पश्चिमी यूरोप में तापमान 5° C से अधिक रहता है। इसके विपरीत ठण्डी धाराएं तापमान को ओर भी कम कर देती हैं, ठण्डी लेब्रेडोर धारा के कारण न्यूफाउण्डलैण्ड के निकट तापमान 2° C से कम होता है।

4. लवण में भिन्नता:
अधिक लवण वाले जल का तापमान ऊंचा होता है क्योंकि वह अधिक गर्मी ग्रहण कर सकता है।

5. स्थल खण्डों की स्थिति:
उष्ण कटिबन्ध में स्थल के घिरे हुए सागरों का तापमान अधिक होता है परन्तु शीत कटिबन्ध में कम होता है।

6. समुद्र की गहराई:
समुद्र की गहराई बढ़ने के साथ-साथ तापमान कम होता है। ऊपरी सतह से लेकर 1800 मीटर की गहराई तक सागरीय जल का तापमान 15° C से घट कर 2° C रह जाता है। 1800 से 4000 मीटर की गहराई तक यह तापमान 2°C से घट कर 1.6° C रह जाता है

7. अंत: समुद्री रोधिकाएं (Submarine Ridges):
कम गहरे भागों में पानी के नीचे रोधिकाएं तापमान में अन्तर डालती हैं।

प्रश्न 5.
सागरीय जल की लवणता से क्या अभिप्राय है? संसार के विभिन्न सागरों में लवणता का वितरण बताइये।
उत्तर:
समुद्र जल में पाये जाने वाले समस्त लवणों का योग समुद्र की लवणता कहलाता है। समस्त घुले हुए लवण एक निश्चित अनुपात में पाये जाते हैं परन्तु विभिन्न स्थानों पर विभिन्न मात्रा में मिलते हैं। महासागरीय लवणता उस अनुपात को कहते हैं जो घुले हुए लवणों की मात्रा और समुद्र जल की मात्रा में होता है। इस लवणता को प्रति हज़ार भागों में प्रकट किया जाता है। समुद्र जल की औसत लवणता प्रति हज़ार ग्राम जल में पैंतीस ग्राम लवण पदार्थ है तथा इसे 35 प्रति हज़ार लिखा जाता है। लवणता का वितरण: कई भौगोलिक तत्त्वों के कारण विभिन्न सागरों में लवणता की मात्रा में अन्तर पाया जाता है।

(क) खुले महासागरों में लवणता (Salinity in Open Seas)
1. भूमध्य रेखा के आस-पास के क्षेत्र (Near the Equator):
इन सागरों में औसत लवणता कम (लगभग 34 प्रति हज़ार ) होती है। ( 33% से 37% ) कारण:

  • अधिक वर्षा
  • अधिक मेघाच्छादन
  • अमेजन तथा कांगो जैसी बड़ी-बड़ी नदियों से विशाल स्वच्छ जल की प्राप्ति।

2. कर्क व मकर रेखा के निकट (Near the Tropics): यहां पर लवणता की मात्रा सबसे अधिक (36 प्रति हज़ार ) पाई जाती है।
कारण:

  1. अधिक वाष्पीकरण
  2. स्वच्छ आकार तथा शुष्क वायु
  3. कम वर्षा
  4. बड़ी नदियों का अभाव। गर्म – शुष्क प्रदेशों में यह 70% है।

3. ध्रुवीय क्षेत्र (Polar Areas): इन क्षेत्रों में लवणता की मात्रा 20 से 30 प्रति हज़ार होती है।
कारण:

  1. कम ताप के कारण वाष्पीकरण का कम होना।
  2. पश्चिमी पवनों द्वारा अधिक वर्षा का होना।
  3. बर्फ़ के पिघलने से स्वच्छ जल की प्राप्ति। आर्कटिक महासागर में यह 0-30% है। उत्तरी सागर में गर्म धारा के कारण अधिक है।

(ख) घिरे हुए समुद्रों में लवणता (Salinity in Enclosed Seas):

इन सागरों में लवणता की मात्रा में काफ़ी अन्तर पाया जाता है, जैसे:

  1. भूमध्य सागर में जिब्रालटर के समीप खारेपन की मात्रा 37 प्रति हज़ार से 39 प्रति हज़ार है। अधिक लवणता शुष्क – ग्रीष्म ऋतु, अधिक वाष्पीकरण तथा नदियों के अभाव के कारण है।
  2. लाल सागर 40 प्रति हज़ार, खाड़ी स्वेज 41 प्रति हज़ार, खाड़ी फारस में 38 प्रति हज़ार लवणता की मात्रा पाई जाती है।
  3. काला सागर में 18 प्रति हज़ार तथा एजोव सागर में 17 प्रति हज़ार लवणता पाई जाती है। बाल्टिक सागर में केवल 11 प्रति हज़ार लवणता पाई जाती है। यहां वाष्पीकरण कम है। बड़ी-बड़ी नदियों से स्वच्छ जल प्राप्त होता है हिम के पिघलने से भी अधिक जल की प्राप्ति होती है।

( ग ) झीलें तथा आन्तरिक सागर (Lake and Inland Seas ):
झीलों तथा आन्तरिक सागरों में लवणता की मात्रा इनमें गिरने वाली नदियों, वाष्पीकरण तथा स्थिति के कारण भिन्न- भिन्न होती है। झीलों में नदियों के गिरने से स्वच्छ जल अधिक हो जाता है तथा लवणता कम होती है। नदियों की कमी वाष्पीकरण अधिक होता है तथा लवणता अधिक होती है। कैस्पियन सागर के उत्तरी भाग में लवणता 14 प्रति हज़ार है परन्तु दक्षिणी भाग में लवणता 170 प्रति हज़ार है। संयुक्त राज्य अमेरिका की साल्ट झील में लवणता 220 प्रति हज़ार है । जार्डन में मृत सागर (Dead Sea) में लवणता 240 प्रति हज़ार है। तुर्की की वैन झील (Van Lake) में लवणता 330 प्रति हज़ार है। यहां अधिक लवणता अधिक वाष्पीकरण तथा नदियों की कमी के कारण है।

JAC Class 11 Geography Important Questions Chapter 13 महासागरीय जल

प्रश्न 6.
महासागरों के महत्त्व का वर्णन करते हुए स्पष्ट करो कि महासागर भविष्य के भण्डार हैं।
उत्तर:
महासागरों का महत्त्व (Importance of Oceans):
महासागरों का प्रभाव तटीय प्रदेश के लोगों पर प्रत्यक्ष रूप से पड़ता है। परन्तु परोक्ष रूप से महासागर मनुष्य के लिए बहुत उपयोगी हैं, महासागर जलवायु पर व्यापक प्रभाव डालते हैं, सागरीय धाराएं तापमान तथा आर्द्रता के वितरण को प्रभावित करती हैं। महासागरों में ज्वार-भाटे की तरंगों से ज्वरीय शक्ति उत्पन्न की जा सकती है। महासागर यातायात तथा परिवहन के सबसे महत्त्वपूर्ण, सस्ते तथा प्राकृतिक साधन हैं। महासागर बढ़ती हुई जनसंख्या के लिए खाद्य पदार्थों के स्रोत हैं।

इसलिए इन्हें ‘ भविष्य का भण्डार’ (Future Store-house) भी कहा जाता है।
1. महासागर तथा खाद्य संसाधन (Oceans and Food Resources ):
आदि मानव अपने भोजन की पूर्ति के लिए महासागरों पर निर्भर करता था आज भी महासागर खाद्य पदार्थों के अनन्त भण्डार हैं। संसार के अनेक क्षेत्रों में मानव भोजन के लिए मछली पर निर्भर है। मछली भोजन के कुल प्रोटीन का 10% भाग प्रदान करती है। पश्चिमी यूरोप, उत्तर-पूर्वी अमेरिका, जापान आदि देशों में मछली क्षेत्रों का आधुनिक स्तर पर विकास हुआ है। कई प्रकार के शैवालों से भी खाद्य पदार्थ प्राप्त होते हैं। निरन्तर बढ़ती हुई जनसंख्या के लिए भोजन प्रदान करने का सब से महत्त्वपूर्ण साधन भविष्य में महासागरों के मछली क्षेत्र ही होंगे।

2. महासागर तथा उपयोगी पदार्थ ( Oceans and Useful Materials):
महासागरों से अनेक उपयोगी पदार्थ मिलते हैं जो वनस्पति तथा जीवों से प्राप्त होते हैं। अनेक समुद्री जन्तु, तेल, रोएं, चमड़ा, सरेस, पशुओं के लिए चारे की वस्तुएं प्रदान करते हैं। कुछ समुद्री पौधों तथा जन्तुओं का प्रयोग दवाइयों के बनाने में भी किया जाता है। इस प्रकार महासागरों से अनेक पदार्थ सुलभ हैं। भविष्य में मानव को कई पदार्थों के लिए महासागरों पर आश्रित रहना पड़ेगा।

3. महासागर तथा खनिज संसाधन (Oceans and Mineral Resources):
महासागर अनेक उपयोगी धात्विक तथा अधात्विक खनिजों के भण्डार हैं। ये खनिज घोल तथा निलम्बित कणों के रूप में मिलते हैं। समुद्रों के खारे जल में नमक सब से महत्त्वपूर्ण पदार्थ है। अन्य पदार्थों में मैंगनीज, गंधक, ब्रोमीन, जिर्कन, मोनोजाइट, सोना, लोहा, बालू, बजरी महत्त्वपूर्ण हैं। महासागरों में कुछ खनिज पदार्थों का पुनः निर्माण होता रहता है जो भविष्य में मानव के लिए उपयोगी सिद्ध होंगे।

4. महासागर तथा खनिज तेल (Oceans and Petroleum ):
आधुनिक समय में महासागरों से खनिज तेल का प्राप्त होना सबसे महत्त्वपूर्ण उपलब्धि है। खनिज तेल मुख्यतः कम गहरे निमग्न चबूतरों में पाया जाता है। एक अनुमान के अनुसार संसार के खनिज तेल के भण्डार का 20% भाग महासागरीय नितल पर है। इस समय 75 से अधिक देशों में तट से दूर कम गहरे भागों में (Offshore Areas) में तेल निकाला जा रहा है। इन क्षेत्रों से प्राकृतिक गैस के भी भण्डार प्राप्त हुए हैं। ‘बम्बई हाई’ (Bombay High) भी ऐसा ही महत्त्वपूर्ण क्षेत्र है जो सागरीय तट से 150 किलोमीटर दूर स्थित है। अनुमान है कि तेल के भण्डार उत्तरी सागर, मैक्सिको की खाड़ी, वेन्जुएला तट तथा हिन्द महासागर में मौजूद हैं। भविष्य में खनिज तेल के भण्डार अधिक-से-अधिक 50 वर्षों तक पर्याप्त होंगे। अधिक खपत के कारण ये समाप्त हो जाएंगे। भविष्य में मानव को खनिज तेल के लिए महासागरों में खोज करनी पड़ेगी।

5. महासागर तथा ऊर्जा (Oceans and Energy):
आधुनिक खोज द्वारा महासागरीय जल से ऊर्जा प्राप्त करने के विभिन्न साधन खोज निकाले गए हैं। ज्वार भाटे के जल से ज्वारीय ऊर्जा (Tidal Power) प्राप्त की जाती है । सोवियत रूस, जापान तथा फ्रांस में ज्वारीय शक्ति उत्पन्न करने के कुछ केन्द्र हैं। महासागरों के ज्वालामुखियों के क्षेत्रों में भू- तापीय ऊर्जा (Geo-thermal Energy ) प्राप्त की जा रही है। बैल्जियम तथा क्यूबा में महासागरीय जल में तापांतर की सहायता से ऊर्जा प्राप्त की जाती है। इस प्रकार भविष्य में ऊर्जा संकट उत्पन्न होने से महासागर ऊर्जा प्रदान करने में सहायक सिद्ध होंगे।

6. महासागर तथा परिवहन साधन (Oceans and Means of Transportation ):
महासागर यातायात के प्राचीनतम, सबसे सस्ते तथा प्राकृतिक साधन हैं। संसार का अन्तर्राष्ट्रीय व्यापार बहुत हद तक समुद्री मार्गों पर निर्भर करता है। भविष्य में खनिज तेल व कोयले की कमी के समय मानव को महासागरीय परिवहन पर अधिक निर्भर रहना पड़ेगा।

JAC Class 11 Geography Important Questions Chapter 12 विश्व की जलवायु एवं जलवायु परिवर्तन

Jharkhand Board JAC Class 11 Geography Important Questions Chapter 12 विश्व की जलवायु एवं जलवायु परिवर्तन Important Questions and Answers.

JAC Board Class 11 Geography Important Questions Chapter 12 विश्व की जलवायु एवं जलवायु परिवर्तन

बहु-विकल्पी प्रश्न (Multiple Choice Questions)

दिए गए चार वैकल्पिक उत्तरों में से सही उत्तर चुनिए
1. जाइरे बेसिन में किस प्रकार की जलवायु मिलती है?
(A) मानसूनी
(B) रूम सागरीय
(C) भूमध्य रेखीय
(D) ध्रुवीय।
उत्तर;
(C) भूमध्य रेखीय।

2. शीत ऋतु की वर्षा वाला खण्ड कौन-सा है?
(A) भूमध्य रेखीय
(B) रूमं सागरीय।
(C) मानसून
(D) टुण्ड्रा।
उत्तर:
(B) रूमं सागरीय।

3. यदि वाष्पीकरण वर्षा से कम हो तो जलवायु कैसी होती है ?
(A) शुष्क
(B) आर्द्र
(C) शीत
(D) उष्ण।
उत्तर:
(B) आर्द्र

4. वाष्पोत्सर्जन में कौन-सी क्रिया होती है?
(A) महासागरों से वाष्पीकरण
(B) पेड़-पौधों से वाष्पीकरण
(C) जल वाष्प का सघनन
(D) धूल-कणों पर जल वाष्प का जमना।
उत्तर:
(B) पेड़-पौधों से वाष्पीकरण।

5. H शब्द किस प्रकार के प्रदेशों की जलवायु प्रकट करता है?
(A) भूमध्य रेखीय
(B) भूमध्य सागरीय
(C) उच्च भूमियां
(D) मानसून
उत्तर:
(C) उच्च भूमियां।

JAC Class 11 Geography Important Questions Chapter 12 विश्व की जलवायु एवं जलवायु परिवर्तन

6. निम्नलिखित में से कौन-सा मौसम तथा जलवायु का तत्त्व नहीं है?
(A) तापमान
(B) आर्द्रता
(C) दृश्यता
(D) वृष्टि।
उत्तर:
(C) दृश्यता।

7. ऊष्ण ऋतु से पूर्णतः रहित ध्रुवीय जलवायु को कोपेन ने दर्शाया है:
(A) A अक्षर द्वारा
(C) D अक्षर द्वारा
(B) B अक्षर द्वारा
(D) E अक्षर द्वारा।
उत्तर:
(D) E अक्षर द्वारा।

8. निम्नलिखित में से कौन-से जलवायु प्रकार की प्रमुख विशेषताएं ऊंचा तापमान, ऊंची सापेक्षिक आर्द्रता, सारा वर्ष होने वाली अधिक वर्षा और कम वार्षिक तापान्तर है?
(A) विषुवतीय जलवायु
(B) सवाना जलवायु
(C) मानसून जलवायु
(D) चीन तुल्य जलवायु।
उत्तर:
(A) विषुवतीय जलवायु।

9. भूमध्य सागरीय जलवायु की विशेषता है:
(A) वर्ष भर वर्षा
(B) मुख्यतः शीत ऋतु में वर्षा
(C) मुख्यतः ग्रीष्म ऋतु में वर्षा
(D) 10 सेंटीमीटर से कम वार्षिक वर्षा।
उत्तर:
(B) मुख्यत: शीत ऋतु में वर्षा।

10. कौन-सी जलवायु में वार्षिक तापान्तर सर्वाधिक होता है?
(A) सवाना जलवायु
(B) उष्ण मरुस्थलीय जलवायु
(C) स्टेपी जलवायु
(D) उच्च पर्वतीय जलवायु।
उत्तर:
(B) उष्ण मरुस्थलीय जलवायु।

अति लघु उत्तरीय प्रश्न (Very Short Answer Type Questions)

प्रश्न 1.
अमेज़न बेसिन में किस प्रकार की जलवायु मिलती है?
उत्तर:
भूमध्यरेखीय जलवायु।

प्रश्न 2.
शीत ऋतु की वर्षा वाला खण्ड कौन-सा है?
उत्तर:
भूमध्यसागरीय।

प्रश्न 3.
A शब्द किस प्रकार की जलवायु का प्रतीक है?
उत्तर:
आर्द्र उष्ण कटिबन्धीय जलवायु।

प्रश्न 4.
दो उष्ण मरुस्थलों के नाम लिखो।
उत्तर:
सहारा तथा थार।

प्रश्न 5.
आर्द्र उष्ण कटिबन्धीय जलवायु की सीमा कौन – सी समताप रेखा निर्धारित करती है?
उत्तर:
20°C समताप रेखा।

JAC Class 11 Geography Important Questions Chapter 12 विश्व की जलवायु एवं जलवायु परिवर्तन

प्रश्न 6.
उष्ण कटिबन्ध में कौन-सी तीन प्रकार की जलवायु मिलती है?
उत्तर:

  1. विषुवतीय रेखीय
  2. सवाना
  3. मानसूनी जलवायु।

प्रश्न 7.
शुष्क जलवायु के दो प्रकार बताओ।
उत्तर:
अर्ध-मरुस्थलीय, स्टेपी।

प्रश्न 8.
शुष्क मरुस्थल किन अक्षांशों के निकट मिलते हैं?
उत्तर:
कर्क रेखा तथा मकर रेखा।

प्रश्न 9.
भूमध्य सागरीय जलवायु खण्ड में शीत ऋतु की वर्षा का मुख्य कारण क्या है?
उत्तर:
वायुदाब पेटियों का खिसकना।

प्रश्न 10.
टैगा जलवायु में कौन-से वन मिलते हैं?
उत्तर:
कोणधारी वन।

प्रश्न 11.
ध्रुवीय जलवायु में उष्णतम मास का तापमान कितना होता है?
उत्तर:
10°C से कम।

JAC Class 11 Geography Important Questions Chapter 12 विश्व की जलवायु एवं जलवायु परिवर्तन

प्रश्न 12.
टैगा जलवायु खण्ड में न्यूनतम ताप कहां मापा गया है?
उत्तर:
वर्खोयांस्क ( 50°C)।

प्रश्न 13.
भारत किस प्रकार की जलवायु वाला देश है?
उत्तर:
उष्ण कटिबन्धीय मानसून जलवायु।

लघु उत्तरीय प्रश्न (Short Answer Type Questions)

प्रश्न 1.
जलवायु विज्ञान की परिभाषा दो।
उत्तर:
जलवायु विज्ञान (Climatology ):
पृथ्वी के चारों ओर वायु का एक विस्तृत आवरण फैला हुआ है। इन वायुमण्डलीय अवस्थाओं ( तापमान, वायुदाब, पवनें, आर्द्रता) का अध्ययन करने वाले शास्त्र को जलवायु विज्ञान कहते हैं। इनमें केवल वायुमण्डलीय क्रियाओं का ही नहीं अपितु जलवायु के विभिन्न तत्त्वों एवं नियन्त्रणों का अध्ययन भी किया जाता है।

प्रश्न 2.
विभिन्न प्रकार के जलवायविक वर्गीकरण के मुख्य आधारों के नाम लिखो।
उत्तर:
संसार की मुख्य जलवायु वर्गीकृत प्रकारों की पहचान इन तत्त्वों के आधार पर की जाती है

  1. तापमान (Temperature)
  2. वर्षा (Rainfall)
  3. वाष्पीकरण (Evaporation)
  4. वाष्पोत्सर्जन (Evapotranspiration)
  5. जल सन्तुलन (Water Balance)।

प्रश्न 3.
तीन प्रसिद्ध जलवायविक वर्गीकरण कौन-कौन से हैं?
उत्तर:
संसार में दो प्रसिद्ध भूगोलवेत्ताओं द्वारा प्रस्तुत किए गए तथा इन्हीं के नाम पर प्रसिद्ध वर्गीकरण निम्नलिखित हैं

  1. थार्नवेट वर्गीकरण (Thornthwaite Classification)
  2. कोपेन वर्गीकरण (Koppen Classification)
  3. ट्रिवार्था का वर्गीकरण

प्रश्न 4.
वृष्टि की एक निश्चित मात्रा आर्द्र और शुष्क जलवायु विभाजक सीमा क्यों नहीं होती?
उत्तर:
केवल वर्षा की मात्रा के आधार पर ही आर्द्र और शुष्क जलवायु की सीमा निर्धारित नहीं होती है। औसत वार्षिक वर्षा के साथ-साथ वर्षा का मौसमी वितरण देखा जाता है। तापमान तथा वाष्पीकरण का प्रभाव भी महत्त्वपूर्ण होता है। ये चारों कारक मिलकर किसी प्रदेश की शुष्क या आर्द्र जलवायु निर्धारित करते हैं।

प्रश्न 5.
कोपेन के जलवायु वर्गीकरण में किस प्रकार के जलवायु आंकड़े प्रयोग किये जाते हैं?
उत्तर:

  1. तापमान
  2. वर्षा
  3. वर्षा तथा तापमान का वनस्पति से सम्बन्ध।

प्रश्न 6.
किस प्रकार की जलवायु में वार्षिक तापान्तर कम-से-कम होता है?
उत्तर:
भूमध्यरेखीय खण्ड में वार्षिक तापान्तर सबसे कम होता है। ये प्रायः 5° सैंटीग्रेड से कम होता है। इस खण्ड में वर्ष भर समान रूप से वर्षा होती है तथा मेघ छाये रहते हैं, उच्च तापमान मिलते हैं तथा दिन-रात सदा समान होते हैं परिणामस्वरूप वार्षिक तापान्तर कम होता है।

JAC Class 11 Geography Important Questions Chapter 12 विश्व की जलवायु एवं जलवायु परिवर्तन

प्रश्न 7.
ग्रीष्मकाल में 10° C समताप रेखा का क्या महत्त्व है?
उत्तर:
टुण्ड्रा खण्ड में सबसे अधिक गर्म मास का तापमान 10° C से कम रहता है। ग्रीष्म ऋतु बहुत छोटी होती है तथा उपज काल भी बहुत छोटा होता है। यह समताप रेखा ध्रुवों की ओर वृक्षों की सीमा निर्धारित करती है। (limit of tree growth) 10° C से कम तापमान के कारण टुण्ड्रा खण्ड में वृक्ष नहीं होते।

प्रश्न 8.
पश्चिमी यूरोपीय जलवायु उत्तरी तथा दक्षिणी अमेरिका में केवल पतली समुद्र तटीय पट्टियों में ही क्यों पाई जाती है?
उत्तर:
उत्तरी तथा दक्षिणी अमेरिका में पश्चिमी यूरोपीय खण्ड एक तंग पट्टी के रूप में चिल्ली तथा कनाडा में मिलता है। निरन्तर ऊंचे रॉकीज तथा एण्डीज पर्वतों की रोक के कारण इस खण्ड का विस्तार सीमित है।

प्रश्न 9.
निम्न अक्षांशीय मरुस्थलीय जलवायु की तुलना स्टेपी जलवायु से करो।
उत्तर:

मरुस्थलीय जलवायु स्टेपी जलवायु
(1) मरुस्थलीय जलवायु 20 30 अक्षांशों के पश्चिमी भागों में मिलती है। (1) स्टेपी जलवायु 30 45 अक्षांशों में महाद्वीपों के अन्दरूनी भागों में पाई जाती है।
(2) इस जलवायु में औसत वार्षिक तापमान 38 रहता है। (2) इस जलवायु में औसत वार्षिक तापमान 20 रहता है।
(3) वार्षिक वर्षा 20 से० मी० से कम होती है। (3) औसत वार्षिक वर्षा 30 से० मी॰ से अधिक रहती है।
(4) प्राकृतिक वनस्पति का अभाव होता है। केवल कांटेदार झाड़ियां पाई जाती हैं। (4) यहां छोटी हरी घास मिलती है जिस पर पशु पालन होता है।


निबन्धात्मक प्रश्न (Essay Type Questions)

प्रश्न 1.
कोपेन द्वारा जलवायु वर्गीकरण की पद्धति का वर्णन करो तथा प्रत्येक जलवायु प्रकार का संक्षिप्त वर्णन करें।
उत्तर:
कोपेन की जलवायु वर्गीकरण की पद्धति ब्लादिमीर कोपेन द्वारा विकसित की गई जलवायु के वर्गीकरण की आनुभाविक पद्धति का सबसे व्यापक उपयोग किया जाता है। कोपेन ने वनस्पति के वितरण और जलवायु के बीच एक घनिष्ठ सम्बन्ध की पहचान की। उन्होंने तापमान तथा वर्षण के कुछ निश्चित मानों का चयन करते हुए उनका वनस्पति के वितरण से संबंध स्थापित किया और इन मानों का उपयोग जलवायु के वर्गीकरण के लिए किया । वर्षा एवं तापमान के मध्यमान वार्षिक एवं मध्यमान मासिक आँकड़ों पर आधारित यह एक आनुभाविक पद्धति है।

उन्होंने जलवायु के समूहों एवं प्रकारों को पहचान करने के लिए बड़े तथा छोटे अक्षरों के प्रयोग का आरंभ किया। सन् 1918 में विकसित तथा समय के साथ संशोधित हुई कोपेन की यह पद्धति आज भी लोकप्रिय और प्रचलित है। कोपेन ने पाँच प्रमुख जलवायु समूह निर्धारित किए जिनमें से चार तापमान पर और एक घर्षण पर आधारित है बड़े अक्षरों का प्रयोग
बड़े अक्षर A, C, D तथा E आर्द्र जलवायुओं को तथा B अक्षर शुष्क जलवायुओं को निरूपित करता है।

सारणी : कोपेन के अनुसार जलवायु समूह

समूह लक्षण
A. उष्णकटिबंधीय सभी महीनों का औसत तापमान 18° सेल्सियस से अधिक-आर्द्र जलवायु।
B. शुष्क जलवायु औसत वार्षिक वर्षा (से॰मी०) औसत वार्षिक तापमान (° सेल्सियस) के दुगुने से कम।
C. कोष्ण शीतोष्ण सर्वाधिक ठंडे महीने का औसत तापमान 3° सेल्सियस से अधिक किन्तु 18° सेल्सियस से कम मध्य अक्षांशीय जलवायु।
D. शीतल हिम-वन वर्ष के सर्वाधिक ठंडे महीने का औसत तापमान. शून्य डिग्री तापमान से 3° नीचे।
E. शीत सभी महीनों का औसत तापमान 10° सेल्सियस से कम।
H. उच्चभूमि ऊँचाई के कारण सर्द।

छोटे अक्षरों का प्रयोग
जलवायु समूहों को तापक्रम एवं वर्षा की मौसमी विशेषताओं के आधार पर कई उप-प्रकारों में विभाजित किया गया है जिसको छोटे अक्षरों द्वारा अभिहित किया गया है। शुष्कता वाले मौसमों को छोटे अक्षरों f, m, w और s द्वारा इंगित किया गया है। इसमेंfशुष्क मौसम के न होने को, m मानसून जलवायु को, w शुष्क शीत ऋतु को और s शुष्क ग्रीष्म ऋतु को इंगित करता है। छोटे अक्षर a, b, c तथा d तापमान की उग्रता वाले भाग को दर्शाते हैं। B समूह की जलवायुओं को उपविभाजित करते हुए स्टैपी अथवा अर्ध-शुष्क के लिए S तथा मरुस्थल के लिए W जैसे बड़े अक्षरों का प्रयोग किया गया है।

समूह प्रकार कुट अक्षर लक्षण
(A) उष्णकटिबंधीय आर्द्र जलवायु उष्णकटिबंधीय आर्द्र Af कोई शुष्क ऋतु नहीं।
उष्णकटिबंधीय मानसून Am मानसून, लघु शुष्क ॠतु
उष्पकटिबंधीय आर्द्र एवं शुष्क Aw जाड़े की शुष्क ऋतु
(B) शुष्क जलवायु उपोष्ण कटिबंधीय स्टैपी BSh निम्न अक्षांशीय अर्द्ध शुष्क एवं शुष्क
उपोष्ण कटिबंधीय मरुस्थल BWh निम्न अक्षांशीय शुष्क
मध्य अक्षांशीय स्टैपी BSk मध्य अक्षांशीय अर्ध शुष्क अथवा शुष्क
मध्य अक्षांशीय मरुस्थल BWk मध्य अक्षांशीय शुष्क
(C) कोष्ण शीतोष्ण (मध्य अक्षांशीय जलवायु) आर्द्र उपोष्ण कटिबंधीय Cfa मध्य अक्षांशीय अर्द्ध शुष्क अथवा शुष्क
भूमध्य सागरीय Csa शुष्क गर्म ग्रीष्म
समुद्री पश्चिमी तटीय Cfb एवं CFc कोई शुष्क ऋतु नहीं, कोष्ण तथा शीतल ग्रीष्म
(D) शीतल हिम-वन जलवायु आर्द्र महाद्वीपीय Df कोई शुष्क ऋतु नहीं, भीषण जाड़ा
उप-उत्तर ध्रुवीय DW जाड़ा शुष्क तथा अत्यंत भीषण
(E) शीत जलवायु टुंड्रा Et सही अर्थों में कोई ग्रीष्म नहीं
ध्रुवीय हिमटोपी Ef सदैव हिमाच्छादित हिम
(F) उच्चभूमि उच्च भूमि H हिमाच्छादित उच्च भूमियाँ

JAC Class 11 Geography Important Questions Chapter 12 विश्व की जलवायु एवं जलवायु परिवर्तन

प्रश्न 2.
जलवायु परिवर्तन से क्या अभिप्राय है? कारण स्पष्ट करें।
उत्तर:
जलवायु परिवर्तन
जिस प्रकार की जलवायु का अनुभव हम अब कर रहे हैं वह थोड़े बहुत उतार-चढ़ाव के साथ विगत 10 हज़ार वर्षों से अनुभव की जा रही है। अपने प्रादुर्भाव से ही पृथ्वी ने जलवायु में अनेक परिवर्तन देखे हैं। भूगर्भिक अभिलेखों से हिमयुगों और अंतर – हिमयुगों में क्रमशः परिवर्तन की प्रक्रिया परिलक्षित होती है। भू-आकृतिक लक्षण, विशेषतः ऊँचाइयों तथा उच्च अक्षांशों में हिमनदियों के आगे बढ़ने व पीछे हटने के शेष चिह्न प्रदर्शित करते हैं। हिमानी निर्मित झीलों में अवसादों का निक्षेपण उष्ण एवं शीत युगों के होने को उजागर करता है। वृक्षों के तनों में पाए जाने वाले वलय भी आर्द्र एवं शुष्क युगों की उपस्थिति का संकेत देते हैं । ऐतिहासिक अभिलेख भी जलवायु की अनिश्चितता का वर्णन करते हैं। ये सभी साक्ष्य इंगित करते हैं कि जलवायु परिवर्तन एक प्राकृतिक एवं सतत प्रक्रिया है।

भारत में जलवायु
भारत में भी आर्द्र एवं शुष्क युग आते-जाते रहे हैं। पुरातत्व खोजें दर्शाती हैं कि ईसा से लगभग 8,000 वर्ष पूर्व राजस्थान मरुस्थल की जलवायु आर्द्र एवं शीतल थी। ईसा से 3,000 से 1,700 वर्ष पूर्व यहाँ वर्षा अधिक होती थी लगभग 2,000 से 1,700 वर्ष ईसा पूर्व यह क्षेत्र हड़प्पा संस्कृति का केन्द्र था। शुष्क दशाएँ तभी से गहन हुई हैं। लगभग 50 करोड़ से 30 करोड़ वर्ष पहले भू-वैज्ञानिक काल के कूम्ब्रियन, आर्डोविसियन तथा सिल्युरिसन युगों में पृथ्वी गर्म थी। प्लीस्टोसीन युगांतर के दौरान हिमयुग और अंतर हिमयुग अवधियां रही हैं। अंतिम प्रमुख हिमयुग आज से 18,000 वर्ष पूर्व था। वर्तमान अंतर हिमयुग 10,000 वर्ष पूर्व आरंभ हुआ था।

अभिनव पूर्व काल में जलवायु:
सभी कालों में जलवायु परिवर्तन होते रहे हैं। पिछली शताब्दी के 90 के दशक में चरम मौसमी घटनाएँ घटित हुई हैं। 1990 के दशक में शताब्दी का सबसे गर्म तापमान और विश्व में सबसे भयंकर बाढ़ों को दर्ज किया है। सहारा मरुस्थल के दक्षिण में स्थित साहेल प्रदेश में 1967 से 1977 के दौरान आया विनाशकारी सूखा ऐसा ही एक परिवर्तन था। 1930 के दशक में संयुक्त राज्य अमेरिका के वृहत मैदान के दक्षिण-पश्चिमी भाग में, जिसे ‘धूल का कटोरा’ कहा जाता है, भीषण सूखा पड़ा।

फसलों की उपज अथवा फसलों के विनाश, बाढ़ों तथा लोगों के प्रवास संबंधी ऐतिहासिक अभिलेख परिवर्तनशील जलवायु के प्रभावों के बारे में बताते हैं। यूरोप अनेकों बार उष्ण, आर्द्र, शीत एवं शुष्क युगों से गुज़रा है। इनमें से महत्त्वपूर्ण प्रसंग 10वीं और 11वीं शताब्दी की उष्ण एवं शुष्क दशाओं का है, जिनमें बाइकिंग कबीले ग्रीनलैंड में जा बसे थे। यूरोप ने सन् 1550 से सन् 1850 के दौरान लघु हिम युग का अनुभव किया है। 1885 से 1940 तक विश्व के तापमान में वृद्धि की प्रवृत्ति पाई गई है । 1940 के बाद तापमान में वृद्धि की दर है ।

जलवायु परिवर्तन के कारण: जलवायु परिवर्तन के अनेक कारण हैं। इन्हें खगोलीय और पार्थिव कारणों में वर्गीकृत किया जा सकता है।

  1. खगोलीय कारणों का सम्बन्ध सौर कलंकों की गतिविधियों से उत्पन्न सौर्थिक निर्गत ऊर्जा में परिवर्तन से है। सौर कलंक सूर्य पर काले धब्बे होते हैं, जो एक चक्रीय ढंग से घटते-बढ़ते रहते हैं । कुछ मौसम वैज्ञानिकों के अनुसार सौर कलंकों की संख्या बढ़ने पर मौसम ठंडा और आर्द्र हो जाता है और तूफानों की संख्या बढ़ जाती है। सौर कलंकों की संख्या घटने से उष्ण एवं शुष्क दशाएँ उत्पन्न होती हैं यद्यपि ये खोजे आँकड़ों की दृष्टि से महत्त्वपूर्ण नहीं हैं।
  2. एक अन्य खगोलीय सिद्धांत ‘मिलैंकोविच दोलन’ है, जो सूर्य के चारों ओर पृथ्वी के कक्षीय लक्षणों में बदलाव के चक्रों, पृथ्वी की डगमगाहट तथा पृथ्वी के अक्षीय झुकाव में परिवर्तनों के बारे में अनुमान लगाता है । ये सभी कारक सूर्य से प्राप्त होने वाले सूर्यातप में परिवर्तन ला देते हैं जिसका प्रभाव जलवायु पर पड़ता है।
  3. ज्वालामुखी क्रिया जलवायु परिवर्तन का एक अन्य कारण है। ज्वालामुखी उद्भेदन वायुमंडल में बड़ी मात्रा में ऐरोसोल फेंक देता है। ये ऐरोसोल लंबे समय तक वायुमंडल में विद्यमान रहते हैं और पृथ्वी की सतह पर पहुँचने वाले सौर्यिक विकिरण को कम कर देते हैं । हाल ही में हुए पिनाटोबा तथा एल सियोल ज्वालामुखी उद्भेदनों के बाद पृथ्वी का औसत तापमान कुछ हद तक गिर गया था
  4. जलवायु पर पड़ने वाला सबसे महत्त्वपूर्ण मानवोद्भवी कारण वायुमंडल में ग्रीन हाऊस गैसों का बढ़ता सांद्रण है। इससे भूमंडलीय तापन हो सकता है।

भूमंडलीय तापन
ग्रीन हाऊस गैसों की उपस्थिति के कारण वायुमंडल एक हरित गृह की भांति व्यवहार करता है। वायुमंडल प्रवेशी सौर विकिरण का पोषण भी करता है किन्तु पृथ्वी की सतह से ऊपर की ओर उत्सर्जित होने वाली अधिकतम दीर्घ तरंगों को अवशोषित कर लेता है। वे गैसें जो विकिरण की दीर्घ तरंगों का अवशोषण करती हैं, हरित गृह गैसें कहलाती हैं। वायुमंडल का तापन करने वाली प्रक्रियाओं को सामूहिक रूप से ‘हरित गृह प्रभाव’ (Green house effect) कहा जाता है।

प्रश्न 3.
भूमण्डलीय ऊष्मन की व्याख्या कीजिए।
उत्तर:
भूमण्डलीय ऊष्मन (Global Warming): भूमण्डलीय ऊष्मन का अर्थ है पृथ्वी के औसत तापमान में वायुमण्डल को गर्म करने के साधन वायुमण्डलीय गैसों के परमाणु एवं अणु ग्रीन हाउस गैसों विशेषकर जल, कार्बन डाइऑक्साइड तथा मेथैन द्वारा सूर्य प्रकाश का अवशोषण तथा पश्च विकिरण करते हैं। महासागरों से होने वाले वाष्पन से वायुमण्डल में जल का संकेंद्रण नियंत्रित होता है। वायुमण्डल में कार्बन डाइऑक्साइड ज्वालामुखी क्रिया द्वारा लाया जाता है। कार्बन डाइऑक्साइड की उतनी ही मात्रा वर्षण द्वारा हटा दी जाती है और महासागरों में कैल्शियम कार्बोनेट के रूप में जमा कर दी जाती है। मेथैन, जो कार्बन डाइऑक्साइड से बीस गुना अधिक प्रभावी है, लकड़ी में बैक्टीरिया के उपापचय तथा घास चरने वाले पशुओं द्वारा उत्पन्न की जाती है। मेथैन का बड़ी शीघ्रता से कार्बन डाइऑक्साइड के रूप में ऑक्सीकरण होता है।

मानवीय क्रियाओं का प्रभाव:
मानवीय क्रियाओं, जैसे जीवाश्मी तेल को जलाने तथा विभिन्न कृषीय क्रियाओं द्वारा मेथैन एवं कार्बन डाइऑक्साइड वायुमंडल में जमा की जा रही है। वायुमण्डल में कार्बन डाइऑक्साइड की मात्रा संपूर्ण विश्व की जलवायु को बदलने में मुख्य भूमिका अदा करती है। यह गैस सूर्यातप के लिए पारदर्शी है, लेकिन बाहर जाने वाले दीर्घ तरंगी पार्थिव विकिरण को अवशोषित कर लेती है। अवशोषित पार्थिव विकिरण भूपृष्ठ पर वापस विकिरित कर दिया जाता है। इस प्रकार यह स्पष्ट हो जाता है कि कार्बन डाइऑक्साइड की मात्रा में कोई भी उल्लेखनीय परिवर्तन वायुमण्डल के निचले स्तर के तापमान में परिवर्तन लाएगा।

तीव्र औद्योगीकरण तथा कृषि और परिवहन क्षेत्रों में हुई तकनीकी क्रांति के फलस्वरूप वायुमण्डल में बड़े पैमाने पर कार्बन डाइऑक्साइड, मेथैन तथा क्लोरोफ्लोरो कार्बन गैसें पहुंचाई जाती हैं। इनमें से कुछ गैस वनस्पति द्वारा उपभोग कर ली जाती हैं तथा कुछ भाग महासागरों में घुल जाता है। फिर भी लगभग 50 प्रतिशत भाग वायुमण्डल में बच जाता है।

  1. पिछले 100 वर्षों में, मेथैन का संकेंद्रण दुगुने से अधिक बढ़ गया है, (7.0 × 10-7 से 15.5 × 10-7 तक)।
  2. कार्बन डाइऑक्साइड की मात्रा 20 प्रतिशत से अधिक (2.90 × 10-4 से 3.49 × 10-4 तक) हो गई है।
  3. 1880-1890 में कार्बन डाइऑक्साइड की मात्रा लगभग 290 भाग प्रति दस लाख थी, जो बढ़ कर 1980 में 315 भाग प्रति दस लाख, 1990 में 340 भाग प्रति दस लाख और 2000 से 400 भाग प्रति दस लाख हो गई है।
  4. इसका अर्थ यह हुआ कि वायुमंडल में कार्बन डाइऑक्साइड का अनुपात बढ़कर 1950 तक 9 प्रतिशत तथा 1990 तक लगभग 17 प्रतिशत अधिक हो गया है। गत दशक में इसकी वृद्धि की दर और भी बढ़ गई है

औद्योगीकरण का प्रभाव:
अनेकों जलवायविक प्राचलों में से तापमान नगरीकरण तथा औद्योगीकरण के परिणामस्वरूप सबसे अधिक प्रभावित हुआ है। नगरीय क्षेत्रों की तापीय विशेषताएं समीपवर्ती ग्रामीण क्षेत्रों से काफ़ी भिन्न हैं। गत 50 वर्षों के तापमान आंकड़ों के विश्लेषण से यह स्पष्ट होता है कि भारत में शीत ऋतु में तापमान में 0.7° से. तथा ग्रीष्म ऋतु में 1.4° से० बढ़ जाता है।

कृषि का प्रभाव:
मानव जलवायु परिवर्तन का एक इंजन समझा जाता है। उदाहरणार्थ चावल उत्पादन करने वाले किसान, कोयला खनिक, डेयरी में लगे लोग तथा स्थानांतरी कृषक भी भूमंडलीय ऊष्मन में अपना-अपना योगदान देते हैं। कुछ अनुमानों के अनुसार विश्व में चावल का उत्पादन 20 प्रतिशत मेथैन तथा कोयला खनन 6 प्रतिशत मेथैन वायुमंडल में जोड़ता है। स्थानांतरी खेती के फलस्वरूप होने वाले वनों के कटाव से 20 प्रतिशत कार्बन डाइऑक्साइड गैस वायुमंडल में जमा कर दी जाती है। इसी प्रकार औद्योगीकरण द्वारा 25 प्रतिशत क्लोरोफ्लोरो कार्बन गैस वायुमंडल के ऐरोसॉल में जोड़ दी जाती है। फलस्वरूप भूमंडलीय तापमान वृद्धि लगभग 1.5° से० है।

वायुमंडलीय ऊष्मन के प्रभाव समुद्र तल के जल का ऊपर उठना:
आज इस बात के लिए काफ़ी चिंता जताई जा रही है कि कार्बन डाइऑक्साइड तथा मेथेन गैस की वायुमंडल में निरंतर वृद्धि से तापमान इस सीमा तक बढ़ जाएगा कि इससे ग्रीनलैंड तथा अंटार्कटिक महाद्वीप में बर्फ पिघलना आरंभ हो जाएगा। फलत: समुद्र तल ऊपर उठेगा जिससे तटीय भाग तथा द्वीप डूब जाएंगे। इससे वाष्पन एवं वर्षा के प्रतिरूपों में परिवर्तन आएगा, पौधों की नई-नई बीमारियां तथा नाशक जीवों की समस्याएं खड़ी होंगी और अंटार्कटिका के ऊपर स्थित ओजोन छिद्र बड़ा हो जाएगा।

अतीत में हुए जलवायविक परिवर्तनों की भरोसेमंद तस्वीर प्राप्त करने के उद्देश्य से अनेक देशों में विशेषकर अंटार्कटिक तथा ग्रीनलैंड की हिम टोपियों में पिछले 1,00,000 वर्षों के दौरान बर्फ़ में फंसी गैसों का विश्लेषण करने के लिए बर्फ कोरिंग कार्यक्रम को लिया गया है। इसके परिणाम बड़े रोचक निकले हैं और भूमंडलीय ऊष्मन की घटना से आगे बढ़कर पृथ्वी के अभिनव इतिहास की झलक दिखाते हैं। पृथ्वी के इतिहास के पिछले 10,000 वर्षों में जलवायु की प्रवृत्ति उसके पहले के वर्षों की तुलना में विशेष रूप से स्थिर रही है। ग्रीनलैंड के बर्फ़-कोर में ऑक्सीजन समस्थानिक अभिलेखों के अध्ययन से यह पता चलता है कि उत्तरी गोलार्द्ध में शीतलन प्रवृत्ति 1725 से 1920 तक चली। इनका संबंध ज्वालामुखी राख के निष्कासन से रहा, जो दो या तीन दशकों के नियमित अंतराल पर होता रहा लेकिन 1945 के बाद किसी प्रमुख ज्वालामुखी विस्फोट तथा वायुमंडल में कार्बन डाइऑक्साइड संकेंद्रण की मात्रा में वृद्धि के बिना ही भूमंडलीय तापमान में वृद्धि से ऊष्मन शुरू हुआ है।

भविष्य: वैज्ञानिकों की भविष्यवाणी है कि 2020 तक समस्त विश्व में पिछले 1,000 वर्षों की तुलना में तापमान अधिक होगा। इस प्रकार यह स्पष्ट है कि कार्बन डाइऑक्साइड की बढ़ती हुई मात्रा भूमंडलीय तापमान को बढ़ाने का कार्य करेगी।

JAC Class 11 Geography Important Questions Chapter 8 वायुमंडल का संघटन तथा संरचना 

Jharkhand Board JAC Class 11 Geography Important Questions Chapter 8 वायुमंडल का संघटन तथा संरचना Important Questions and Answers.

JAC Board Class 11 Geography Important Questions Chapter 8 वायुमंडल का संघटन तथा संरचना

बहु-विकल्पी प्रश्न (Multiple Choice Questions)

प्रश्न-दिए गए प्रश्नों के चार वैकल्पिक उत्तरों में से सही उत्तर चुनकर लिखो
1. वायुमण्डल का 99 प्रतिशत भाग कौन-सी गैसों द्वारा बना है?
(A) नाइट्रोजन व ऑक्सीजन
(B) हाइड्रोजन व ऑक्सीजन
(C) ऑक्सीजन व आर्गन
(D) नाइट्रोजन व हाइड्रोजन।
उत्तर:
नाइट्रोजन व ऑक्सीजन

2. वायुमण्डल की सबसे निचली परत को कहते हैं
(A) मध्यमण्डल
(B) आयनमण्डल
(C) क्षोभमण्डल
(D) बाह्यमण्डल।
उत्तर:
क्षोभमण्डल।

3. प्रकाश की क्या गति है?
(A) 3 लाख कि० मी० प्रति सैं०
(B) 5000 कि० मी० प्रति सैं०
(C) 10 कि० मी० प्रति सैं०
(D) 100 कि० मी० प्रति सैं।
उत्तर:
(A) 3 लाख कि० मी० प्रति सैं।

JAC Class 11 Geography Important Questions Chapter 8 वायुमंडल का संघटन तथा संरचना

4. वायुमण्डल का 99% द्रव्यमान कितनी ऊंचाई तक है?
(A) 12 कि० मी०
(B) 22 कि० मी०
(C) 32 कि० मी०
(D) 42 कि० मी०।
उत्तर:
32 कि० मी०।

5. कितनी ऊंचाई पर ऑक्सीजन गैस नगण्य हो जाती है?
(A) 100 कि० मी०
(B) 110 कि० मी०
(C) 120 कि० मी०
(D) 130 कि० मी०।
उत्तर:
120 कि० मी०।

6. कार्बन डाइऑक्साइड गैस कितनी ऊंचाई तक सीमित है?
(A) 70 कि० मी०
(B) 80 कि० मी०
(C) 90 कि० मी०
(D) 100 कि० मी०।
उत्तर:
90 कि० मी०।

7. वायुमण्डल में ऑक्सीजन की मात्रा कितनी % है?
(A) 15.95%
(B) 17.95%
(C) 20.95%
(D) 25.95%।
उत्तर:
(C) 20.95%।

8. वायुमण्डल में नाइट्रोजन गैस की मात्रा कितनी है?
(A) 72.08%
(B) 74.08%
(C) 76.08%
(D) 78.08%।
उत्तर:
(D) 78.08%।

9. मानव जीवन के लिए आवश्यक है-
(A) नाइट्रोजन
(B) ऑक्सीजन
(C) आर्गन
(D) ओज़ोन।
उत्तर:
(D) ओज़ो।

10. पौधों के लिए आवश्यक गैस है-
(A) कार्बन डाइऑक्साइड
(B) ऑक्सीजन।
(C) नाइट्रोजन
(D) आर्गन।
उत्तर:
(A) कार्बन डाइऑक्साइड।

11. कौन-सी गैस सौर विकिरण को सोख लेती है?
(A) ऑक्सीजन
(B) आर्गन
(C) ओज़ोन
(D) कार्बन डाइऑक्साइड।
उत्तर:
(D) कार्बन डाइऑक्साइड।

JAC Class 11 Geography Important Questions Chapter 8 वायुमंडल का संघटन तथा संरचना

12. कौन-सी गैस ग्रीन हाऊस गैस है?
(A) कार्बन डाइ ऑक्साइड
(B) ओज़ोन
(C) ऑक्सीजन
(D) नाइट्रोजन।
उत्तर:
(A) कार्बन डाइऑक्साइड।

अति लघु उत्तरीय प्रश्न (Very Short Answer Type Questions)

प्रश्न 1.
वायुमण्डल से क्या अभिप्राय है?
उत्तर:
धरती के गिर्द गैसों का आवरण।

प्रश्न 2.
वायुमण्डल की लगभग कितनी ऊंचाई है?
उत्तर:
1000 किलोमीटर से अधिक।

प्रश्न 3.
वायुमण्डल में मुख्य दो गैसें कौन-कौन सी हैं?
उत्तर:
ऑक्सीजन तथा नाइट्रोजन।

प्रश्न 4.
वायुमण्डल में नाइट्रोजन गैस कितने प्रतिशत है?
उत्तर:
78%.

प्रश्न 5.
वायुमण्डल में ऑक्सीजन गैस कितने प्रतिशत है?
उत्तर:
21%

JAC Class 11 Geography Important Questions Chapter 8 वायुमंडल का संघटन तथा संरचना

प्रश्न 6.
वायुमण्डल पृथ्वी के साथ क्यों सटा रहता है?
उत्तर:
गुरुत्वाकर्षण के कारण।

प्रश्न 7.
वायुमण्डल क्यों महत्त्वपूर्ण है?
उत्तर:
इसके कारण पृथ्वी पर जीवन मौजूद है।

प्रश्न 8.
वायुमण्डल की सबसे निचली परत को क्या नाम दिया गया है?
उत्तर:
क्षोभमण्डल।

प्रश्न 9.
वायुमण्डल की ऊपरी परत में पाई जाने वाली दो गैसों के नाम लिखो।

उत्तर:
आर्गन, हीलियम।

प्रश्न 10.
ओज़ोन गैस किन किरणों को सोख लेती है?
उत्तर:
सूर्य से निकलने वाली पराबैंगनी किरणें।

प्रश्न 11.
वायुमण्डल की कौन-सी परत मौसम की रचना करती है?
उत्तर:
क्षोभमण्डल।

प्रश्न 12.
तापमान की सामान्य घटन दर क्या है?
उत्तर:
165 मीटर के लिये 1°C.

प्रश्न 13.
किस परत पर वायुमण्डलीय विघ्न पाए जाते हैं?
उत्तर:
क्षोभमण्डल में।

JAC Class 11 Geography Important Questions Chapter 8 वायुमंडल का संघटन तथा संरचना

प्रश्न 14.
किस परत पर स्थिर तापमान रहता है?
उत्तर:
समताप मण्डल।

प्रश्न 15.
वायुमण्डल की किस परत को समताप मण्डल कहा जाता है?
उत्तर;
सट्रेटोस्फीयर ( तापमान की स्थिरता होने के कारण)।

लघु उत्तरीय प्रश्न (Short Answer Type Questions)

प्रश्न 1.
वायुमण्डल का क्या महत्त्व है?
उत्तर:
वायुमण्डल मानवीय जीवन पर कई प्रकार से प्रभाव डालता है।

  1. ऑक्सीजन गैस पृथ्वी पर मानवीय जीवन का आधार है।
  2. कार्बन डाइऑक्साइड वनस्पति जीवन का आधार है।
  3. वायुमण्डल सूर्यातप को जज़ब करके एक Glass House का काम करता है।
  4. वायुमण्डल का जलवाष्प वर्षा का मुख्य साधन है।
  5. वायुमण्डल फसलों, मौसम, जलवायु तथा वायुमार्गों पर प्रभाव डालता है।

प्रश्न 2.
आयनमण्डल पर टिप्पणी लिखो।
उत्तर:
यह धरातल के ऊपर वायुमण्डल का चौथा संस्तर है। इसकी ऊंचाई 80 से 400 कि० मी० के मध्य है। इस मण्डल में तापमान फिर से ऊंचाई के साथ बढ़ता है। यहां की हवा विद्युत् आवेशित होती है। रेडियो तरंगें इसी मण्डल से परिवर्तित
हो कर पुनः पृथ्वी पर लौट जाती हैं। यह परत रेडियो प्रसारण में उपयोगी है।

प्रश्न 3.
क्षोभमण्डल सीमा (Tropopause) से क्या अभिप्राय है?
उत्तर:
ऊंचाई के साथ-साथ तापमान में एक असमान दर से परिवर्तन होता है।

  1. 15 कि० मी० तक तापमान धीमी गति से कम होता है।
  2. 80 कि० मी० तक तापमान स्थिर रहता है।
  3. 80 कि० मी० से ऊपर तापमान में वृद्धि होने लगती है।

इस ऊंचाई के पश्चात् अर्थात् क्षोभमण्डल से ऊपर समताप मण्डल का भाग आरम्भ होता है। समताप मण्डल तथा क्षोभमण्डल को अलग करने वाले संक्रमण क्षेत्र को क्षोभमण्डल सीमा कहते हैं।

JAC Class 11 Geography Important Questions Chapter 8 वायुमंडल का संघटन तथा संरचना

प्रश्न 4.
कार्बन डाइऑक्साइड गैस का महत्त्व बताओ।
उत्तर:
कार्बन डाइऑक्साइड अन्तरिक्ष विज्ञान की दृष्टि से बहुत ही महत्त्वपूर्ण गैस है, क्योंकि यह सौर विकिरण के लिए पारदर्शी है, लेकिन पार्थिव विकिरण के लिए अपारदर्शी है। यह सौर विकिरण के एक अंश को सोख लेती है तथा इसके कुछ भाग को पृथ्वी की सतह की ओर प्रतिबिम्बित कर देती है। यह ग्रीन हाऊस प्रभाव के लिए पूरी तरह उत्तरदायी है। दूसरी गैसों का आयतन स्थिर है, जबकि पिछले कुछ दशकों में मुख्यतः जीवाश्म ईंधन को जलाये जाने के कारण कार्बन डाइऑक्साइड के आयतन में लगातार वृद्धि हो रही है। इसने हवा के ताप को भी बढ़ा दिया है।

प्रश्न 5.
वायुमण्डल में धूल कणों का वितरण बताओ।
उत्तर:
धूलकण वायुमण्डल में छोटे-छोटे कणों को भी रखने की क्षमता होती है। ये छोटे कण विभिन्न स्रोतों जैसे – समुद्री नमक, महीन मिट्टी, धुएं की कालिमा, राख, पराग, धूल तथा तारे के टूटे हुए कण से निकलते हैं। धूलकण प्रायः वायुमण्डल के निचले भाग में मौजूद होते हैं, फिर भी संवहनीय वायु प्रवाह इन्हें काफ़ी ऊंचाई तक ले जा सकता है। धूलकणों का सबसे अधिक जमाव उपोष्ण और शीतोष्ण प्रदेशों में सूखी हवा के कारण होता है, जो विषुवत् रेखीय और ध्रुवीय प्रदेशों की तुलना में यहां अधिक मात्रा में होती है। धूल और नमक के कण द्रवग्राही केन्द्र की तरह कार्य करते हैं जिसके चारों ओर जलवाष्प संघनित होकर मेघों का निर्माण करते हैं।

प्रश्न 6.
वायुमण्डल में धूल-कणों (Dust Particles) का क्या महत्त्व है?
उत्तर:
वायुमण्डल में धूल-कण निचले भागों में पाए जाते हैं। वायुमण्डल में धूल-कणों का कई प्रकार से विशेष महत्त्व है।

  1. धूल-कण सौर ताप का कुछ भाग सोख लेते हैं तथा कुछ भाग का परावर्तन हो जाता है। ताप सोख लेने के कारण वायुमण्डल का तापक्रम अधिक हो जाता है।
  2. धूल-कण आर्द्रताग्राही नाभि के रूप में काम करते हैं। इनके चारों ओर जलवाष्प का संघनन होता है जिससे वर्षा, कोहरा, बादल बनते हैं। धूल-कणों के अभाव के कारण वर्षा नहीं हो सकती
  3. धूल-कणों के कारण वायुमण्डल की दर्शन क्षमता (Visibility) कम होती है तथा धुंधलापन छा जाता है। धूल-कणों के संयोग से कई रंग-बिरंगे दृश्य सूर्य उदय, सूर्य अस्त तथा इन्द्र धनुष दृश्य बनते हैं।

प्रश्न 7.
ओज़ोन परत पर एक नोट लिखो ओजोन परत के घटने के क्या कारण हैं? इससे क्या हानि है?
उत्तर:
ऊपरी वायुमण्डल में ओजोन गैस की एक मोटी परत मिलती है। यह परत पराबैंगनी किरणों को सोख लेती है। परन्तु कार्बन तथा रसायनों के अधिक प्रयोग के कारण यह परत कम हो रही है। अणु शक्ति के परीक्षणों से भी यह परत घट गई है। 1980 में अण्टार्कटिका महाद्वीप के ऊपर इस परत में एक सुराख देखा गया है। इस सुराख के कारण पराबैंगनी किरणें पृथ्वी पर पहुंच सकती हैं। इन किरणों से मनुष्य अन्धे हो जाते हैं तथा शरीर झुलस जाता है

प्रश्न 8.
जलवाष्प का महत्त्व बताओ।
उत्तर:
जलवाष्प (Water Vapours ): वायुमण्डल में लगभग 4% मात्रा में जलवाष्प पाया जाता है। ऊँचाई के साथ जलवाष्प की मात्रा कम होती जाती है। कुछ जलवाष्प का लगभग आधा हिस्सा दो हज़ार मीटर ऊंचाई के नीचे ही पाया जाता है। जलवाष्प तापमान पर भी निर्भर करता है। भूमध्य रेखा से ध्रुवों की ओर जलवाष्प की मात्रा कम होती जाती है। पृथ्वी पर वृष्टि तथा संघनन का मुख्य स्रोत जलवाष्प ही है। सूर्यतप को सोख कर जलवाष्प तापमान का नियन्त्रण करता है

प्रश्न 9.
क्षोभ सीमा की ऊँचाई भूमध्य रेखा पर अधिक क्यों होती है?
उत्तर:
क्षोभ सीमा की ऊंचाई में विभिन्नता पाई जाती है। ध्रुवों पर यह 8 कि० मी० है। भूमध्य रेखा पर सबसे अधिक सूर्याताप प्राप्त होता है। इसके परिणामस्वरूप संवहन धाराएं चलती हैं। ये धाराएं अधिक ऊंचाई तक ताप पहुंचा देती हैं। इससे क्षोभमण्डल का विस्तार बढ़ जाता है तथा ऊंचाई अधिक हो जाती है।

JAC Class 11 Geography Important Questions Chapter 8 वायुमंडल का संघटन तथा संरचना

प्रश्न 10.
वायुमण्डल कैसे पृथ्वी से जुड़ा रहता है?
उत्तर:
वायुमण्डल का अधिकतर भाग भू-पृष्ठ से केवल 32 किलोमीटर की ऊंचाई तक सीमित है। इसे सूर्य से ऊर्जा प्राप्त होती है। यह पृथ्वी का एक अंग नहीं है। परन्तु पृथ्वी से जुड़ा हुआ है। गुरुत्वाकर्षण शक्ति द्वारा वायुमण्डल पृथ्वी से जुड़ा हुआ है।

प्रश्न 11.
वायुमण्डल के मुख्य संघटकों के नाम बताओ
उत्तर:
वायुमण्डल के प्रमुख संघटक तीन हैं:

  1. गैसें
  2. धूलकण
  3. जलवाष्प।

प्रश्न 12.
वायुमण्डल की मुख्य परतों के नाम बताओ ।
उत्तर:
वायुमण्डल की संरचना परतदार है। इसमें पांच मुख्य परतें पाई जाती हैं-

  1. क्षोभमण्डल (Troposphere)
  2. समताप मण्डल (Stratosphere)
  3. आयन मण्डल (Ionosphere)
  4. बाह्यमण्डल (Exosphere)
  5. चुम्बक मण्डल (Magnetosphere)। (मध्यमण्डल)

प्रश्न 13.
वायुमण्डल की परतों में क्षोभमण्डल को अधिक महत्त्वपूर्ण क्यों माना जाता है?
उत्तर:

  1. इस मण्डल में जीवन उपयोगी गैस ऑक्सीजन मिलती है।
  2. इस मण्डल में प्रति 165 मी० की ऊँचाई पर 1°C तापमान गिर जाता है।
  3. ऋतु, मौसम सम्बन्धी सभी घटनायें, जैसे बादल, वर्षा, भूकम्प आदि जो मानव जीवन को प्रभावित करती हैं, इसी परत में घटित होती हैं।
  4. इस मण्डल में अस्थिर वायु के कारण विक्षोभ तथा आँधी तूफान आते रहते हैं।
  5. क्षोभमण्डल के मध्य अक्षांशीय क्षेत्र में चक्रवात उत्पन्न होते हैं।

तुलनात्मक प्रश्न (Comparison Type Questions)

प्रश्न 1.
क्षोभमण्डल तथा समताप मण्डल में अन्तर स्पष्ट करो।
उत्तर:

क्षोभमण्डल (Troposphere) समताप मण्डल (Stratosphere)
(1) यह वायुमण्डल की सब से निचली परत है। (2) क्षोभमण्डल की ऊंचाई ध्रुवों पर 8 कि० मी० तथा भूमध्य रेखा पर 20 कि० मी० होती है। (1) यह धरातल से ऊपर वायुमण्डल की दूसरी परत है। (2) समताप मण्डल की ऊंचाई 16 कि० मी० से लेकर 72 कि० मी० तक होती है ।
(3) इस परत में तापमान 1°C प्रति 165 मीटर की दर से कम होता है। (3) इस परत में तापमान लगभग समान रहते हैं ।
(4) इस परत में संवाहिक धाराएं, मेघ तथा धूल कण पाए जाते हैं। (4) इस परत में संवाहिक धाराओं, मेघ तथा धूल का अभाव होता है ।
(5) इस मण्डल में ऋतु परिवर्तन सम्बन्धी घटनाएं होती रहती हैं। (5) यह मण्डल एक शान्त मण्डल है ।

निबन्धात्मक प्रश्न (Essay Type Questions)

प्रश्न 1.
विभिन्न क्षेत्रों में वायुमण्डल का महत्त्व बताओ ।
उत्तर:
वायुमण्डल निम्नलिखित क्षेत्रों में कई प्रकार से महत्त्वपूर्ण है

  1. जीवन का आधार: पृथ्वी पर मानव जीवन का आधार वायुमण्डल ही है। सौर मण्डल में केवल पृथ्वी ही एक ऐसा ग्रह है जिस पर वायुमण्डल विद्यमान है। ऑक्सीजन तथा नाइट्रोजन गैसें मानव तथा वनस्पति जीवन का आधार हैं।
  2. ऊष्मा सन्तुलन: वायुमण्डल एक ग्रीन हाऊस (Green House) की भान्ति कार्य करता है। इस प्रभाव से पृथ्वी का तापमान औसत रूप से 17°C रहता है। वायुमण्डल के बिना बहुत अधिक तापमान पर जीवन असम्भव होता
  3. हानिकारक विकिरण: ओजोन परत सूर्य से आने वाली हानिकारक पराबैंगनी किरणों को सोख कर पृथ्वी पर मानव की सुरक्षा करती है।
  4. रेडियो तरंगें: आयनमण्डल रेडियो तरंगों को पृथ्वी पर लौटा कर रेडियो प्रसारण में सहायता करता है।
  5. मौसम: वायुमण्डल की विभिन्न घटनाएं जैसे वाष्पीकरण, वर्षा, पवनें आदि मानव जीवन पर प्रभाव डालती हैं।
  6. उल्काओं से सुरक्षा: सौर मण्डल से पृथ्वी की ओर गिरने वाली उल्काएं वायुमण्डल में जल कर नष्ट हो जाती हैं।
  7. वायु: परिवहन वायुमण्डल वायुयानों की उड़ानों पर प्रभाव डालता है। जैट वायुयान समताप मण्डल में उड़ान भर सकते हैं।

JAC Class 11 Geography Important Questions Chapter 8 वायुमंडल का संघटन तथा संरचना

प्रश्न 2.
उदाहरण के साथ किसी स्थान की जलवायु तथा मौसम में अन्तर स्पष्ट करो।
उत्तर:
तापक्रम, दबाव हवाएं, नमी, मेघ और वर्षा ये मौसम के प्रधान तत्त्व (Elements of Weather) हैं। वायु मण्डल की इन दशाओं का अध्ययन ही जलवायु या मौसम है।

मौसम (Weather):
मौसम शब्द का अर्थ है ” किसी स्थान पर किसी विशेष या निश्चित समय में वायुमण्डल की दशाओं, तापक्रम, दबाव, हवाओं, नमी, मेघ और वर्षा के कुल जोड़ का अध्ययन करना” (“Weather is the condition of atmosphere at any given moment.” ) इसीलिए मौसम मानचित्रों (Weather Maps ) पर दिन व समय अवश्य लिखे जाते हैं। मौसम प्रतिदिन, प्रति सप्ताह, प्रति मास बदलता रहता है।

एक ही स्थान पर कभी मौसम गर्म (Hot), कभी उमस वाला (Sultry), कभी आर्द्र (Wet) हो सकता है। इंग्लैण्ड में दिन-प्रतिदिन के मौसम में इतनी विभिन्नता है कि कहा जाता है, “Britain has no climate, only weather.” इस प्रकार वायुमण्डल की बदलती हुई अवस्थाओं को मौसम कहा जाता है। आकाशवाणी से मौसम की स्थितियों का प्रसारण भी होता है। भारतीय मौसम विज्ञान मौसम मानचित्र प्रकाशित करता है।

जलवायु (Climate):
किसी स्थान की जलवायु उस स्थान पर एक लम्बे समय की वायुमण्डल की दशाओं के कुल जोड़ का अध्ययन होती है। यह एक लम्बे समय का औसत मौसम होती है। (“Climate is the average weather of a place over a long period.”) जलवायु तथा मौसम में भिन्नता समय पर निर्भर होती है। मौसम का सम्बन्ध थोड़े समय से है जबकि जलवायु का सम्बन्ध एक लम्बे समय से है। मिस्र में हर रोज़ एक जैसा मौसम होने के कारण जलवायु तथा मौसम में कोई अन्तर नहीं है।

इसलिए कहा जाता है, “Egypt has no weather, only climate ” इस प्रकार किसी स्थान पर कम-से-कम पिछले 35 वर्षों के मौसम की औसत दशाओं को उस स्थान की जलवायु कहते हैं। भारतीय जलवायु के अध्ययन के आधार आंकड़ों का सम्बन्ध पिछले 100 वर्षों से है। उदाहरण: देहली में किसी विशेष दिन अधिक वर्षा हो तो हम कहते हैं कि आज मौसम आर्द्र है परन्तु इसका अर्थ यह नहीं कि देहली की जलवायु आर्द्र है। देहली में ग्रीष्मकाल में अधिक वर्षा होती है तथा जलवायु मानसूनी है।

JAC Class 11 Geography Important Questions Chapter 7 भू-आकृतियाँ तथा उनका विकास

Jharkhand Board JAC Class 11 Geography Important Questions Chapter 7 भू-आकृतियाँ तथा उनका विकास Important Questions and Answers.

JAC Board Class 11 Geography Important Questions Chapter 7 भू-आकृतियाँ तथा उनका विकास

बहु-विकल्पी प्रश्न (Multiple Choice Questions)

प्रश्न – दिए गए प्रश्नों के चार वैकल्पिक उत्तरों में से सही उत्तर चुनकर लिखो-
1. संसार के किस प्रदेश में पाताल तोड़ कुएं सबसे अधिक पाए जाते हैं?
(A) गुजरात ( भारत )
(B) क्वींसलैंड (आस्ट्रेलिया)
(C) यूक्रेन (रूस)
(D) ह्वांग – हो मैदान (चीन)।
उत्तर:
(B) क्वीसलैंड (आस्ट्रेलिया)।

2. निम्नलिखित में से कौन-सा कारक भौम जल स्तर को प्रभावित नहीं करता?
(A) वाष्पीकरण
(B) वर्षा
(C) चट्टानों की कठोरता
(D) चट्टानों की पारगम्यता।
उत्तर:
(D) चट्टानों की पारगम्यता।

3. कार्स्ट प्रदेशों में भूमिगत जल का कार्य अधिकतर किस क्रिया द्वारा होता है?
(A) अपरदन
(B) अपवाहन
(C) अपघर्षण
(D) घुलन।
उत्तर:
(D) घुलन।

JAC Class 11 Geography Important Questions Chapter 7 भू-आकृतियाँ तथा उनका विकास

4. ओल्ड फेथफुल गाईज़र किस देश में स्थित है?
(A) संयुक्त राज्य अमेरिका
(B) आइसलैंड
(C) न्यूज़ीलैंड
(D) ऑस्ट्रेलिया।
उत्तर:
(A) संयुक्त राज्य अमेरिका।

5. विलयन रंध्र किस प्रकार के धरातल पर बनते हैं ?
(A) कार्स्ट प्रदेश में
(B) नदी घाटी में
(C) महासागरों में
उत्तर:
(A) कार्स्ट प्रदेश में।

6. ‘ग्रेट आर्टेजियन बेसिन’ स्थित है-
(A) संयुक्त राज्य अमेरिका के येलोस्टोम पार्क में
(B) न्यूज़ीलैंड के उत्तरी द्वीप में
(C) ऑस्ट्रेलिया महाद्वीप के मध्य पूर्व भाग में
(D) आइसलैंड में।
उत्तर:
ऑस्ट्रेलिया महाद्वीप के मध्य पूर्व भाग में।

7. ओल्ड फेथफुल गीज़र पाया जाता है-
(A) संयुक्त राज्य अमेरिका के येलोस्टोन पार्क में
(B) न्यूज़ीलैंड के उत्तरी द्वीप में
(C) ऑस्ट्रेलिया महाद्वीप के मध्य पूर्व भाग में
(D) आइसलैंड में।
उत्तर:
(C) ऑस्ट्रेलिया महाद्वीप के मध्य पूर्व भाग में।

8. हिमानी प्रदेशों में शृंग की रचना का मुख्य कारण क्या है?
(A) उत्थान क्रिया
(B) चारों ओर हिमसागर का बनना
(C) अपरदन
(D) अपक्षरण।
उत्तर:
(C) अपरदन।

9. गुम्बद आकार भाग से बहने वाली नदियों द्वारा किस प्रवाह की रचना होती है?
(A) युग्म आकृतिक प्रवाह
(B) समान्तर प्रवाह
(C) जलायित प्रवाह
(D) आरीय प्रवाह
उत्तर:
(D) आरीय प्रवाह।

10. नदी के मुहाने पर बनने वाली संकीर्ण तथा गहरी घाटी को क्या कहते हैं?
(A) डेल्टा
(B) ज्वार नदमुख
(C) मुहाना
(D) घाटी।
उत्तर:
(B) ज्वार नदमुख।

JAC Class 11 Geography Important Questions Chapter 7 भू-आकृतियाँ तथा उनका विकास

11. गुफ़ाओं की रचना किस क्रिया द्वारा होती है?
(A) ऑक्सीकरण
(B) कार्बोनेटीकरण
(C) जल योजना
(D) घोलीकरण।
उत्तर:
(B) कार्बोनेटीकरण।

12. नदी के किस भाग में जल प्रपात की रचना होती है?
(A) ऊपरी भाग
(B) मध्य भाग
(C) निचला भाग
(D) मुहाना।
उत्तर:
(A) ऊपरी भाग।

13. स्थैतिक साधनों द्वारा चट्टानों का विघटन तथा अपघटन
(A) अपक्षय
(B) अपरदन
(C) निक्षेपण
(D) परिवहन।
उत्तर:
(A) अपक्षय।

14. कौन-सी अपवाह प्रणाली में महाखड्ड का निर्माण होता है?
(A) द्रुमाकृतिक
(B) पूर्ववर्ती
(C) अरीय
(D) जालीनुमा।
उत्तर:
(C) पूर्ववर्ती।

15. नदी के मध्य मार्ग में इसकी कौन-सी मुख्य विशेषता होती है?
(A) घाटी को गहरा करना
(B) डेल्टा का निर्माण
(C) घाटी को चौड़ा करना
(D) वितरकाओं का निर्माण।
उत्तर:
(C) घाटी को चौड़ा करना

अति लघु उत्तरीय प्रश्न (Very Short Answer Type Questions)

प्रश्न 1.
मुख्य भू-आकृतिक कारक कौन-से हैं?
उत्तर:

  1. नदी
  2. हिमनदी
  3. वायु
  4. सागरीय तरंगें।

प्रश्न 2.
अपक्षय के दो मुख्य प्रकार बताओ।
उत्तर:
भौतिक तथा रासायनिक।

प्रश्न 3.
रासायनिक अपक्षय की मुख्य क्रियाएं बताओ
उत्तर:

  1. ऑक्सीकरण
  2. जल योजन
  3. विलयन
  4. कार्बोनेटीकरण।

प्रश्न 4.
अपरदन चक्र में मुख्य अवस्थाएं कौन-सी हैं?
उत्तर:
युवा, प्रौढ़, वृद्धावस्था।

प्रश्न 5.
भू-आकृतिक कारकों के कार्य के प्रकार बताओ।
उत्तर:

  1. अपरदन
  2. परिवहन
  3. निक्षेपण।

JAC Class 11 Geography Important Questions Chapter 7 भू-आकृतियाँ तथा उनका विकास

प्रश्न 6.
मुख अपवाह ढांचों के नाम लिखो।
उत्तर:

  1. आरीय
  2. द्रुमाकृतिक
  3. समान्तर
  4. जलायित।

प्रश्न 7.
नदी की युवा अवस्था में बनने वाले भू-आकार बताओ।
उत्तर:
गार्ज, केनियन, झरने।

प्रश्न 8.
नदी निक्षेप से बनने वाले भू-आकार बताओ।
उत्तर:
विसर्प, तटबन्ध, बाढ़ का मैदान।

प्रश्न 9.
वायु अपरदन से बनने वाले चार भू-आकार बताओ।
उत्तर:

  1. छत्रक,
  2. यारडांग
  3. ज्यूज़न,
  4. इन्सेलबर्ग।

प्रश्न 10.
हिम नदी अपरदन से बनने वाले भू-आकार बताओ।
उत्तर:
सर्क, श्रृंग, लटकती घाटी, ‘U’ आकार घाटी।

प्रश्न 11.
निम्नीकरण (Degradation) किसे कहते हैं?
उत्तर:
बाह्य कारकों द्वारा धरातल के अपरदन को निम्नीकरण कहते हैं।

प्रश्न 12.
निम्नीकरण में कौन-सी क्रियाएं शामिल हैं?
उत्तर:

  1. अपक्षय
  2. अपरदन
  3. परिवहन।

प्रश्न 13.
अपरदन चक्र की धारणा का सबसे पहले प्रतिपादन किसने किया था?
उत्तर:
विलियम मारिस डेविस ने।

प्रश्न 14.
यदि नदी का वेग दुगुना हो जाए तो उसकी भार वाहन शक्ति कितने गुना बढ़ जाएगी?
उत्तर:
64 गुना।

JAC Class 11 Geography Important Questions Chapter 7 भू-आकृतियाँ तथा उनका विकास

प्रश्न 15.
ग्रैंड केनियन कहाँ स्थित हैं?
उत्तर:
संयुक्त राज्य में, कोलोरेडो नदी पर।

प्रश्न 16.
जल प्रपात किसे कहते हैं?
उत्तर:
जब नदी चट्टानी कगार पर ऊपर से सीधे नीचे गिरती है तो उसे जल प्रपात कहते हैं।

प्रश्न 17.
उत्तरी अमेरिका के एक प्रसिद्ध जल प्रपात का नाम लिखो।
उत्तर:
नियाग्रा जल प्रपात 120 मीटर ऊंचा।

प्रश्न 18.
भारत में दो प्रसिद्ध जल प्रपातों के उदाहरण दो।
उत्तर:

  1. जोग जल प्रपात 260 मीटर ऊंचा।
  2. हुण्ड्र जल प्रपात 97 मीटर ऊँचा।

प्रश्न 19.
चीन का शोक किस नदी को कहते हैं?
उत्तर:
ह्वांग-हो नदी।

प्रश्न 20.
कोई तीन प्रसिद्ध डैल्टाओं के नाम लिखो।
उत्तर:

  1. नील डैल्टा
  2. गंगा डैल्टा
  3. मिसीसिपी डेल्टा।

प्रश्न 21.
लोएज प्रदेश कहां पाये जाते हैं?
उत्तर:
पश्चिमी चीन में ।

प्रश्न 22.
भारतीय उपमहाद्वीप के सबसे बड़े डेल्टा का नाम बताएं
उत्तर:
सुन्दरवन डेल्टा।

JAC Class 11 Geography Important Questions Chapter 7 भू-आकृतियाँ तथा उनका विकास

प्रश्न 23.
स्टैलेक्टाइट का निर्माण किस भू-प्राकृतिक कारक द्वारा होता है?
उत्तर:
चूना युक्त जल की गुफा की छत पर लटकी बूंदों के वाष्पीकृत से स्टैलेक्टाइट का निर्माण होता है।

लघु उत्तरीय प्रश्न (Short Answer Type Questions)

प्रश्न 1.
भारत के पश्चिमी तट पर नर्मदा व ताप्ती नदियां डैल्टे क्यों नहीं बनातीं?
उत्तर:
पश्चिमी तट पर नदियों के निचले भाग में तीव्र ढाल है। नदियां तेज़ गति से समुद्र में गिरती हैं तथा मिट्टी का निक्षेप नहीं होता। इसलिए डैल्टा का निर्माण नहीं होता। पश्चिमी तटीय मैदान की चौड़ाई भी कम है। इसलिए डैल्टा निर्माण के लिए समतल भूमि की कमी है।

प्रश्न 2.
गार्ज तथा प्रपात खड्ड में क्या अन्तर होता है?
उत्तर:
पर्वतीय भागों में गहरे और तंग नदी मार्ग को गार्ज कहते हैं। नदी कठोर चट्टानों के बने अपने किनारों को तोड़-फोड़ नहीं सकती। इसलिए जब पर्वत ऊंचे उठते रहते हैं तो नदी घाटी लगातार गहरी होती रहती है। प्रपाती खड्ड (कैनियन) प्रायः शुष्क प्रदेशों में बनती हैं। इस घाटी में ऋतु प्रहार (weathering) के कारण घाटी का ऊपरी भाग कुछ अधिक चौड़ा हो जाता है। गार्ज में नदी मार्ग के किनारे दीवार की भांति खड़े (Vertical) होते हैं, परन्तु कैनियन की दीवारें पूर्ण रूप से लम्बवत् नहीं होतीं।

प्रश्न 3.
किसी जल प्रपात की रचना कैसे होती है?
उत्तर:
जब अधिक ऊंचाई से जल अधिक वेग से खड़े ढाल पर बहता है तो उसे जल-प्रपात कहते हैं। जब नदी के मार्ग में ऊपरी पर्त कठोर चट्टानों की हो और नीचे की पर्त नर्म चट्टानों की हो, तो नीचे की नर्म चट्टान जल्दी कट जाती है। दोनों परतों में एक अन्तर आ जाता है। कठोर चट्टान आगे बढ़ी रहती है। इस प्रकार इस ढाल पर पानी झरने के रूप में बहता है। भारत में जोग जल प्रपात 260 मीटर ऊंचा है।

प्रश्न 4.
छत्रक शैल किसे कहते हैं? इसके विभिन्न कारक कौन-से हैं?
उत्तर:
मरुस्थलीय भागों में छतरी के आकार की कठोर चट्टानों को छत्रक कहते हैं। कठोर चट्टानों के निचले भागों में चारों तरफ से नीचे का कटाव (under cutting) होता है। एक पतले आधार के ऊपर छतरी के आकार की चट्टानें खड़ी रहती नीचे की चट्टान एक गुफा के समान कट जाती है। जोधपुर के निकट ग्रेनाइट चट्टानें नीचे से कट कर छत्रक बन गई हैं।

प्रश्न 5.
नदी की शक्ति किन कारकों पर निर्भर करती है? उसकी भार वहन की क्षमता को प्रभावित करने वाले मुख्य कारक कौन-से हैं?
उत्तर:
नदी की अपरदन तथा परिवहन शक्ति दो कारकों पर निर्भर करती है

  1. नदी का वेग
  2. जल की मात्रा।

नदी के वेग तथा अपरदन शक्ति में वर्ग का अनुपात होता है, यदि वेग दुगुना हो जाए तो अपरदन शक्ति चौगुनी होती है तथा परिवहन शक्ति 64 गुना बढ़ जाती है। जल की मात्रा बढ़ जाने से नदी अधिक भार का परिवहन कर सकती है। नदी की परिवहन शक्ति मुख्यतः नदी के ढाल तथा नदी के भार पर निर्भर करती है।

JAC Class 11 Geography Important Questions Chapter 7 भू-आकृतियाँ तथा उनका विकास

प्रश्न 6.
समुद्र द्वारा किए जाने वाले विभेदी अपरदन से आप क्या समझते हैं? इस क्रिया से किन स्थलाकृतियों का निर्माण होता है?
उत्तर:
समुद्र तट पर स्थित नर्म तथा कठोर चट्टानों की क्षैतिज अथवा लम्ब स्थिति के कारण विभेदी अपरदन होता है। नर्म चट्टानें शीघ्र टूट जाती हैं परन्तु कठोर चट्टानें खड़ी रहती हैं। इस प्रकार नर्म चट्टानों के अपरदन से खाड़ियां बनती हैं। कठोर चट्टानें भृगु के रूप में खड़ी रहती हैं। घुलनशील चट्टानों के घुल जाने से गुफाओं का निर्माण होता है। उन क्षेत्रों में मेहराब तथा प्राकृतिक पुल बनते हैं। कठोर चट्टानें स्टैक के रूप में बची-खुची रहती हैं।

प्रश्न 7.
पुनर्योवन के प्रभावों का वर्णन करें।
उत्तर:
भू-संचरण या जलवायु परिवर्तन के कारण पुरानी नदियां यदि युवा अवस्था में आ जाती हैं तो इसे पुनर्योवन कहते हैं। इस क्रिया के निम्नलिखित प्रभाव होते हैं:

  1. नदी की अपरदन क्षमता बढ़ जाती है।
  2. पुरानी घाटी में एक नई घाटी का निर्माण शुरू हो जाता है।
  3. नदी वेदिकाएं बनती हैं।
  4. गहरे विसर्प बनते हैं ।

प्रश्न 8.
महाद्वीपीय हिमनदी तथा घाटी हिमनदी में अन्तर बताओ।
उत्तर:
महाद्वीपीय हिमनदी या हिम चादरों का विस्तार ध्रुवीय प्रदेशों में अंटार्कटिका तथा ग्रीनलैण्ड में है। ये विशाल क्षेत्रों में फैले हुए हैं। घाटी हिमनदियां हिमालय, आल्पस आदि ऊंचे पर्वतों के हिम क्षेत्रों से जन्म लेती हैं। यह पर्वतीय क्षेत्रों में तथा ढलानों पर अपरदन का कार्य करती हैं। हिम चादरें किसी क्षेत्र को समतल करने का कार्य करती है। परन्तु घाटी हिमनदी कई भू-आकार जैसे यू-घाटी, सर्प आदि की रचना करती है।

प्रश्न 9.
‘V’ आकार घाटी तथा ‘U’ आकार घाटी में अन्तर बताओ।
उत्तर:
नदी के अपरदन से ‘V’ आकार घाटी बनती है, परन्तु हिमनदी के अपरदन से ‘U’ आकार घाटी बनती है। ‘V’ आकार घाटी अधिक गहरी होती है। ‘U’ आकार घाटी की दीवारें लगभग लम्बवत् होती हैं परन्तु ‘V’ आकार घाटी की दीवारें पूर्ण रूप से लम्ब नहीं होतीं । हिमनदी अपनी कोई घाटी नहीं बनाती, अपितु वह नदी घाटी का रूप बदल कर उसे ‘U’ आकार बना देती है।

प्रश्न 10.
हिम तोंदों तथा हिमनदियों में क्या अन्तर है?
उत्तर:
हिम तोंदे हिम चादरों से जन्म लेते हैं। जैसे ग्रीनलैण्ड तथा अण्टार्कटिका में परन्तु हिमनदियां ऊंचे पर्वतों के हिम क्षेत्रों से निकलती हैं जैसे हिमालय तथा आल्पस पर्वत हिम तोंदे समुद्र में तैरते हुए दिखाई देते हैं तथा गर्म प्रदेशों में आकर पिघल जाते हैं। हिमनदियां पर्वतीय ढलानों पर खिसकती हैं तथा मैदानों में आकर पिघलती हैं। हिम तोंदों का 1/10 भाग जल से बाहर दिखाई देता है, परन्तु हिमनदी का सारा भाग धरातल पर दिखाई देता है।

प्रश्न 11.
लटकती घाटी (Hanging Valley) की रचना कैसे होती है?
उत्तर:
हिमनदी की मुख्य घाटी अधिक हिम के कारण अधिक गहरी होती है। पर्वतों की ढलानों से उतरने वाली सहायक हिमनदी की घाटी कम गहरी होती है। जब हिम पिघलती है तो सहायक हिम नदी की घाटी मुख्य घाटी से ऊंची रह जाती है तथा लटकती हुई प्रतीत होती है। इसे लटकती हुई घाटी कहते हैं।

प्रश्न 12.
वायु तथा नदी के कार्य की तुलना करो।
उत्तर:

वायु के कार्य नदी का कार्य
(1) वायु का कार्य मरूस्थल प्रदेशों में महत्त्वपूर्ण होता है। (1) नदी का कार्य आर्द्र प्रदेशों में महत्त्वपूर्ण होता है।
(2) नदी अपने मलबे को दूर-दूर प्रदेशों तक परिवहन करती है। (2) नदी का कार्य अपनी घाटी तक ही सीमित होता है।
(3) वायु का कार्य वायु की गति पर निर्भर करता है। (3) नदी का कार्य क्षेत्र की ढलान पर निर्भर करता है।
(4) वायु का कार्य धीमी गति से होता है तथा वायु भार कम होता है। (4) नदी का कार्य तीव्र गति से होता है तथा नदी भार बहुत अधिक होता है।

प्रश्न 13.
“वायु द्वारा अपरदन की तुलना रेगमार से की जाती है।” व्याख्या कीजिए।
उत्तर:
वायु रेत के बारीक कणों को उड़ा कर दूर-दूर तक ले जाती है। वायु के धूलि कण इस अपरदन के यन्त्र (Tools) हैं। तीव्र गति से वायु रेत के कणों को चट्टानों से टकराती है। ये कण एक रेगमार (Sand paper) की तरह कटाव करते हैं। ये कण चट्टानों को रगड़ कर इस तरह मुलायम कर देते हैं जैसे पालिश किया हो या रेगमार से घिसा हो।

JAC Class 11 Geography Important Questions Chapter 7 भू-आकृतियाँ तथा उनका विकास

प्रश्न 14.
ज्यूजन तथा यारडांग में क्या अन्तर है?
उत्तर:

यारडांग ज्यूजन
(1) इसमें कठोर तथा नर्म चट्टानें लम्ब रूप में मिलती हैं। (1) इसमें कठोर तथा नर्म चट्टाने एक-दूसरे के समानान्तर होती हैं।
(2) इसका आकार पसलियों (Ribs) जैसा होता है। (2) इसका आकार एक ढक्कन दवात जैसा होता है।
(3) इसमें लम्बे कटाव मिलते हैं। (3) इसमें गहरे कटाव मिलते हैं।
(4) इसकी ऊंचाई 15 मीटर होती है। (4) इनकी ऊंचाई 10 मीटर होती है।

JAC Class 11 Geography Important Questions Chapter 7 भू-आकृतियाँ तथा उनका विकास 1

प्रश्न 15.
लम्बे बालू स्तूप तथा आड़े बालू स्तूप में क्या अन्तर है?
उत्तर:

लम्बे बालू स्तूप आड़े बालू स्तूप
(1) ये रेत की लम्बी दीवारें होती हैं। (1) ये अर्द्ध-चन्द्राकार टीले होते हैं।
(2) ये प्रचलित पवनों की दिशा के समानान्तर बनते हैं। (2) ये प्रचलित पवनों की दिशा के समकोण पर बनते हैं।
(3) ये 100 मीटर तक ऊँचे होते हैं। (3) ये 50 मीटर तक ऊंचे होते हैं।
(4) ये तलवार के आकार जैसे होते हैं तथा इन्हें सीफ़ भी कहा जाता है। (4) ये लहरों के आकार जैसे होते हैं।

प्रश्न 16.
सागरीय लहरों द्वारा अपरदन किन तत्त्वों पर निर्भर करता है?
उत्तर:
सागरीय लहरों द्वारा अपरदन प्रायः तूफ़ानी लहरों द्वारा 200 मीटर की गहराई तक होता है। यह अपरदन कई तत्त्वों पर निर्भर करता है-

  1. लहरों का अधिक वेग तथा ऊँचाई
  2. चट्टानों की रचना
  3. तट का ढाल
  4. जल की गहराई
  5. छिद्रों तथा दरारों का होना
  6. वायु की गति।

प्रश्न 17.
भृगु (Cliff) की रचना कैसे होती है
उत्तर:
समुद्र तट पर खड़ी ढाल वाली चट्टान को भृगु कहते हैं। लहरों के सीधे अपरदन से चट्टान आधार पर कट जाती है तथा दांत (Notch ) की रचना होती है। यह खोखला स्थान धीरे-धीरे बड़ा हो जाता है। ऊपरी भाग धीरे- धीरे आगे बढ़कर लटकने लगता है। जब ऊपर का भाग टूट कर गिर जाता है, तो नीचे का कठोर भाग एक भृगु का रूप धारण कर लेता है।

प्रश्न 18.
गुम्फित नदी से क्या अभिप्राय है?
उत्तर:
गुम्फित नदी ( Braided Channels):
नदी द्वारा प्रवाहित नद्य भार का निक्षेपण अगर नदी के मध्य में लम्बी रोधिका या रोध के रूप में हो जाए तो नदी धारा दो शाखाओं में विभाजित हो जाती हैं और यह प्रवाह किनारों पर क्षैतिज अपरदन करता है। नदी घाटी की चौड़ाई बढ़ने पर और जल आयतन कम होने पर, प्रजाहित जलोढ़ अधिक मात्रा में एक क्षैतिज अवरोध के रूप में द्वीप की भान्ति निक्षेपित हो जाते हैं तथा मुख्य जल धारा कई जल धाराओं में बंट जाती है।

गुम्फित नदी प्रारूप के लिए तटों पर अपरदन व निक्षेप आवश्यक है। या जब नदी में जल की मात्रा कम तथा जलोढ़ अधिक हो जाए, तब नदी प्रवाह में ही रेत, मिट्टी, बजरी आदि की लम्बी अवरोधिकाओं का जमाव हो जाता है और नदी प्रवाह कई जल वितरिकाओं में बंट जाता है। जल प्रवाह की ये विरिक्ताएं मिलती हैं और फिर उपधाराओं में बंट जाती हैं। इस प्रकार एक गुम्फित नदी प्रारूप का विकास होता है।

JAC Class 11 Geography Important Questions Chapter 7 भू-आकृतियाँ तथा उनका विकास

प्रश्न 19.
गम्भीरभूत विसर्प तथा नदी वेदिकाएं किस प्रकार बनती हैं?
उत्तर:
अधः कर्तित विसर्प या गम्भीरभूत विसर्प (Incised or Entrenched Meanders ):
तीव्र ढालों में तीव्रता से बहती हुई नदियां सामान्यतः नदी तल पर अपरदन करती हैं। तीव्र नदी ढालों में भी पार्श्व/ क्षैतिज अपरदन अधिक नहीं होता लेकिन मंद ढालों पर बहती हुई नदियां अधिक पार्श्व अपरदन करती हैं। क्षैतिज अपरदन अधिक होने के कारण, मंद ढालों पर बहती हुई नदियां वक्री होकर बहती हैं या नदी विसर्प बनाती हैं।
JAC Class 11 Geography Important Questions Chapter 7 भू-आकृतियाँ तथा उनका विकास 2
नदी वेदिकाएं (River Terraces):
नदी वेदिकाएं प्रारम्भिक बाढ़ मैदान या पुरानी नदी घाटियों के चिह्न हैं। ये चट्टानी धरातल या नदियों के तल हैं जो जलोढ़ रहित वा निक्षेपित जलोढ़ वेदिकाओं के रूप में पाए जाते हैं। नदी वेदिकाएं मुख्यतः अपरदित स्थलरूप हैं क्योंकि ये नदी निक्षेपित बाढ़ मैदानों के लम्बवत् अपरदन से निर्मित होते हैं। ये वेदिकाएं संख्या में कई तथा भिन्न ऊंचाई की भी हो सकती हैं जो आरम्भिक नदी जल स्तर को दर्शाते हैं। नदी वेदिकाएं नदी के दोनों तरफ समान ऊंचाई वाली हो सकती हैं और इनके इस स्वरूप को युग्म (Paired ) वेदिकाएं कहते हैं।

प्रश्न 20.
नदी के मार्ग को विभिन्न अवस्थाओं में बांट कर प्रत्येक की विशेषताएं बताओ।
उत्तर:
नदी मार्ग को उद्गम से लेकर सागर में गिरने तक तीन प्रमुख भागों में बांटा जाता है:
1. तरुणावस्था (Youth ):
इस अवस्था में नदियों की संख्या बहुत कम होती है तथा इनका मूल ढाल पर दुर्बल एकीकरण होता है। ये नदियां उथली V-आकार घाटी बनाती हैं और जिनमें बाढ़ के मैदान लगभग अनुपस्थित या संकरे बाढ़ मैदान मुख्य नदी के साथ-साथ पाए जाते हैं। जल विभाजक अत्यधिक विस्तृत (चौड़े) व समतल होते हैं, जिनमें दलदल व झीलें होती हैं। इन ऊंचे समतल धरातल पर नदी विसर्प विकसित हो जाते हैं। ये विसर्प अंततः ऊंचे धरातलों में गम्भीरभूत हो जाते हैं (अर्थात् विसर्प की तली में निम्न कटाव होता है और ये गहराई में बढ़ते हैं ।) जहां कठोर चट्टानों का अनावरण होता है वहां जलप्रपात व क्षिप्रितकाएं पाई जाती हैं।
JAC Class 11 Geography Important Questions Chapter 7 भू-आकृतियाँ तथा उनका विकास 3

2. प्रौढ़ावस्था (Mature ):
इस अवस्था में नदियों में जल की मात्रा अधिक होती है और सहायक नदियां भी इसमें आकर मिलती हैं। नदी घाटियां V-आकार की होती हैं लेकिन गहरी होती हैं। मुख्य नदी व्यापक विस्तृत होने से विस्तृत बाढ़ के मैदान पाए जाते हैं जिसमें घाटी के भीतर ही नदी विसर्प बनाती हुई प्रवाहित होती है। युवावस्था में निर्मित समतल, विस्तृत व दलदली अन्तर नदी क्षेत्र विलीन हो जाते हैं और नदी विभाजक स्पष्ट होते हैं। जलप्रपात व क्षिपिकाएं लुप्त हो जाती हैं।

3. जीर्णावस्था (Old ):
जीर्णावस्था में छोटी सहायक नदियां कम होती हैं और ढाल मंद होता है। नदियां स्वतन्त्र रूप से विस्तृत बाढ़ के मैदानों में बहती हुई नदी विसर्प, प्राकृतिक तटबन्ध, गोखुर झील आदि बनाती है विभाजक विस्तृत तथा समतल होते हैं जिनमें दलदल, कच्छ व अनूप उपस्थित होते हैं। अधिकतर भूदृश्य समुद्रतल के बराबर या थोड़ा ऊंचा होता है।

प्रश्न 21.
पेडीमेंट तथा पदस्थली की रचना बताओ।
उत्तर:
अपरदनात्मक स्थलरूप पेडीमेंट (Pediment ) और पदस्थली (Pediplain): मरुस्थलों में भूदृश्य का विकास मुख्यतः पेडीमेंट का निर्माण व उसका ही विकसित रूप है। पर्वतों के पाद पर मलबे रहित अथवा मलबे सहित मंद ढाल वाले चट्टानी तल पेडीमेंट कहलाते हैं। पेडीमेंट का निर्माण पर्वतीय अग्रभाग के अपरदन मुख्यतः सरिता के क्षैतिज अपरदन व परत बाढ़- दोनों के संयुक्त अपरदन से होता है। पर्वतों के तीव्र अग्रभाग या किनारों का अपरदन होता है और ये अपरदित भाग धरातल पर गिरते हैं। बाढ़ के कारण व तत्पश्चात् क्लिफ निर्माण से जो कटाव होते हैं उससे पेडीमेंट निर्मित होते हैं।

ये बाढ़ कटाव व क्लिफ निवर्तन करते हैं। यह अपरदन क्रिया पृष्ठक्षरण (backwasting) द्वारा की गई समानान्तर ढाल निवर्तन प्रक्रिया (paralled retreat of slope) कहलाती है। अत: समानान्तर ढाल विवर्तन द्वारा पेडीमेंट पर्वतों के अग्रभाग को अपरदित करते हुए पीछे हटते हैं और धीरे-धीरे पर्वतों का अपरदन होता है तथा इन्सेलबर्ग (Inselberg ) निर्मित होते हैं जो कि पर्वतों के अवशिष्ट रूप,हैं। इस प्रकार मरुस्थलीय प्रदेशों में एक उच्च धरातल आकृति विहीन मैदान में परिवर्तित हो जाता जिसे पेड़ीप्लेन या पदस्थली कहते हैं।

JAC Class 11 Geography Important Questions Chapter 7 भू-आकृतियाँ तथा उनका विकास

प्रश्न 22.
प्लाया तथा ब्नजादा की रचना बताओ।
उत्तर:
प्लाया और बजादा (Playa and Bajada):
मरुभूमियों में मैदान (Plains) प्रमुख स्थलरूप हैं। पर्वत व पहाड़ियों से घिरे बेसिनों में अपवाह मुख्यतः बेसिन के मध्य में होता है, तथा बेसिन के किनारों से लगातार तलछट जमाव के कारण बेसिन के मध्य में लगभग समतल मैदान की रचना हो जाती है। पर्याप्त जल उपलब्ध होने पर यह मैदान उथले जल क्षेत्र में परिवर्तित हो जाता है। इस प्रकार की उथली जल झीलें ही प्लाया ( Playa) कहलाती हैं। प्लाया में वाष्पीकरण के कारण जल अल्प समय के लिए ही रहता है और अक्सर प्लाया में लवणों के घने निक्षेप पाए जाते हैं।

ऐसे प्लाया मैदान जो लवणों से भरे हों, कल्लर भूमि या क्षारीय क्षेत्र (Alkali flats) कहलाती हैं। अगर प्लाया मरुस्थल बेसिन के समतल मध्य मैदान हैं तो इनके ऊपर के ढाल वाले मैदान (जिनका ढाल 1° से 5° के मध्य है ) और जो गिरिपद पर जमाव से बने हैं, उन्हें बजादा (Bajada) कहते हैं। प्लाया व बजादा मुख्यतः पर्वतों के निवर्तन से बने पेडिमेंट के ही हिस्से हैं, जो बाढ़ अपरदित मलबे से ढके हैं। चूंकि प्लाया व बजादा पर तलछटों के जमाव की मोटी परत नहीं होती और इनके तल अपरदन के फलस्वरूप बनते हैं; इसलिए इन्हें कई बार अपरदनात्मक स्थलरूपों में वर्णित किया जाता है।

प्रश्न 23.
जल चक्र से क्या अभिप्राय है?
उत्तर:
प्रकृति का जल अपने कई रूपों में बदलते हुए पुन: उसी रूप में लौट आता है। इसे ही जल चक्र की संज्ञा दी जाती है। जैसे पृथ्वी के जलाशयों का जल सूर्य के ताप के कारण जलवाष्प में बदलकर वायुमण्डल में चला जाता है। वायुमण्डल में यह संघनित होकर बर्फ या जल के रूप में पुनः पृथ्वी पर आ जाता है। इस प्रकार जल वायुमण्डल, स्थलमण्डल तथा जलमण्डल पर स्थानान्तरित होता है।

तुलनात्मक प्रश्न (Comparison Type Questions)

प्रश्न 1.
अपरदन तथा अपक्षय में अन्तर स्पष्ट करो
उत्तर:

अपरदन (Erosion) अपक्षय (Weathering)
(1) भू-तल पर खुरचने, काँट-छाँट तथा मलवे को परिवहन करने के कार्य को अपरदन कहते हैं। (1) चट्टानों को अपघटन तथा विघटन के द्वारा अपने मूल स्थान पर तोड़-फोड़ करने की क्रिया को अपक्षय कहते हैं।
(2) अपरदन एक बड़े क्षेत्र में होता है। (2) अपक्षय छोटे क्षेत्रों की क्रिया है।
(3) अपरदन गतिशील कारकों द्वारा जैसे जल, हिमनदी, वायु आदि से होता है। (3) अपक्षय सूर्यतप, पाला तथा रासायनिक क्रियाओं द्वारा होता है।
(4) अपक्षय अपरदन में सहायक होता है। (4) अपक्षय चट्टानों को कमज़ोर करके अपरदन में सहायता करता है ।

JAC Class 11 Geography Important Questions Chapter 7 भू-आकृतियाँ तथा उनका विकास

प्रश्न 2.
‘V’ आकार घाटी तथा ‘U’ आकार घाटी में अन्तर स्पष्ट करो।
उत्तर:

V आकार घाटी ‘U’ आकार घाटी
(1) नदी अपरदन से ‘V’ आकार घाटी बनती है। (1) हिमनदी के अपरदन से ‘U’ आकार घाटी बनती है।
(2) यह घाटी अधिक गहरी होती है। जब यह ‘T’ आकार में बन जाती है तो इसे कैनियन कहते हैं। (2) यह घाटी अधिक गहरी नहीं होती है। इससे ऊंचाई पर बनी घाटी को लटकती घाटी कहते हैं।
(3) इसके किनारे पूर्ण रूप से लम्ब नहीं होते। (3) इसकी दीवारें पूर्ण रूप से लम्ब होती हैं।
(4) नदी अपने गहरे कटाव से इस घाटी की रचना करती है। (4) हिमनदी की अपनी कोई घाटी नहीं होरी। यह नदी घाटी का रूप बदल कर उसे ‘U’ आकार की घाटी बना देती है।

प्रश्न 3.
निम्नीकरण तथा अधिवृद्धि में अन्तर स्पष्ट करो।
उत्तर:

निम्नीकरण अधिवृद्धि
(1) बाह्य कारकों द्वारा स्थानीय धरातल के अपरदन को निम्नीकरण कहते हैं। (1) धरातल के निचले स्थानों को मलवे के निक्षेप से भरने की क्रिया को अधिवृद्धि कहते हैं।
(2) यह अपरदन का ही दूसरा रूप है। (2) यह निक्षेप का ही दूसरा रूप है।
(3) इससे ऊंचे प्रदेश निचले प्रदेश हो जाते हैं। (3) इससे निचले प्रदेश का तल ऊँचा हो जाता है।

निबन्धात्मक प्रश्न (Essay Type Questions)

प्रश्न 1.
तरंगों (लहरों) द्वारा अपरदन, परिवहन तथा निक्षेप के कार्य का वर्णन करो।
उत्तर:
समुद्री लहरों का कार्य (Work of Sea Waves) दूसरे कार्यकर्त्ताओं के समान लहरें भी अपरदन, परिवहन तथा निक्षेप का कार्य करती हैं।
(क) अपरदन (Erosion ): तट पर तोड़-फोड़ का कार्य प्राय: सर्फ ( Surf) लहरें तथा तूफानी लहरों द्वारा ही होता है। समुद्री लहरों द्वारा अपरदन अधिक-से-अधिक 600 फुट की गहराई तक होता है। यह अपरदन चार प्रकार से होता है:

  1. जल दबाव क्रिया (Water Pressure ): छिद्रों में जल के दबाव से चट्टानें टूटने-बिखरने लगती हैं।
  2. अपघर्षण (Abrasion): बड़े – बड़े पत्थर चट्टानों से टकरा कर उन्हें तोड़ते रहते हैं।
  3. संनिघर्षण (Attrition ): पत्थरों के टुकड़े आपस में टकरा कर टूटते रहते हैं।
  4. घुलन क्रिया (Solution ): समुद्री जल चूना युक्त चट्टानों को घुला कर अपरदन कर देता है।

अपरदन द्वारा बने स्थलाकार:
1. खाड़ियां (Bays ): जब किसी तट पर कोमल तथा कठोर चट्टानें लम्ब रूप में स्थित हों तो कोमल चट्टानें (Soft Rocks) भीतर की ओर अधिक कट जाती हैं। इस प्रकार कोमल चट्टानों में खाड़ियां (Bays) बन जाती हैं तथा कठोर चट्टानें आगे निकल जाती हैं तथा अन्तरीप (Cape ) की रचना करती हैं।
JAC Class 11 Geography Important Questions Chapter 7 भू-आकृतियाँ तथा उनका विकास 4

2. भृगु (Cliff): समुद्र तट पर खड्ढे ढाल वाले पर्वत को भृगु या क्लिफ (Cliff) कहते हैं। लहरों के लगातार सीधे आक्रमण के कारण चट्टानें अपने आधार पर कट जाती हैं तथा दांत (Notch ) की रचना होती है। ऊपर का भाग धीरे-धीरे आगे बढ़कर लटकने लगता है। कुछ समय के पश्चात् यह भाग टूट कर गिर जाता है। इससे तटों पर खड़े किनारों की रचना होती है जिसे भृगु ( Cliff) कहते हैं। भारत के पश्चिमी तट पर भृगुओं के उदाहरण मिलते हैं।
JAC Class 11 Geography Important Questions Chapter 7 भू-आकृतियाँ तथा उनका विकास 5

3. समुद्री गुफाएं (Sea Caves): आधार की कोमल चट्टानों के टूटने या घुल जाने पर तट के समीप खोखले स्थान बन जाते हैं । लहरों द्वारा वायु के बार-बार घुसने व निकलने की क्रिया से ये स्थान चौड़े हो जाते हैं तथा गुफाएं बन जाती हैं। दो पड़ोसी गुफाओं के बीच की दीवार टूट जाने से मेहराब (Arch) बन जाती है जिसे प्राकृतिक पुल भी कहते हैं, जैसे- इंग्लैण्ड में Needle’s Eye।
JAC Class 11 Geography Important Questions Chapter 7 भू-आकृतियाँ तथा उनका विकास 6

4. वात छिद्र (Blow Holes): वायु के दबाव से कई बार गुफा की छत में छिद्र हो जाते हैं, इनमें से जल फव्वारे के रूप में बाहर निकलता है। इन छिद्रों से पवन सीटी बजाती हुई ऊपर निकलती है और साथ ही पानी फव्वारे के रूप में बाहर निकलता है। इन्हें उगलने वाले छिद्र (Spouting Horns) कहते हैं।

5. सागरीय स्तम्भ (Stack): मेहराब के छत के टूट जाने से कठोर चट्टानों के कुछ अंश स्तम्भ के रूप में खड़े रह जाते हैं जिन्हें स्तम्भ (Stack) कहते हैं। इंग्लैण्ड में St. Kilda के स्तम्भ 627 फुट ऊंचे हैं।

परिवहन का कार्य (Work of Transportation ): समुद्र में खुरचा हुआ पदार्थ लहरों, वायु तथा धाराओं द्वारा परिवहन होता है। लहरों द्वारा सागरीय पदार्थों का परिवहन दो रूपों में होता है:

  1. तट की ओर
  2. सागर की ओर

निक्षेप के कार्य (Work of Deposition): लहरों द्वारा अपरदन से प्राप्त पदार्थ तट के सहारे या तट के दूर जमा होते हैं। लहरों के नीचे के प्रवाह (Undertow) के कारण तट के दूर सागर में निक्षेप होता है। इससे तट का ढाल कम होता है तथा तट का विस्तार आगे बढ़ता रहता है। इस निक्षेप से कई भू-आकार बनते हैं:

1. पुलिन अथवा बीच (Beach ):
सागरीय तट के साथ-साथ मलवा के निक्षेप से बनी तटीय श्रेणियों को पुलिन कहते हैं। तट के निकट ये ऊंचे से प्रदेश क्रीड़ा के उत्तम स्थल बन जाते हैं। समतल तटों पर चौड़े पुलिन मिलते हैं, परन्तु पर्वतीय भागों में पुलिन कम मिलते हैं, जैसे – चिल्ली का तट।

2. रोधिका (Bars):
लहरों द्वारा तट के समानान्तर निक्षेप से ऊंची मुंडेर या श्रेणी बन जाती है। इन्हें तटीय रोध (Offshore Bars) कहते हैं। ये तट से दूर खुले समुद्र में बनती हैं। ये श्रेणियां तट रेखा तथा सागरीय जल के बीच रुकावट का कार्य करती हैं।

3. लैगून झीलें (Lagoons):
यदि रोधिका के दोनों छोर स्थल भाग से जुड़ जाते हैं तो बीच में खारे जल की झील बन जाती है। तट रेखा और मुंडेरों के बीच का जल एक छोटी झील का रूप धारण कर लेता है जिसे लैगून कहते हैं। भारत के पूर्वी तट पर चिल्का (Chilka) तथा पुलीकट (Pulicat) झीलें तथा केरल में वेम्बनाद झील इसी प्रकार बनी हैं।

4. भूजिहवा (Spit ):
भूजिहवा लहरों के निक्षेप से बनी एक रेतीली श्रेणी होती है। यह एक बांध के समान है जिसका एक सिरा तट से जुड़ा होता है तथा दूसरा सिरा खुले सागर में डूबा रहता है।

JAC Class 11 Geography Solutions Chapter 6 मृदा

Jharkhand Board JAC Class 11 Geography Solutions Chapter 6 मृदा Textbook Exercise Questions and Answers.

JAC Board Class 11 Geography Solutions Chapter 6 मृदा

बहु-विकल्पी प्रश्न (Multiple Choice Questions)

प्रश्न- दिए गए चार वैकल्पिक उत्तरों में से सही उत्तर चुनिए
1. भारत में कौन-सी मृदा सबसे विस्तृत उपजाऊ है?
(A) जलोढ़।
(B) काली मृदा
(C) लेटराइट
(D) वन मृदा।
उत्तर:
(A) जलोढ़।

2. किस मृदा को रेगड़ मृदा भी कहते हैं?
(A) नमकीन
(B) काली
(C) शुष्क
(D) लेटराइट।
उत्तर:
(B) काली।

JAC Class 11 Geography Solutions Chapter 6 मृदा

3. मृदा की ऊपरी तह के उड़ जाने का मुख्य कारण
(A) पवन अपरदन
(B) अपक्षातान
(C) जल अपरदन
(D) कोई नहीं।
उत्तर:
(A) पवन अपरदन।

4. कृषिकृत भूमि में जल सिंचित क्षेत्र में लवण का क्या कारण है?
(A) जिप्सम
(B) अति जल सिंचाई
(C) अति पशुचारण
(D) उर्वरक।
उत्तर:
(B) अति जल सिंचाई।

लघु उत्तरीय प्रश्न (Short Answer Type Questions)

प्रश्न 1.
मृदा क्या है?
उत्तर:
मृदा भूतल की ऊपरी सतह का आवरण है। भू-पृष्ठ पर मिलने वाले असंगठित पदार्थों की ऊपरी परत को मृदा कहते हैं।

JAC Class 11 Geography Solutions Chapter 6 मृदा

प्रश्न 2.
मृदा निर्माण के प्रमुख उत्तरदायी कारक कौन-से हैं?
उत्तर:

  1. मूल पदार्थ
  2. उच्चावच
  3. जलवायु
  4. प्राकृतिक वनस्पति।

प्रश्न 3.
मृदा परिच्छेदिका के तीन संस्तरों के नामों का उल्लेख करो।
उत्तर:

  1. A-स्तर
  2. B-स्तर
  3. C-स्तर।

प्रश्न 4.
मृदा अवकर्षण क्या होता है?
उत्तर:
मृदा की उर्वरता के ह्रास को मृदा अवकर्षण कहते हैं। इससे मृदा का पोषण स्तर गिर जाता है तथा मृदा की गहराई कम हो जाती है।

प्रश्न 5.
खादर तथा बांगर में क्या अन्तर है?
उत्तर:
पुरातन जलोढ़ के निक्षेप से बने ऊंचे प्रदेश को बांगर कहते हैं। नवीन जलोढ़ के निक्षेप से बने निचले प्रदेश को खादर कहते हैं।

निबन्धात्मक प्रश्न (Essay Type Questions)

प्रश्न – निम्नलिखित प्रश्नों के उत्तर 125 शब्दों तक दीजिए
प्रश्न 1.
काली मृदाएं किसे कहते हैं? इनके निर्माण तथा विशेषताओं का वर्णन कीजिए
उत्तर:
काली मिट्टी (Black Soil)

1. विस्तार ( Extent ):
इस मिट्टी का विस्तार प्रायद्वीप के उत्तर-पश्चिमी भाग में लगभग 5 लाख वर्ग किलोमीटर क्षेत्र में है। यह गुजरात, महाराष्ट्र के अधिकांश भाग, मध्य प्रदेश, दक्षिणी उड़ीसा, उत्तरी कर्नाटक, दक्षिणी आंध्र प्रदेश में मिलती है। यह मिट्टी नर्मदा, ताप्ती, गोदावरी तथा कृष्णा नदी की घाटियों में पाई जाती है।

2. विशेषताएं (Characteristics):
इस मिट्टी का निर्माण लावा प्रवाह से बनी आग्नेय चट्टानों के टूटने-फूटने से हुआ है। यह अधिकतर दक्कन लावा (Deccan Trap) के क्षेत्र में मिलता है। इसलिए इसे लावा मिट्टी भी कहते हैं। इस मिट्टी का रंग काला होता है इसलिए इसे काली मिट्टी (Black Soil) भी कहते हैं। इसे रेगड़ मिट्टी (Regur Soil) भी कहा जाता है। इस मिट्टी में चूना, लोहा, मैग्नीशियम की मात्रा अधिक होती है, परन्तु फॉस्फोरस, पोटाश, नाइट्रोजन तथा जैविक पदार्थों की कमी होती है। इस मिट्टी में नमी धारण करने की शक्ति अधिक है। इसलिए सिंचाई की आवश्यकता कम होती है। इस मिट्टी की तुलना में इसकी ‘शर्नीज़म’ तथा अमेरिका की ‘प्रेयरी’ मिट्टी से की जाती है।

3. फसलों की उपयुक्तता (Suitability to Crops):
यह मिट्टी कपास की कृषि के लिए उपयुक्त है। इसलिए इसे ‘कपास की मिट्टी’ (Cotton Soil) भी कहते हैं। पठारों की उच्च भूमि में कम उपजाऊ होने के कारण यहां ज्वारबाजरा, दालों आदि की कृषि होती है। मैदानी भैग में गेहूं, कपास, तम्बाकू, मूंगफली तथा तिलहन की कृषि की जाती है।

JAC Class 11 Geography Solutions Chapter 6 मृदा

प्रश्न 2.
मृदा संरक्षण क्या होता है? मृदा संरक्षण के कुछ उपाय सुझाइए
उत्तर:
मृदा संरक्षण यदि मृदा अपरदन और मृदाक्षय मानव द्वारा किया जाता है, तो स्पष्टतः मानवों द्वारा ही इसे रोका भी जा सकता है। मृदा संरक्षण एक विधि है, जिसमें मिट्टी की उर्वरता बनाए रखी जाती है, मिट्टी के अपरदन और क्षय को रोका जाता है और मिट्टी की अपक्षरित दशाओं को सुधारा जाता है। मृदा अपरदन वास्तव में मनुष्यकृत समस्या है।

1. किसी भी तर्कसंगत समाधान में पहला काम ढालों की कृषि योग्य खुली भूमि पर खेती को रोकना है। 15 से 25 प्रतिशत डाल वाली भूमि का उपयोग कृषि के लिए नहीं होना चाहिए। यदि ऐसी भूमि पर खेती करना ज़रूरी हो जाए, तो इस पर सावधानी से सीढ़ीदार खेत बना लेने चाहिएं।

2. भारत के विभिन्न भागों में, अति चराई और झूम कृषि ने भूमि के प्राकृतिक आवरण को दुष्प्रभावित किया है। इसी प्रकार विस्तृत क्षेत्र अपरदन की चपेट में आ गए हैं। ग्रामवासियों को इनके दुष्परिणामों से अवगत करवा कर इन्हें ( अति चराई और झूम कृषि) नियमित और नियन्त्रित करना चाहिए।

3. समोच्च रेखा के अनुसार मेड़बंदी, समोच्च रेखीय सीढ़ीदार खेती बनाना, नियमित वानिकी, नियन्त्रित चराई, वरणात्मक खरपतवार नाशन, आवरण फसलें उगाना, मिश्रित खेती तथा शस्यावर्तन, उपचार के कुछ ऐसे तरीके हैं, जिनका उपयोग मृदा अपरदन को कम करने के लिए प्रायः किया जाता है।

4. अवनालिका अपरदन को रोकने तथा उनके बनने पर नियन्त्रण के प्रयत्न किए जाने चाहिएं। अंगुल्याकार अवनालिकाओं को सीढ़ीदार खेत बनाकर खत्म किया जा सकता है। अवनालिकाओं के शीर्ष की ओर के विकास को नियन्त्रित करने पर विशेष ध्यान दिया जाना चाहिए। इस कार्य को अवनालिकाओं को बन्द करके, सीढ़ीदार खेत बनाकर या आवरण वनस्पति का रोपण करके किया जा सकता है।

5. मरुस्थलीय और अर्द्ध-मरुस्थलीय क्षेत्रों में कृषि योग्य भूमि पर बालू के टीलों के प्रसार को वनों की रक्षक मेखला बनाकर रोकना चाहिए। कृषि के अयोग्य भूमि को चराई के लिए चरागाहों में बदल देना चाहिए। बालू के टीलों को स्थिर करने के उपाय भी अपनाए जाने चाहिएं।

JAC Class 11 Geography Solutions Chapter 6 मृदा

भारत – स्थिति  JAC Class 11 Geography Notes

→ मिट्टी (Soil): भू-पृष्ठ पर मिलने वाले असंगठित पदार्थों की ऊपरी पर्त को मृदा कहते हैं।

→ मिट्टी के तत्त्व (Constituents of Soil): खनिज, ह्यूमस, जल, वायु तथा बैक्टीरिया ।

→ मिट्टी निर्माण के लिये आवश्यक तत्त्व (Formation of Soil): मूल पदार्थ, धरातल, जलवायु, ऋतु प्रहार।

→ मृदा के संस्तर (Horizons of Soil) अ – संस्तर, ब- संस्तर, स- संस्तर

→ मिट्टी की महत्ता (Importance of Soils ): मिट्टी एक महत्त्वपूर्ण संसाधन है। मानव तथा आर्थिक क्रियाएं मिट्टी का उपजाऊपन पर निर्भर करती हैं।

→ भारत में मिट्टी के प्रकार (Types of Soils): भारतीय मिट्टी के समूह निम्नलिखित हैं

  • काली मिट्टी
  • लाल मिट्टी
  • लेटराइट मिट्टी
  • जलोढ़ मिट्टी
  • मरुस्थलीय मिट्टी
  • पर्वतीय मिट्टी

→ मृदा अपरदन (Soil erosion ): जल, वायु आदि द्वारा मिट्टी की ऊपरी परत का बहना मृदा अपरदन कहलाता है।

→ मृदा अपरदन के कारण (Causes of Soil Erosion): तीव्र ढलाने, भारी वर्षा, तेज़ हवाएं, चरागाहों का मृदा अत्यधिक प्रयोग, वनों की कटाई आदि।

→ मृदा संरक्षण (Conservation of Soil): मृदा संरक्षण के लिए मृदा का संरक्षण, पुन: नवीकरण तथा उपजाऊपन कायम रखना आवश्यक है। इसके लिए वनारोपण, नियन्त्रित पशु चारण, सीढ़ीनुमा कृषि तथा फ़सलों का हेर-फेर आवश्यक है।

JAC Class 11 Geography Solutions Chapter 5 प्राकृतिक वनस्पति

Jharkhand Board JAC Class 11 Geography Solutions Chapter 5 प्राकृतिक वनस्पति Textbook Exercise Questions and Answers.

JAC Board Class 11 Geography Solutions Chapter 5 प्राकृतिक वनस्पति

बहु-विकल्पी प्रश्न (Multiple Choice Questions)

दिए गए चार वैकल्पिक उत्तरों में से सही उत्तर चुनिए
1. प्राजैक्ट टाइगर का उद्देश्य क्या था?
(A) शेरों का शिकार करना
(B) अवैध शिकार को रोक कर शेरों की सुरक्षा
(C) शेरों को चिड़ियाघरों में रखना
(D) शेरों पर चित्र बनाना।
उत्तर:
(B) अवैध शिकार को रोककर शेरों की सुरक्षा।

2. नन्दा देवी जीव आरक्षण क्षेत्र किस राज्य में है?
(A) बिहार
(B) उत्तराखण्ड
(C) उत्तर प्रदेश
(D) उड़ीसा।
उत्तर:
(B) उत्तराखण्ड।

JAC Class 11 Geography Solutions Chapter 5 प्राकृतिक वनस्पति

3. संदल किस प्रकार के वन की लकड़ी है?
(A) सदाबहार
(B) डैल्टा वन
(C) पतझड़ीय
(D) कंटीले वन।
उत्तर:
(C) पतझड़ीय।

4. IUCN द्वारा कितने जीव आरक्षण स्थल मान्यता प्राप्त है?
(A) 1
(B) 2
(C) 3
(D) 4
उत्तर:
(D) 4

5. वन नीति के अधीन वन क्षेत्र का लक्ष्य कितना था?
(A) 33%
(B) 53%
(C) 44%
(D) 22%.
उत्तर:
(A) 33%.

लघु उत्तरीय प्रश्न (Short Answer Type Questions)

प्रश्न 1.
प्राकृतिक वनस्पति क्या है? जलवायु की किन परिस्थितियों में ऊष्ण कटिबन्धीय वन उगते हैं?
उत्तर:
प्राकृतिक वनस्पति से अभिप्राय उस पौधा समुदाय से है जो लम्बे समय तक बिना किसी बाहरी हस्तक्षेप के उगता है। इसकी विभिन्न प्रजातियां मृदा व जलवायु के अनुसार पनपती हैं। उष्ण कटिबन्धीय वन-ये वन उष्ण एवं आई प्रदेशों में उगते हैं। यहां वार्षिक वर्षा 200 सें० मी० से अधिक तथा औसत वार्षिक तापमान 22°C से अधिक रहता है। इससे कम वर्षा तथा तापमान में उष्ण कटिबन्धीय पर्णपाती वन मिलते हैं।

JAC Class 11 Geography Solutions Chapter 5 प्राकृतिक वनस्पति

प्रश्न 2.
जलवायु की कौन-सी परिस्थितियां सदाबहार वन उगने के लिए अनुकूल हैं?
उत्तर:

  1. वार्षिक वर्षा 200 सें० मी०
  2. औसत वार्षिक तापमान 22°C. ये दशाएं भूमध्य रेखीय जलवायु में मिलती हैं।

प्रश्न 3.
सामाजिक वानिकी से आपका क्या अभिप्रायः है?
उत्तर:
सामाजिक वानिकी का अर्थ है- पर्यावरणीय, सामाजिक व ग्रामीण विकास में मदद करने के उद्देश्य से वनों का प्रबन्ध और सुरक्षा तथा ऊसर भूमि पर वनारोपण। इसका उद्देश्य है ग्रामीण जनसंख्या के लिए ज्लावन छोटी इमारती लकड़ी और छोटे-छोटे वन उत्पादों की पूर्ति करना।

JAC Class 11 Geography Solutions Chapter 5 प्राकृतिक वनस्पति

प्रश्न 4.
जीवमण्डल निचय को परिभाषित करें, वन क्षेत्र और वन आवरण में क्या अन्तर है?
उत्तर:
जीवमण्डल निचय आरक्षित क्षेत्र (Bioreserves) विशेष प्रकार के भौमिक और तटीय पारिस्थितिक तन्त्र हैं जिन्हें UNESCO द्वारा मान्यता प्राप्त है। वन क्षेत्र और वन आवरण एक-दूसरे से भिन्न-भिन्न हैं। वन क्षेत्र राजस्व विभाग के अनुसार अधिसूचित क्षेत्र हैं चाहे वहां वृक्ष हो या न हो। वन आवरण वास्तविक रूप से वनों से ढका प्रदेश है। सन् 2001 में भारत में वन आवरण, 20.55% था, परन्तु इनमें 12.6% सघन तथा 7.8% विरल वन थे।

निबन्धात्मक प्रश्न (Essay Type Questions)

प्रश्न-निम्नलिखित प्रश्नों के उत्तर लगभग 125 शब्दों में दोप्रश्न
प्रश्न 1.
वन संरक्षण के लिए क्या कदम उठाए गए हैं?
उत्तर:
जनसंख्या के अत्यधिक दबाव तथा पशुओं की संख्या में अत्यधिक वृद्धि के कारण वन सम्पदा का संरक्षण आवश्यक है। वन संरक्षण कृषि एवं चराई के लिए अधिक भूमि की आवश्यकता के कारण आवश्यक है। इसके लिए वनवर्द्धन के उत्तम तरीकों को अपनाया जा रहा है। तेज़ी से उगने वाले पौधों की जातियों को लगाया जा रहा है। घास के मैदानों का पुनर्विकास किया जा रहा है। वन क्षेत्रों का विस्तार किया जा रहा है।
वनों का जीवन तथा पर्यावरण के साथ जटिल सम्बन्ध है, वन संरक्षण के लिए कई कदम उठाए गए हैं:

  1. वन नीति-1952 तथा 1988 में वन संरक्षण हेतु वन नीति लागू की गई।
  2. वन संसाधनों का सतत्-पोषणीय विकास किया जाएगा तथा स्थानीय लोगों की आवश्यकताओं की पूर्ति होगी।
  3. देश में 33% वन लगाने के यत्न किए जा रहे हैं।
  4. वनों में जैव विविधता तथा पारिस्थितिक सन्तुलन कायम रखा जाएगा।
  5. मृदा अपरदन तथा बाढ़ पर नियन्त्रण।
  6. सामाजिक वानिकी तथा वनरोपण द्वारा वन आवरण का विस्तार हो रहा है।
  7. वनों की उत्पादकता को बढ़ाया जा रहा है।

JAC Class 11 Geography Solutions Chapter 5 प्राकृतिक वनस्पति

प्रश्न 2.
वन और वन्य जीव संरक्षण में लोगों की भागीदारी कैसे महत्त्वपूर्ण है?
उत्तर:
वन संरक्षण में लोगों की भागीदारी द्वारा सामाजिक वानिकी की विचारधारा का प्रयोग किया जा रहा है। इसमें ग्रामीण क्षेत्रों की संस्थाएं तथा महिलाएं योगदान दे रही हैं।
1. 1976 के राष्ट्रीय कृषि आयोग ने पहले-पहल ‘सामाजिक वानिकी’ शब्दावली का प्रयोग किया था। इसका अर्थ है-ग्रामीण जनसंख्या के लिए जलावन, छोटी इमारती लकड़ी और छोटे-छोटे वन उत्पादों की आपूर्ति करना।

2. अधिकतर राज्यों में वन विभागों के अन्तर्गत सामाजिक वानिकी के अलग से प्रकोष्ठ बनाए गए हैं।

3. सामाजिक वानिकी के मुख्य रूप से तीन अंग हैं: कृषि वानिकी किसानों को अपनी भूमि पर वृक्षारोपण के लिए प्रोत्साहित करना, वन-भूखण्ड (वुडलाट्स) वन विभागों द्वारा लोगों की ज़रूरतों को पूरा करने के लिए सड़कों, के किनारे, नहर के तटों तथा ऐसी अन्य सार्वजनिक भूमि पर वृक्षारोपण, सामुदायिक वन-भूखण्ड लोगों द्वारा स्वयं बराबर की हिस्सेदारी के आधार पर भूमि पर वृक्षारोपण।

4. यह लोगों की आधारभूत आवश्यकताओं को पूरा करने वाले कार्यक्रम के स्थान पर किसानों का धनोपार्जन कार्यक्रम बन गया।

प्राकृतिक वनस्पति JAC Class 11 Geography Notes

→ वन-एक प्राकृतिक साधन (Forests-A Natural Resource): वन एक नदी की भाँति बहुमुखी। संसाधन हैं। किसी देश का आर्थिक विकास वनों के उपयोग पर निर्भर करता है। जलवायु तथा प्राकृतिक वनस्पति-जलवायु तथा प्राकृतिक वनस्पति में घना सम्बन्ध है। विभिन्न प्रकार के वन-सदाबहार, पतझड़ीय, शुष्क; सभी वन तापमान तथा वर्षा पर निर्भर करते हैं।

→ वनों की महत्ता (Importance of Forests):

  • वन हमें भोजन सम्बन्धी वस्तुएं प्रदान करते हैं।
  • इनसे लकड़ी प्राप्त होती है।
  • वन विश्व का 40% ईंधन प्रदान करते हैं। लकड़ी का प्रयोग विभिन्न प्रकार से किया जाता है, जैसे घरों में ईंधन, प्रगलन उद्योगों में तथा रेल इंजनों में।
  • नर्म लकड़ी, लकड़ी की लुग्दी कागज़, रेयॉन उद्योग के लिये कच्चा माल, प्रदान करती है।
  • वन वायु में नमी ग्रहण करके वर्षा लाने में सहायता करते हैं।
  • वन मिट्टी अपरदन तथा बाढ़ को रोकते हैं।
  • ये मरुस्थलों को बढ़ने से रोकते हैं।

→ भारत में वनों के प्रकार (Types of Forests in India): भारत में वर्षा, तापमान तथा ऊंचाई की विभिन्नता होने के कारण विभिन्न प्रकार के वन पाये जाते हैं

  • उष्ण कटिबन्धीय सदाबहार वन
  • पतझड़ीय वन
  • शुष्क वन
  • ज्वारीय वन
  • पर्वती वन।

→ वनों का ह्रास (Depletion of Forests): निम्नलिखित कारणों द्वारा उत्तम वनों का विशाल क्षेत्रों पर ह्रास हुआ है

  • विशाल पैमाने पर वन क्षेत्र की कटाई
  • स्थानान्तरी कृषि
  • भारी मृदा अपरदन
  • चरागाहों की अत्यधिक चराई
  • लकड़ी तथा ईंधन के लिये वनों की कटाई
  • भूमि का मानव के लिये प्रयोग।

→ वनों का संरक्षण (Conservation of Forests): देश के वन संसाधनों पर जनसंख्या का भारी दबाव है। बढ़ती। हुई जनसंख्या को कृषि योग्य भूमि की अधिक आवश्यकता है। इसलिए वन संरक्षण उपाय आवश्यक हैं।

→ वृक्षारोपण का विभिन्न क्षेत्रों में विकास हुआ है। घास के मैदानों का विकास किया गया है। रेशम के कीड़े पालने के विकसित तरीकों का प्रयोग किया जा रहा है। तेज़ी से बढ़ने वाले पौधों को लगाया जा रहा है। वनों । के अन्तर्गत क्षेत्रों में वृद्धि की गई है। राष्ट्रीय वन नीति-1988 की राष्ट्रीय वन नीति ने वनों का संरक्षण, पुनः विकास किया है। इसका उद्देश्य वातावरण, का संरक्षण तथा पारिस्थितिक सन्तुलन बनाये रखना है। इसके अन्य उद्देश्य सामाजिक वाणिकी तथा वन उत्पादन में वृद्धि करना है।

JAC Class 11 Geography Solutions Chapter 4 जलवायु

Jharkhand Board JAC Class 11 Geography Solutions Chapter 4 जलवायु Textbook Exercise Questions and Answers.

JAC Board Class 11 Geography Solutions Chapter 4 जलवायु

बहु-विकल्पी प्रश्न (Multiple Choice Questions)

दिए गए चार वैकल्पिक उत्तरों में से सही उत्तर चुनिए
1. शीतकाल में तमिलनाडु के तटीय मैदान में किन पवनों द्वारा वर्षा होती है?
(A) दक्षिण-पश्चिम मानसून
(B) उत्तर-पूर्वी मानसून
(C) शीतोष्ण चक्रवात
(D) स्थानिक पवनों।
उत्तर:
(B) उत्तर-पूर्वी मानसून

2. कितने प्रतिशत भाग में भारत में 75 सें० मी० से कम वार्षिक वर्षा होती है?
(A) आधे
(B) दो-तिहाई
(C) एक-तिहाई
(D) तीन चौथाई।
उत्तर:
(D) तीन चौथाई।

JAC Class 11 Geography Solutions Chapter 4 जलवायु

3. दक्षिणी भारत के सम्बन्ध में कौन-सा कथन सही नहीं?
(A) दैनिक तापान्तर कम है
(B) वार्षिक तापान्तर कम है
(C) वर्ष भर तापमान अधिक है
(D) यहां कठोर जलवायु है।
उत्तर:
(D) यहां कठोर जलवायु है।

4. जब सूर्य दक्षिणी गोलार्द्ध में मकर रेखा पर लम्ब चमकता है तो क्या होता है?
(A) उत्तर-पश्चिमी भारत में उच्च वायु दाब
(B) उत्तर-पश्चिमी भारत में निम्न वायु दाब
(C) उत्तर-पश्चिमी भाग में तापमान-वायु दाब में
(D) उत्तर-पश्चिमी भाग में लू चलती है।
उत्तर:
(A) उत्तर-पश्चिमी भारत में उच्च वायु दाब।

5. भारत के किस देश-प्रदेश में कोपेन के ‘AS’ वर्ग की जलवायु है?
(A) केरल तथा कर्नाटक
(B) अण्डेमान निकोबार द्वीप
(C) कोरोमण्डल तट
(D) असम-अरुणाचल।
उत्तर:
(C) कोरोमण्डल तट।

लघु उत्तरीय प्रश्न (Short Answer Type Questions)

प्रश्न 1.
भारतीय मौसम के रचना तन्त्र को प्रभावित करने वाले तीन महत्त्वपूर्ण कारक कौन-कौन से हैं?
उत्तर:
भारतीय मौसम के रचना तन्त्र को निम्नलिखित तीन कारक प्रभावित करते हैं
1. दाब तथा हवा का धरातलीय वितरण जिसमें मानसून पवनें, कम वायु दाब क्षेत्र तथा उच्च वायु दाब क्षेत्रों की स्थिति महत्त्वपूर्ण कारक हैं।
2. ऊपरी वायु परिसंचरण (Upper air Circulation) में विभिन्न वायु राशियां तथा जेट प्रवाह महत्त्वपूर्ण तत्त्व हैं।
3. विभिन्न वायु विक्षोभ (Atmospheric disturbances) में ऊष्ण कटिबन्धीय तथा पश्चिमी चक्रवात द्वारा वर्षा होना महत्त्वपूर्ण प्रभाव डालता है।

JAC Class 11 Geography Solutions Chapter 4 जलवायु

प्रश्न 2.
अन्तर ऊष्ण कटिबन्धीय अभिसरण कटिबन्ध (I.T.C.Z.) क्या है?
उत्तर:
अन्तर ऊष्ण कटिबन्धीय अभिसरण कटिबन्ध (I.T. C. Z. ):
भूमध्यरेखीय निम्न वायुदाब कटिबन्ध है जो धरातल के निकट पाया जाता है। इसकी स्थिति ऊष्ण कटिबन्ध के बीच सूर्य की स्थिति के अनुसार बदलती रहती है। ग्रीष्मकाल में इसकी स्थिति उत्तर की ओर तथा शीतकाल में दक्षिण की ओर सरक जाती है । यह भूमध्य रेखीय निम्न दाब द्रोणी दोनों दिशाओं में हवाओं के प्रवाह को प्रोत्साहित करती है ।

प्रश्न 3.
‘मानसून प्रस्फोट’ किसे कहते हैं? भारत में सबसे अधिक वर्षा वाला स्थान बताओ।
उत्तर:
भारत के पश्चिमी तट पर अरब सागर की मानसून पवनें दक्षिणी-पश्चिमी दिशा में चलती हैं। यहां ये पवनें जून के प्रथम सप्ताह में अचानक आरम्भ हो जाती हैं। मानसून के इस अकस्मात आरम्भ को ‘मानसून प्रस्फोट’ (Monsoon Burst) कहा जाता है क्योंकि मानसून आरम्भ होने पर बड़े ज़ोर की वर्षा होती है। जैसे किसी ने पानी से भरा गुब्बारा फोड़ दिया हो। भारत में मासिनराम (Mawsynram) में सबसे अधिक वार्षिक वर्षा ( 1080 सें० मी०) होती है। यह स्थान खासी पहाड़ियों में स्थित है।

प्रश्न 4.
जलवायु प्रदेश क्या होता है? कोपेन की पद्धति के प्रमुख आधार कौन से हैं?
उत्तर:
जलवायु प्रदेश – वह प्रदेश जहां वायुमण्डलीय दशाएं तापमान, वर्षा, मेघ, पवनें, वायुदाब आदि सब में लगभग समानता है। कोपेन की जलवायु पद्धति का आकार – कोपेन ने भारत के जलवायु प्रदेशों का वर्गीकरण किया है। इस वर्गीकरण का आधार दो तत्त्वों पर आधारित है। इसमें तापमान तथा वर्षा के औसत मासिक मान का विश्लेषण किया गया है। प्राकृतिक वनस्पति द्वारा किसी स्थान के तापमान और वर्षा के प्रभाव को आंका जाता है।

JAC Class 11 Geography Solutions Chapter 4 जलवायु

प्रश्न 5.
उत्तर-पश्चिमी भारत में रबी की फसलें बोने वाले किसानों को किस प्रकार के चक्रवातों से वर्षा प्राप्त होती है? वे चक्रवात कहां उत्पन्न होते हैं?
उत्तर:
भारत में उत्तर-पश्चिमी भाग में पंजाब, हरियाणा, हिमाचल तथा पश्चिमी उत्तर प्रदेश में शीतकाल में चक्रवातीय वर्षा होती है। ये चक्रवात पश्चिमी एशिया तथा भूमध्यसागर में उत्पन्न होते हैं तथा पश्चिमी जेट प्रवाह के साथ-साथ भारत में पहुंचते हैं। औसतन शीत ऋतु में 4 से 5 चक्रवात दिसम्बर से फरवरी के मध्य इस प्रदेश में वर्षा करते हैं। पर्वतीय भागों में हिमपात होता है। पूर्व की ओर इस वर्षा की मात्रा कम होती है। इन प्रदेशों में वर्षा 5 सेंटीमीटर से 25 सेंटीमीटर तक होती है। ये चक्रवात रबी की फसलों गेहूँ, बाजरा आदि के लिए लाभदायक हैं

(ग) निबन्धात्मक प्रश्न (Essay Type Questions )

निम्नलिखित प्रश्नों के उत्तर लगभग 125 शब्दों में लिखिए

प्रश्न 1.
जलवायु में एक प्रकार का ऐक्य होते हुए भी भारत की जलवायु में क्षेत्रीय विभिन्नताएं पाई जाती हैं। उपयुक्त उदाहरण देते हुए इस कथन को स्पष्ट कीजिए।
उत्तर:
भारत एक विशाल देश है। यहां पर अनेक प्रकार की जलवायु मिलती है। परन्तु मुख्य रूप से भारत मानसून पवनों के प्रभावाधीन है। मानसून पवनों के प्रभाव के कारण देश में जलवायविक एकता पाई जाती है। फिर भी देश के विभिन्न भागों में तापमान, वर्षा आदि जलवायु तत्त्वों में काफ़ी अन्तर पाए जाते हैं। ये प्रादेशिक अन्तर एक प्रकार से मानसूनी जलवायु के उपभेद हैं। आधारभूत रूप से सारे देश में जलवायु की व्यापक एकता पाई जाती है
प्रादेशिक अन्तर मुख्य रूप से तापमान, वायु तथा वर्षा के ढांचे में पाए जाते हैं।

1. राजस्थान के मरुस्थल में, बाड़मेर में ग्रीष्मकाल में 50°C तक तापमान मापे जाते हैं। इसके पर्वतीय प्रदेशों में तापमान 20°C सैंटीग्रेड के निकट रहता है।

2. दिसम्बर मास में द्रास एवं कारगिल में तापमान 40°C तक पहुंच जाता है जबकि तिरुवनन्तपुरम तथा चेन्नई में तापमान + 28°C रहता है ।

3. इसी प्रकार औसत वार्षिक वर्षा में भी प्रादेशिक अन्तर पाए जाते हैं। एक ओर मासिनराम में (1080 सेंटीमीटर) संसार में सबसे अधिक वर्षा होती है तो दूसरी ओर राजस्थान शुष्क रहता है। जैसलमेर में वार्षिक वर्षा शायद ही 12 सें०मी० से अधिक होती है । गारो पहाड़ियों में स्थित तुरा नामक स्थान में एक ही दिन में उतनी वर्षा हो सकती है जितनी जैसलमेर में दस वर्षों में होती है ।

4. कई भागों में मानसून वर्षा जून के पहले सप्ताह में आरम्भ हो जाती है तो कई भागों में जुलाई में वर्षा की प्रतीक्षा हो रही होती है।

5. तटीय भागों में मौसम की विषमता महसूस नहीं होती परन्तु अन्तःस्थ स्थानों में मौसम विषम रहता है। शीतकाल में उत्तर-पश्चिमी भारत में शीत लहर चल रही होती है तो दक्षिणी भारत में काफ़ी ऊंचे तापमान पाए जाते हैं। इस प्रकार विभिन्न प्रदेशों में ऋतु की लहर लोगों की जीवन पद्धति में एक परिवर्तन तथा विभिन्नता उत्पन्न कर देती है। इस प्रकार इन उदाहरणों से स्पष्ट है कि भारतीय जलवायु में एक व्यापक एकता होते हुए भी प्रादेशिक अन्तर पाए जाते हैं।

JAC Class 11 Geography Solutions Chapter 4 जलवायु

प्रश्न 2.
भारतीय मौसम विज्ञान विभाग के अनुसार भारत में कितने स्पष्ट मौसम पाए जाते हैं ? किसी एक मौसम की दशाओं की सविस्तार व्याख्या कीजिए।
उत्तर:
मानसून व्यवस्था के अनुसार भारत में चार ऋतुएं होती हैं

  1. शीत ऋतु (जनवरी-फरवरी)।
  2. ग्रीष्म ऋतु (मार्च से मध्य जून )।
  3. वर्षा ऋतु (मध्य जून से मध्य सितम्बर)।
  4. लौटती मानसून की ऋतु (मध्य सितम्बर से दिसम्बर)।

1. शीत ऋतु:
कम तापमान, मेघ रहित आकाश, सुहावना मौसम तथा कम आर्द्रता इस ऋतु की विशेषताएं होती हैं। इस ऋतु में सूर्य की स्थिति दक्षिणी गोलार्द्ध में होने के कारण भारत में कम तापमान मिलते हैं। शीत लहर तथा हिमपात के कारण उत्तरी गोलार्द्ध में औसत तापमान 21°C से कम रहते हैं। रात्रि का तापमान हिमांक से नीचे हो जाता है। दक्षिणी भारत में सागरीय प्रभाव के कारण 21°C से अधिक तापमान पाए जाते हैं।

यहां कोई शीत नहीं होती उत्तर – पश्चिमी भारत में उच्च वायु दाब क्षेत्र बन जाता है। उत्तर-पश्चिमी भारत में भूमध्य सागर से आने वाले चक्रवात हल्की-हल्की वर्षा करते हैं। यह वर्षा रबी की फसल के लिए लाभदायक होती है। मानसून पवनें स्थल से सागर (Land to Sea) की ओर चलने के कारण शुष्क होती हैं। ये लौटती हुई मानसून पवनें खाड़ी बंगाल से नमी ग्रहण करके पूर्वी तट पर वर्षा करती हैं।

2. ग्रीष्म ऋतु:
ग्रीष्म ऋतु में तापमान बढ़ना आरम्भ कर देता है। सूर्य की किरणें कर्क रेखा पर लम्बवत् पड़ती हैं।
उत्तर:
पश्चिमी भारत में उच्चतम तापमान 42°C से अधिक होते हैं। दिन के समय शुष्क, गर्म लू चलती है। दक्षिणी भारत में औसत तापमान 25°C तक रहता है। तटीय भागों में सागर के समकारी प्रभाव के कारण मौसम सुहावना रहता है। उत्तर – पश्चिमी भारत में निम्न वायुदाब केन्द्र बन जाता है। यह निम्न दाब दक्षिणी-पश्चिमी मानसून पवनों को उत्तर की ओर आकर्षित करता है। ग्रीष्म ऋतु प्रायः शुष्क रहती है। उत्तर-पूर्वी भारत में उष्ण कटिबन्धीय चक्रवातों के कारण कुछ वर्षा होती है। गर्मी के कारण स्थानीय रूप से धूल भरी आंधियां (Dust Storms) चलती हैं। इनके कारण उत्तर-पश्चिमी भारत में कुछ वर्षा हो जाती है जो गर्मी को कम करती है।

3. वर्षा ऋतु:
दक्षिण-पश्चिम मानसून के आगमन के साथ भारत में वर्षा ऋतु आरम्भ हो जाती है। तटीय क्षेत्रों में जून के प्रथम सप्ताह में मानसून आरम्भ हो जाती है तथा अचानक भारी वर्षा होती है। इसे ‘मानसून प्रस्फोट’ कहा जाता है। मानसून वर्षा के कारण इस ऋतु में तापमान 5° से 8° कम हो जाते हैं। औसत तापमान 25°C से 30°C तक रहते हैं। इस ऋतु में सागर से स्थल की ओर (Sea to Land) दक्षिण-पश्चिमी मानसून पवनें चलती हैं जो भारत की जलवायु में सबसे महत्त्वपूर्ण तत्त्व है। प्रायद्वीप के तिकोने आकार के कारण ये पवनें दो शाखाओं में बंट जाती हैं

  1. अरब सागर की मानसून शाखा।
  2. खाड़ी बंगाल की मानसून शाखा।

मानसून की खाड़ी बंगाल की शाखा अराकान पर्वत से टकरा कर उत्तर – पूर्वी भारत में भारी वर्षा करती है। ये पवनें उत्तरी मैदान में पंजाब तक वर्षा करती हैं। पश्चिम की ओर जाते हुए वर्षा की मात्रा कम होती है। मानसून की अरब सागरीय शाखा पश्चिमी तट, पश्चिमी घाट, मध्य प्रदेश में वर्षा करती है, परन्तु दक्षिणी पठार पश्चिमी घाट की वृष्टि छाया में रहने के कारण शुष्क रहता है।

4. लौटती मानसून की ऋतु:
इस ऋतु में दक्षिण-पश्चिमी मानसून उत्तरी भारत से लौटने लगती है । इस ऋतु में तापमान, वायु दाब तथा अन्य सभी दशाएं शनैः-शनैः बदलने लगती हैं। यह ऋतु वर्षा तथा शीत ऋतु में मध्य संक्रात काल होता है। दिन के समय तापमान कम होने लगता है। औसत तापमान 20°C से कम हो जाता है। पवनों की दिशा तथा गति अनिश्चित होती है। धीरे-धीरे प्रति चक्रवातीय दशाएं उत्पन्न होने लगती हैं। वर्षा कम होती है । पूर्वी तटीय प्रदेश पर कुछ वर्षा चक्रवातों द्वारा होती है।

मानचित्र कार्य (Map Skill)

प्रश्न-भारत के रेखा चित्र पर निम्नलिखित को दर्शाइए।

  1. शीतकालीन वर्षा के क्षेत्र
  2. ग्रीष्म ऋतु में पवनों की दिशा
  3. 50 प्रतिशत से अधिक वर्षा की परिवर्तिता के क्षेत्र
  4. जनवरी माह में 15°C से कम तापमान वाले क्षेत्र
  5. भारत में 100 सें० मी० की सम वर्षा रेखा।

JAC Class 11 Geography Solutions Chapter 4 जलवायु 1

जलवायु JAC Class 11 Geography Notes

→ भारत का जलवायु (Climate of India): भारत में उष्णकटिबन्धीय मानसून प्रकार का जलवायु है। कर्क रेखा भारत को दो समान भागों-उष्ण कटिबन्धीय तथा शीतोष्ण कटिबन्धीय में बांटती है।

→ जलवायु विविधता (Climatic Diversity): अधिक विस्तार के कारण जलवायु में बहुत विविधता है। बाड़मेर। में सबसे अधिक तापमान 50° C जबकि कारगिल (लद्दाख ) में सबसे कम तापमान 45°C होता है। मासिनराम (मेघालय) में सबसे अधिक वर्षा 1080 cm जबकि जैसलमेर (रेगिस्तान) में 15 cm से कम वर्षा होती है। भारत की जलवायु को निर्धारित करने वाले कारक

  • मानसूनी पवनें: भारत में मानसूनी जलवायु है । यह मानसून पवनों द्वारा नियन्त्रित होता है।
  • देश का विस्तार: शीतोष्ण प्रदेश में उत्तरी भाग स्थित है। यहां उष्ण ग्रीष्मकाल तथा ठण्डी शीतकाल ऋतु होती है।
  • हिमालय की स्थिति: हिमालय एक जलवायु विभाजक का कार्य करता है। यह पर्वतीय दीवार सर्दियों में भारत को मध्यवर्ती एशिया की शीत पवनों से बचाती है। हिमालय पर्वत हिन्द महासागर की दक्षिण-पश्चिमी मानसून पवनों को रोक कर वर्षा करने में सहायक है।
  • हिन्द महासागर: दक्षिण-पश्चिमी मानसून का उद्गम इस महासागर से होता है। ये पवनें देश के अधिकतर भागों पर वर्षा प्रदान करती हैं।
  • पश्चिमी विक्षोभ: पश्चिमी विक्षोभ (चक्रवात) सर्दियों में भारत के उत्तर-पश्चिमी भागों में वर्षा प्रदान करते हैं।
  • समुद्र से दूरी: तटीय भागों में समकारी (सागरीय) जलवायु है। इन क्षेत्रों में समकारी जलवायु होता है। परन्तु देश के भीतरी भागों में कठोर या महाद्वीपीय जलवायु मिलता है।
  • धरातल: सम्मुख ढलानों पर भारी वर्षा होती है परन्तु विमुख ढलान पर ( दक्कन पठार) कम वर्षा के कारण वृष्टि छाया रहती है।

→ मानसून (Monsoons): ‘मानसून’ शब्द वास्तव में अरबी भाषा के शब्द ‘मौसिम’ से बना है। मानसून शब्द का अर्थ है – मौसम के अनुसार पवनों के ढांचे में परिवर्तन होना। मानसून व्यवस्था के अनुसार मौसमी पवनें चलती हैं जिनकी दिशा मौसम के अनुसार विपरीत हो जाती है। ये पवनें ग्रीष्मकाल के 6 मास समुद्र से स्थल | की ओर तथा शीतकाल के 6 मास स्थल से समुद्र की ओर चलती हैं।

→ (क) गर्मियों में मानसून: मई तथा जून के महीने के समय ये भारत के उत्तरी मैदानों में बहुत गर्म होती हैं। इसके परिणामस्वरूप उत्तर पश्चिमी भारत में निम्न वायुदाब उत्पन्न होता है। पवनें हिन्द महासागर से भारत के उत्तरी मैदान की ओर चलना आरम्भ कर देती हैं। ये पवनें भारत में दो विभिन्न दिशाओं द्वारा प्रवेश करती हैं

  1. खाड़ी बंगाल की मानसून की शाखा: खाड़ी बंगाल मानसून उत्तर – पूर्वी भारत में भारी वर्षा करती है। ये पवनें हिमालय पर्वत के साथ-साथ पश्चिम की ओर वर्षा करती हैं।
  2. अरब सागर की मानसून शाखा: इन पवनों की एक शाखा पश्चिमी घाट पर भारी वर्षा करती है। दूसरी शाखा मध्यवर्ती भारत में प्रवेश करके वर्षा करती है। तीसरी शाखा राजस्थान में अरावली पर्वत के समानान्तर चलती है तथा अरावली पर्वत इन्हें रोक नहीं पाता इसलिए यहां बहुत कम वर्षा होती है।

(ख) शीतकालीन मानसून पवनें: उत्तर-पूर्व मानसून शीतकाल में स्थल से सागर की ओर चलती है तथा शुष्क होती है परन्तु खाड़ी बंगाल को पार करने के पश्चात् तमिलनाडु तट पर वर्षा करती है।

भारत में वर्षा की विशेषताएं (Characteristics of Rainfall in India):

  • मानसूनी वर्षा: भारत में अधिकतर वर्षा मध्य जून से सितम्बर तक दक्षिण पश्चिमी मानसून पवनों द्वारा होती है। यह वर्षा मौसमी वर्षा है।
  • अनिश्चित वर्षा: गर्मियों में वर्षा अनिश्चित होती है। समय से पहले या बाद में आने वाली मानसून पवनें अकाल तथा बाढ़ का कारण बनती हैं।
  • असमान वितरण: देश में वर्षा का वितरण बहुत असमान है।
  • भारी वर्षा: भारतीय वर्षा बौछारों के रूप में भारी वर्षा करती हैं।
  • उच्चावच से सम्बन्धित वर्षा: पर्वतों के सहारे अधिक वर्षा होती है।
  • वर्षा का निरन्तर न होना: गर्मियों में वर्षा में शुष्क Spells आते हैं।
  • वर्षा की परिवर्तनशीलता: वर्षा की परिवर्तनशीलता लगभग 30% है जिसके कारण अकाल पड़ जाते हैं।

JAC Class 11 Geography Solutions Chapter 4 जलवायु

→ भारत में वर्षा का वितरण (Distribution of Rainfall in India)

  1. भारी वर्षा वाले क्षेत्र: ये क्षेत्र 200 सेंटीमीटर से अधिक वार्षिक वर्षा प्राप्त करते हैं। इनमें पश्चिमी तट, पश्चिमी घाट, उप- हिमालय तथा उत्तर-पू -पूर्वी भाग शामिल हैं।
  2. दरमियानी वर्षा वाले क्षेत्र: ये क्षेत्र 100-200 सेंटीमीटर वार्षिक वर्षा प्राप्त करते हैं। इन क्षेत्रों में पश्चिमी बंगाल, उड़ीसा, बिहार, उत्तर प्रदेश के पूर्वी भाग तथा मध्य प्रदेश तथा तमिलनाडु के तटीय मैदान शामिल हैं।
  3. कम वर्षा वाले क्षेत्र: ये क्षेत्र 50-100 सेंटीमीटर तक वर्षा प्राप्त करते हैं। इनमें उत्तर प्रदेश के पूर्वी भाग, हरियाणा, पंजाब, प्रायद्वीपीय पठार तथा पूर्वी राजस्थान शामिल हैं।
  4. अति कम वर्षा वाले क्षेत्र: ये 50 सेंटीमीटर से कम वार्षिक वर्षा प्राप्त करते हैं। इनमें लद्दाख, दक्षिण-पश्चिमी पंजाब, दक्षिणी हरियाणा, पश्चिमी राजस्थान, कच्छ तथा थार मरुस्थल शामिल हैं।

→ भारत में मौसम (Seasons in India): मानसून पवनों के आगमन तथा पीछे हटने के क्रम से मौसमों में भी एक क्रम पाया जाता है।

→ मौसम (Seasons)
उत्तर-पूर्व मानसून की ऋतुएं

  • शीत ऋतु: दिसम्बर से फरवरी
  • ग्रीष्म ऋतु: मार्च से मई ।

(ख) दक्षिण-पश्चिम मानसून की ऋतुएं

  • वर्षा ऋतु: जून से सितम्बर।
  • लौटती मानसून पवनों की ऋतु: अक्तूबर से नवम्बर।

JAC Class 11 Geography Solutions Chapter 3 अपवाह तंत्र

Jharkhand Board JAC Class 11 Geography Solutions Chapter 3 अपवाह तंत्र Textbook Exercise Questions and Answers.

JAC Board Class 11 Geography Solutions Chapter 3 अपवाह तंत्र

बहु-विकल्पी प्रश्न (Multiple Choice Questions )

प्रश्न – दिए गए चार वैकल्पिक उत्तरों में से सही उत्तर चुनिए
1. किस नदी को ‘बंगाल का शोक’ कहा जाता था?
(A) गंडक
(B) कोसी
(C) सोन
(D) दामोदर।
उत्तर:
(D) दामोदर।

2. किस नदी का बेसिन भारत में सबसे बड़ा है?
(A) सिन्ध
(B) गंगा
(C) ब्रह्मपुत्र
(D) कृष्णा।
उत्तर:
(B) गंगा।

JAC Class 11 Geography Solutions Chapter 3 अपवाह तंत्र

3. कौन-सी नदी पंचनद में सम्मिलित नहीं है?
(A) रावी
(C) चनाब
उत्तर:
(B) सिन्ध |

4. कौन-सी नदी दरार घाटी में बहती है?
(A) सोन
(B) यमुना
(C) नर्मदा
(D) लूनी।
उत्तर:
(C) नर्मदा|

5. अलकनन्दा तथा भागीरथी नदी के संगम को क्या कहते हैं?
(A) विष्णु प्रयाग
(B) कर्णप्रयाग
(C) रुद्र प्रयाग
(D) देव प्रयाग।
उत्तर:

लघु उत्तरीय प्रश्न (Short Answer Type Questions)

प्रश्न 1.
नदी द्रोणी और जल संभर में अन्तर स्पष्ट करें।
उत्तर:
नदी द्रोणी (River Basin ): विशाल नदी एक विशिष्ट क्षेत्र से जल बहा कर अपने साथ लाती है तथा सागर में गिरती है। इसमें कई सहायक नदियां भी अपना जल शामिल करती हैं । इस क्षेत्र को नदी द्रोणी कहते हैं ।
जल संभर (Water- Shed): छोटी-छोटी नदियों द्वारा अपवाहित क्षेत्र को जल संभर कहते हैं । इसका आकार एक नदी द्रोणी से छोटा होता है ।

JAC Class 11 Geography Solutions Chapter 3 अपवाह तंत्र

प्रश्न 2.
वृक्षाकार और जालीनुमा अपवाह प्रारूप में अन्तर स्पष्ट करें।
उत्तर:
वृक्षाकार अपवाद (Dendritic Pattern): जो अपवाह प्रतिरूप (Drainage pattern) पेड़ की शाखाओं के अनुरूप हो उसे वृक्षाकार प्रतिरूप कहते हैं। उत्तरी मैदान की नदियां वृक्षाकार अपवाह का उदाहरण हैं । यह वृक्ष की आकृति जैसा है।
जालीनुमा अपवाह (Trellis Drainage ): जब मुख्य नदियां एक-दूसरे के समानान्तर बहती हों तथा सहायक नदियां उनसे समकोण पर मिलती हों तो इस प्रतिरूप को जालीनुमा अपवाह प्रतिरूप कहते हैं ।

प्रश्न 3.
अपकेन्द्रीय और अभिकेन्द्रीय अपवाह प्रारूप |
उत्तर:
अपकेन्द्रीय अपवाह (Radial Drainage ): जब नदियां किसी पर्वत से निकल कर सभी दिशाओं में बहती हैं तो इसे अपकेन्द्रीय अपवाद कहते हैं । अमरकंटक पर्वत श्रृंखला से निकलने वाली नदियां इस प्रतिरूप का उदाहरण हैं ।
अभिकेन्द्रीय अपवाह (Centripetal Drainage ): जब सभी दिशाओं से नदियां बहकर किसी झील या गर्त में मिल जाती हैं तो इसे अभिकेन्द्रीय अपवाद कहते हैं जैसे थार मरुस्थल में ।

प्रश्न 4.
डेल्टा तथा ज्वारनदमुख में अन्तर स्पष्ट करो ।
उत्तर:
डेल्टा (Delta): नदियां अपने उद्गम से लेकर अपने साथ तलछट सागर में गिरने से पहले निक्षेप करती हैं। यहां तक त्रिभुजाकार भू-भाग (∆) की रचना होती है जिसे डेल्टा कहते हैं। संसार में सबसे बड़ा डेल्टा गंगा ब्रह्मपुत्र डेल्टा है।
ज्वारनदमुख (Estuary): जब नदियां अपने अन्तिम भाग में तीव्र ढलान के कारण तलछट निक्षेप नहीं करतीं तो वह तलछट सागर में बह जाता है। इस तंग चैनल को ज्वारनदमुख कहते हैं। जैसे नर्मदा नदी डेल्टा की बजाय एक ज्वारनदमुख बनाती है।

JAC Class 11 Geography Solutions Chapter 3 अपवाह तंत्र

प्रश्न 5.
भारत में नदियों को आपस में जोड़ने के सामाजिक आर्थिक लाभ क्या हैं?
उत्तर:
उत्तरी भारत की नदियां प्रायद्वीपीय नदियों से भिन्न हैं। मध्य भारत में सतपुड़ा – विन्ध्याचल श्रेणी तट जलविभाजक का कार्य करती है। उत्तरी भारत की नदियों में जलाधिकय है तथा सारा वर्ष जल प्राप्त होता है । परन्तु प्रायद्वीपीय नदियां मौसमी हैं । उत्तरी भारत की नदियों को प्रायद्वीपीय भारत की नदियों से जोड़ा जा सकता है। इस योजना को गंगा – कावेरी योजना कहते हैं। उत्तरी भारत की अतिरिक्त जल वाली नदियां प्रायद्वीपीय भारत की कम जल वाली नदियों को जल प्रदान कर सकती हैं। इससे जल सिंचाई में विस्तार किया जा सकता है। खाद्यान्न उत्पादन में वृद्धि की जा सकती है। देश में सूखे की समस्या समाप्त की जा सकती है। देश में जलविद्युत् उत्पादन को बढ़ाया जा सकता है परन्तु विभिन्न राज्यों की सीमाओं तथा जल संसाधनों पर उनके अधिकार की समस्याएं हैं।

प्रश्न 6.
प्रायद्वीपीय नदियों के तीन लक्षण बताओ।
उत्तर:

  1. प्रायद्वीपीय नदियां अधिक लम्बी नहीं हैं तथा कम हैं।
  2. ये नदियां मौसमी हैं। केवल वर्षा ऋतु में ही इनसे जल प्राप्त होता है।
  3. प्रायद्वीपीय नदियां जहाजरानी तथा जलसिंचाई के अनुकूल नहीं

JAC Class 11 Geography Solutions Chapter 3 अपवाह तंत्र

प्रश्न 7.
हिमालय के गिरिपद के साथ-साथ हरिद्वार से सिलीगुड़ी तक की यात्रा में आने वाली मुख्य नदियों के नाम लिखो।
उत्तर:

  1. गंगा
  2. शारदा नदी
  3. गोमती नदी
  4. घाघरा नदी
  5. गंडक नदी
  6. कोसी नदी।

अपवाह तंत्र  JAC Class 11 Geography Notes

→ भारत का जल-प्रवाह (Drainage of India): भारत में कई विशाल नदियां हैं इसलिए इसे ‘नदियों की | धरती’ कहते हैं। भारत के स्थल भाग का 90% जल प्रवाह बंगाल की खाड़ी में गिरता है।

→ जल विभाजक (Water Sheds): विभिन्न जल-प्रवाह तन्त्रों को अलग-अलग करने वाले क्षेत्र को जल विभाजक कहते हैं। भारत में मुख्य तीन जल विभाजक हैं- हिमालय श्रेणी, विंध्याचल, सतपुड़ा तथा पश्चिमी
घाट।

→ भारत का जल प्रवाह तन्त्र (Drainage System of India ): भारतीय नदियों को दो तन्त्रों में बाँटा जा सकता है
(i) हिमालय नदियां
(ii) प्रायद्वीपीय नदियां।

→ हिमालयाई नदियां (The Himalayan Rivers ): हिमालय नदियों में विशाल बेसिन हैं। नदियां गहरे गार्ज | बनाती हैं। ये सदा वाहिनी नदियां हैं क्योंकि ये हिम नदियों से जन्म लेती हैं। इनमें तीन मुख्य नदियां सिन्धु, गंगा तथा ब्रह्मपुत्र नदियां हैं। कई नदियां पूर्ववर्ती हैं। ये हिमालय पर्वत के उत्थान से पहले बहती थीं।

→ प्रायद्वीपीय नदियां (The Peninsular Rivers ): ये नदियां कम गहरी घाटियों में बहती हैं जो आधार त को पहुंच चुकी हैं। कई नदियां मौसमी हैं जो वर्षा पर निर्भर करती हैं। इन्हें दो भागों में बांटा जा सकता है- I पूर्व में बहने वाली नदियां जो बंगाल की खाड़ी में गिरती (महानदी, गोदावरी, कृष्णा, कावेरी) हैं तथा पश्चिम में बहने वाली नदियां जो अरब सागर में गिरती हैं ( नर्मदा तथा ताप्ती) ।

→ मुख्य नदियां (Major Rivers ): सिन्धु नदी की कुल लम्बाई 2880 किलोमीटर है। इसका उद्गम तिब्बत में (मानसरोवर) होता है तथा यह ट्रांस हिमालयाई नदी है। इसकी पांच सहायक नदियां सतलुज, ब्यास, रावी, जेहलम तथा चिनाब हैं। गंगा भारत की मुख्य नदी है जो गंगोत्री से निकलती है। ब्रह्मपुत्र को तिब्बत में सांपो कहते हैं जो बांग्लादेश से होकर बंगाल की खाड़ी की ओर बहती है। गोदावरी प्रायद्वीपीय भारत की सबसे | लम्बी नदी है। इसे वृद्ध गंगा कहते हैं। महानदी, कृष्णा, कावेरी प्रायद्वीपीय भारत की मुख्य नदियां हैं।

JAC Class 11 Geography Solutions Chapter 10 वायुमंडलीय परिसंचरण तथा मौसम प्रणालियाँ

Jharkhand Board JAC Class 11 Geography Solutions Chapter 10 वायुमंडलीय परिसंचरण तथा मौसम प्रणालियाँ Textbook Exercise Questions and Answers.

JAC Board Class 11 Geography Solutions Chapter 10 वायुमंडलीय परिसंचरण तथा मौसम प्रणालियाँ

(क) बहु-विकल्पी प्रश्न (Multiple Choice Questions)

प्रश्न-दिए गए चार वैकल्पिक उत्तरों में से सही उत्तर चुनिए
1. धरातल पर वायुदाब 1000 मिलीबार है। धरातल से 1 कि० मी० की ऊंचाई पर वायुदाब क्या होगा?
(A) 700 मिलीबार
(B) 900 मिलीबार
(C) 1,100 मिलीबार
(D) 1,300 मिलीबार।
उत्तर:
(B) 900 मिलीबार।

2. अन्तःउष्ण कटिबन्धीय अभिसरण क्षेत्र प्रायः कहां होता है?
(A) भूमध्य रेखा के निकट
(B) कर्क रेखा के निकट
(C) मकर रेखा के निकट
(D) आर्कटिक वृत्त के निकट।
उत्तर:
(A) भूमध्य रेखा के निकट।

JAC Class 11 Geography Solutions Chapter 10 वायुमंडलीय परिसंचरण तथा मौसम प्रणालियाँ

3. उत्तरी गोलार्द्ध में निम्न वायुदाब के चारों तरफ पवनों की दिशा क्या होगी?
(A) घड़ी की सुइयों के चलने की दिशा के अनुरूप
(B) घड़ी की सुइयों के चलने की दिशा के विपरीत
(C) समदाब रेखाओं के समकोण पर
(D) समदाब रेखाओं के समानान्तर।
उत्तर:
(B) घड़ी की सुइयों के चलने की दिशा के विपरीत।

4. किस वातान के आने पर मेघों का क्रम पक्षाभ मेघ से स्तरी मेघ होता है?
(A) अधिविष्ट वाताग्र
(B) शीत वाताग्र
(C) उष्ण वाताग्र
(D) अचर वाताग्र।
उत्तर:
(C) उष्ण वाताग्र।

5. वायुराशियों के निर्माण के उद्गम क्षेत्र निम्नलिखित में से कौन-से हैं?
(A) भूमध्यरेखीय वन
(B) साइबेरिया का मैदानी भाग
(C) हिमालय पर्वत
(D) दक्खन पठार।
उत्तर:
(B) साइबेरिया का मैदानी भाग।

JAC Class 11 Geography Solutions Chapter 10 वायुमंडलीय परिसंचरण तथा मौसम प्रणालियाँ

6. समुद्र तल पर वायु दाब कितना पाया जाता है?
(A) 1010 मिलीबार
(B) 1011 मिलीबार
(C) 1013 मिलीबार
(D) 1012 मिलीबार।
उत्तर:
(C) 1013 मिलीबार।

लघु उत्तरीय प्रश्न (Short Answer Type Questions)

निम्नलिखित प्रश्नों के उत्तर लगभग 30 शब्दों में दीजिएप्रश्न
1. पवनों की दिशा व वेग को प्रभावित करने वाले कारक बताओ।
उत्तर:
पवनों की दिशा व वेग को प्रभावित करने वाले बल-वायुमण्डलीय दाब में भिन्नता के कारण वायु गतिमान होती है। इस क्षैतिज गतिज वायु को पवन कहते हैं। पवनें उच्च दाब से कम दाब की तरफ प्रवाहित होती हैं। भूतल पर धरातलीय विषमताओं के कारण घर्षण पैदा होता है, जो पवनों की गति को प्रभावित करता है। इसके साथ पृथ्वी का घूर्णन भी पवनों के वेग को प्रभावित करता है। पृथ्वी के घूर्णन द्वारा लगने वाले बल को कोरिऑलिस बल कहा जाता है। अतः पृथ्वी के धरातल पर क्षैतिज पवनें तीन संयुक्त प्रभावों का परिणाम हैं।

  1. दाब प्रवणता प्रभाव,
  2. घर्षण बल,
  3. तथा कोरिऑलिस बल।

इसके अतिरिक्त, गुरुत्वाकर्षण बल पवनों को नीचे प्रवाहित करता है।
1. दाब-प्रवणता बल:
वायुमण्डलीय दाब भिन्नता एक बल उत्पन्न करता है। दूरी के सन्दर्भ में दाब परिवर्तन की दर दाब प्रवणता है। जहाँ समदाब रेखाएँ पास-पास हों, वहाँ दाब प्रवणता अधिक व समदाब रेखाओं के दूर-दूर होने से दाब प्रवणता कम होती है।

2. घर्षण बल:
यह पवनों की गति को प्रभावित करता है। धरातल पर घर्षण सर्वाधिक होता है और इसका प्रभाव प्रायः धरातल से 1 से 3 कि०मी० ऊँचाई तक होता है। समुद्र सतह पर घर्षण न्यूनतम होता है।
3. कोरिऑलिस बल
पृथ्वी का अपने अक्ष पर घूर्णन पवनों की दिशा को प्रभावित करता है। सन् 1844 में फ्रांसिसी वैज्ञानिक ने इसका विवरण प्रस्तुत किया और इसी पर इस बल को कोरिऑलिस बल कहा जाता है। इसके प्रभाव से पवनें उत्तरी गोलार्द्ध में अपनी मूल दिशा से दाहिने तरफ व दक्षिण गोलार्द्ध में बाईं तरफ विक्षेपित (deflect) हो जाती हैं। जब पवनों का वेग अधिक होता है, तब विक्षेपण भी अधिक होता है। कोरिऑलिस बल अक्षांशों के कोण के सीधा समानुपात में बढ़ता है। यह ध्रुवों पर सर्वाधिक और भूमध्यरेखा पर अनुपस्थित है।

JAC Class 11 Geography Solutions Chapter 10 वायुमंडलीय परिसंचरण तथा मौसम प्रणालियाँ

प्रश्न 2.
उष्ण कटिबन्धीय अभिसरण क्षेत्र (ITCZ) से क्या अभिप्राय है? हेडले कोष्ठ तथा फैरल कोष्ठ में अन्तर स्पष्ट करो।
उत्तर:
ऊष्ण कटिबन्धीय अभिसरण क्षेत्र-उच्च सूर्यातप व निम्न वायुदाब होने से अन्तः- उष्ण कटिबन्धीय अभिसरण क्षेत्र (ITCZ) पर वायु संवहन धाराओं के रूप में ऊपर उठती है। उष्णकटिबन्धों से आने वाली पवनें इस निम्न दाब क्षेत्र में अभिसरण करती हैं। अभिसरित वायु संवहन कोष्ठों के साथ ऊपर उठती हैं। यह क्षोभमण्डल के ऊपर 14 किमी० की ऊँचाई तक ऊपर चढ़ती हैं और फिर ध्रुवों की तरफ प्रवाहित होती हैं। इसके परिणामस्वरूप लगभग 30° उत्तर व 30° दक्षिण अक्षांश पर वायु एकत्रित हो जाती है। इस एकत्रित वायु का अवतलन होता है और यह उपोष्ण उच्चदाब बनाता है।

अवतलन का एक कारण यह है कि जब वायु 30° उत्तरी व दक्षिणी अक्षांश पर पहुंचती है तो यह ठंडी हो जाती है। धरातल के निकट वायु का अपसरण होता है और यह भूमध्यरेखा की ओर पूर्वी पवनों के रूप में बहती है। भूमध्यरेखा के दोनों तरफ से प्रवाहित होने वाली पूर्वी पवनें अन्तर उष्ण कटिबन्धीय अभिसरण (ITCZ) पर मिलती हैं। पृथ्वी की सतह से ऊपर की दिशा में होने वाले परिसंचरण और इसके विपरीत दिशा में होने वाले परिसंचरण को कोष्ठ (Cell) कहते हैं। उष्णकटिबन्धीय क्षेत्र में ऐसे कोष्ठ को हेडले कोष्ठ (Hadley cell) कहा जाता है।

मध्य अक्षांशीय वायु परिसंचरण में ध्रुवों से प्रवाहित होती ठण्डी पवनों का अवतलन होता है और उपोष्ण उच्चदाब कटिबन्धीय क्षेत्रों से आती गर्म हवा ऊपर उठती है। धरातल पर ये पवनें पछुआ पवनों के नाम से जानी जाती हैं और यह कोष्ठ फैरल कोष्ठ के नाम से जाने जाते हैं। ध्रुवीय अक्षांशों पर ठण्डी सघन वायु का ध्रुवों पर अवतलन होता है और मध्य अक्षांशों की ओर ध्रुवीय पवनों के रूप में प्रवाहित होती हैं। इस कोष्ठ को ध्रुवीय कोष्ठ कहा जाता है। ये तीन कोष्ठ वायमण्डल के सामान्य परिसंचरण का प्रारूप निर्धारित करते हैं। तापीय ऊर्जा का निम्न अक्षांशों से उच्च अक्षांशों में स्थानान्तर सामान्य परिसंचरण को बनाये रखता है।

JAC Class 11 Geography Solutions Chapter 10 वायुमंडलीय परिसंचरण तथा मौसम प्रणालियाँ

प्रश्न 3.
भूविक्षेपी पवनें क्या हैं?
उत्तर-पृथ्वी की सतह से 2-3 कि०मी० की ऊँचाई पर ऊपरी वायुमण्डल में पवनें धरातलीय घर्षण के प्रभाव से मुक्त होती हैं और दाब प्रवणता तथा कोरिऑलिस बल | से नियन्त्रित होतो हैं । जब समदाब रेखाएँ सीधी हों और घर्षण का प्रभाव न हो, तो दाब प्रवणता बल कोरिऑलिस बल से सन्तुलित हो जाता है और फलस्वरूप पवनें समदाब रेखाओं के समानान्तर बहती हैं। ये पवनें भूविक्षेपी (Geostrophic) पवनों के नाम से जानी जाती हैं।
JAC Class 11 Geography Solutions Chapter 10 वायुमंडलीय परिसंचरण तथा मौसम प्रणालियाँ 1

निबन्धात्मक प्रश्न (Essay Type Questions)

निम्नलिखित प्रश्नों के उत्तर लगभग 150 शब्दों में दीजिए।
प्रश्न 1.
पवनों की दिशा व वेग को प्रभावित करने वाले कारक बताएं।
उत्तर:
पवनों की दिशा व वेग को प्रभावित करने वाले बल-वायुमण्डलीय दाब में भिन्नता के कारण वायु गतिमान होती है। इस क्षैतिज गतिज वायु को पवन कहते हैं। पवनें उच्च दाब से कम दाब की तरफ प्रवाहित होती हैं। भूतल पर धरातलीय विषमताओं के कारण घर्षण पैदा होता है, जो पवनों की गति को प्रभावित करता है। इसके साथ पृथ्वी का घूर्णन भी पवनों के वेग को प्रभावित करता है। पृथ्वी के घूर्णन द्वारा लगने वाले बल को कोरिऑलिस बल कहा जाता है। अतः पृथ्वी के धरातल पर क्षैतिज पवनें तीन संयुक्त प्रभावों का परिणाम हैं

  1. दाब प्रवणता प्रभाव,
  2. घर्षण बल,
  3. तथा कोरिऑलिस बल।

इसके अतिरिक्त, गुरुत्वाकर्षण बल पवनों को नीचे प्रवाहित करता है।
1. दाब-प्रवणता बल:
वायुमण्डलीय दाब भिन्नता एक बल उत्पन्न करता है। दूरी के सन्दर्भ में दाब परिवर्तन की दर दाब प्रवणता है। जहां समदाब रेखाएं पास-पास हों, वहां दाब प्रवणता अधिक व समदाब रेखाओं के दूर-दूर होने से दाब प्रवणता कम होती है।

2. घर्षण बल:
यह पवनों की गति को प्रभावित करता है। धरातल पर घर्षण सर्वाधिक होता है और इसका प्रभाव प्रायः धरातल से 1 से 3 कि०मी० ऊंचाई तक होता है। समुद्र सतह पर घर्षण न्यूनतम होता है।

3. कोरिऑलिस बल:
पृथ्वी का अपने अक्ष पर घूर्णन पवनों की दिशा को प्रभावित करता है। सन् 1844 में फ्रांसिसी वैज्ञानिक ने इसका विवरण प्रस्तुत किया और इसी पर इस बल को कोरिऑलिस बल कहा जाता है। इसके प्रभाव से पवनें उत्तरी गोलार्द्ध में अपनी मूल दिशा से दाहिने तरफ व दक्षिण गोलार्द्ध में बाईं तरफ विक्षेपित (deflect) हो जाती हैं। जब पवनों का वेग अधिक होता है, तब विक्षेपण भी अधिक होता है। कोरिऑलिस बल अक्षांशों के कोण के सीधा समानुपात में बढ़ता है। यह ध्रुवों पर सर्वाधिक और भूमध्य रेखा पर अनुपस्थित है।

JAC Class 11 Geography Solutions Chapter 10 वायुमंडलीय परिसंचरण तथा मौसम प्रणालियाँ

प्रश्न 2.
पृथ्वी पर वायु मण्डलीय सामान्य परिसंचरण का वर्णन करो। 30° उत्तर व दक्षिण अक्षांशों पर उपोष्ण कटिबन्धीय उच्च वायु दाब के कारण बताओ।।
उत्तर:
वायु दबाव की नक्षत्रीय बांट-पृथ्वी पर वायु दबाव की बांट सरल तथा लगातार नहीं है। कई कारणों से बाधाएं तथा विभिन्नताएं उत्पन्न हो जाती हैं। “धरातल पर पूर्व-पश्चिम दिशा में अक्षांशों के समानान्तर लगभग एक ही चौड़ाई के बराबर क्षेत्रों को वायु दबाव की पेटियां (Pressure Belts) कहा जाता है।” धरातल पर वायु दाब की कुल 7 पेटियां हैं। धरातल पर उच्च वायु दबाव (High Pressure) की चार पेटियां तीन निम्न वायु दबाव (Low Pressure) पेटियों को अलग-अलग करती हैं

1. भूमध्य रेखीय निम्न वायु दबाव पेटी (Equatorial Low Pressure Belt):
यह कम वायु दबाव पेटी भूमध्य रेखा के दोनों ओर 5° उत्तर तथा 5° दक्षिण अक्षांश तक फैली हुई है। इस खण्ड में सारा वर्ष अधिक गर्मी के कारण हवा गर्म होकर संवाहिक धाराओं (Convectional Currents) के रूप में ऊपर उठती रहती है। इस प्रकार वायु दबाव कम हो जाता है। एक शान्त मण्डल (Belt of Calms) स्थापित हो जाता है जिसे डोल ड्रमस् (Dol Drums) भी कहते हैं।

2. उपोष्ण कटिबन्धीय उच्च वायु दबाव पेटियां (Sub-tropical High Pressure Belts):
कर्क रेखा तथा मकर रेखा के निकट 30°_35° अक्षांशों के बीच दो उच्च वायु दबाव पेटियां पाई जाती हैं। इस प्रकार यहां भूमध्य रेखा से आने वाली पवनें नीचे उतरती हैं, इन नीचे उतरती हुई पवनों (Descending Winds) के कारण उच्च दबाव (High Pressure) स्थापित हो जाता है तथा एक शान्त मण्डल बन जाता है। इन्हें अश्व अक्षांश (Horse Latitudes) भी कहते हैं।
JAC Class 11 Geography Solutions Chapter 10 वायुमंडलीय परिसंचरण तथा मौसम प्रणालियाँ 2

3. उप-ध्रुवीय कम वायु दबाव पेटियां (Sub-polar Low Pressure Belts):
यह कम दबाव की पेटियां 60° से 65° उत्तरी तथा दक्षिणी अक्षांशों के बीच स्थित हैं। उत्तरी गोलार्द्ध में इन्हें Arctic Low तथा दक्षिणी गोलार्द्ध में इन्हें Antarctic Low कहते हैं। स्थिति के कारण इन्हें शीतोष्ण निम्न वायु दबाव (Temperate Low) भी कहते हैं। इस कम वायु दबाव के कई कारण हैं

(i) दैनिक गति (Rotation): पृथ्वी की दैनिक गति के कारण ध्रुवों को वायु भूमध्य रेखा की ओर खिसक जाती है। ध्रुवों पर अधिक सर्दी के कारण कम दबाव उच्च दबाव में बदल जाता है तथा ध्रुवों से कुछ दूरी पर 60° के निकट कम वायु दबाव हो जाता है।

(ii) चक्रवात (Cyclones): ध्रुवों से आने वाली वायु पृथ्वी के तल के साथ रगड़ खा कर बहुत बड़े चक्रवातों का रूप धारण कर लेती है। यह चक्रवात भी कम वायु भार के केन्द्र (Low pressure cells) बन जाते हैं।

(iii) समुद्री धाराएं (Ocean Currents): गर्म समुद्री धाराओं के कारण कम वायु दाब होता है।

(iv) वायुमण्डल की मोटाई भी बहुत कम होती है।

4. ध्रुवीय उच्च दबाव पेटियां (Polar High Pressure Belts): उत्तरी ध्रुव तथा दक्षिणी ध्रुव पर लगातार अधिक सर्दी तथा स्थायी बर्फ के कारण उच्च वायु भार होता है। दक्षिणी ध्रुव पर Antarctic महाद्वीप के कारण अधिक दबाव होता है, परन्तु उत्तरी ध्रुव पर Arctic महासागर होने के कारण इतना अधिक दबाव नहीं होता।

JAC Class 11 Geography Solutions Chapter 10 वायुमंडलीय परिसंचरण तथा मौसम प्रणालियाँ

प्रश्न 3.
उष्ण कटिबन्धीय चक्रवातों की उत्पत्ति केवल समुद्रों पर ही क्यों होती है? उष्ण कटिबन्धीय चक्रवातों के किस भाग में मूसलाधार वर्षा तथा उच्च वेग की पवनें चलती हैं?
उत्तर:
उष्ण कटिबन्धीय चक्रवात (Tropical Cyclones)
1. स्थिति (Location):
ये चक्रवात भूमध्य रेखा के दोनों ओर उष्ण कटिबन्ध में चलते हैं इसलिए इन्हें उष्ण कटिबन्धीय चक्रवात कहा जाता है। इनका क्षेत्र व्यापारिक पवनों की पेटी में 5°—20° उत्तरी और दक्षिणी अक्षांशों के मध्य होता है।

2. उत्पत्ति (Origin):
उष्ण कटिबन्धीय चक्रवातों का विकास तापीय (Thermal) है।

  • इनकी निम्नलिखित मुख्य विशेषताओं से इनकी उत्पत्ति का पता चलता है
    1. ये ग्रीष्म काल में पाए जाते हैं।
    2. ये पूर्व से पश्चिम दिशा में चलते हैं।
    3. इनका क्षेत्र 5°- 20° अक्षांश तक है।
    4. इनका विस्तार सामान्य महासागरों पर होता है।
  • उपर्युक्त विशेषताओं के आधार पर इनकी उत्पत्ति अग्रलिखित अवस्थाओं में होती है
    1. शान्त वायु (Calm Air)
    2. संतृप्त वायु मण्डल (Saturated Atmosphere)
    3. उच्च ताप (Abundant Heat)
    4. संवहनीय धाराएं (Convection Currents)

कुछ विद्वानों के अनुसार दो विभिन्न प्रकार की वायु राशियों के मिलने से एक गर्त (Depression) का निर्माण होता है। ये वायु राशियां एक-दूसरे से भिन्न होती हैं। ये चक्रवात भूमध्य रेखीय अभिसरण क्षेत्र में उत्पन्न होते हैं। वायु मण्डलीय आर्द्रता की गुप्त ऊष्मा (Latent Heat) इनके विकास में सहायक होती हैं। स्थानीय संवहनीय धाराओं के कारण इनका विकास होता है। अभिसरण क्षेत्र के उत्तर-दक्षिण खिसकने से पूर्वी पवनों की व्यवस्था एक चक्रीय रूप धारण करके उष्ण कटिबन्धीय चक्रवात बन जाती है। पृथ्वी की परिभ्रमण (Rotation) के कारण वायु समूह बिखर जाते हैं तथा चक्रीय अवस्था में गोलाकार हो जाते हैं।
JAC Class 11 Geography Solutions Chapter 10 वायुमंडलीय परिसंचरण तथा मौसम प्रणालियाँ 3

3. आकार तथा विस्तार (Shape and Size): ये चक्रवात आकार एवं विस्तार में छोटे होते हैं। इनका व्यास लगभग 100 कि०मी० होता है। ये अण्डाकार (ellipitcal) होते हैं। पूर्णतया विकसित चक्रवात का व्यास 1000 कि०मी० तक होता है।

4. गति (Speed): इन चक्रवातों की गति सागरों पर अधिक होती है। स्थल भागों पर प्राकृतिक बाधाओं तथा घर्षण के कारण इनकी गति कम होती है। कहीं-कहीं इनकी गति 150 कि०मी० प्रति घण्टा होती है। यह चक्रवात महासागरों तथा तटों का बहुत प्रलयकारी होते हैं।

5. मार्ग तथा प्रवाह (Track and Movement): ये चक्रवात व्यापारिक पवनों के साथ-साथ पूर्व पश्चिम दिशा में चलते हैं। ये चक्रवात नमी के समाप्त हो जाने पर स्थल के भीतर नहीं पहुंच पाते वरन् तट के निकट ही समाप्त हो जाते हैं। इन चक्रवातों का मार्ग एक वक्राकार होता है। यह मार्ग इस प्रकार है

  1. यह चक्रवात 5° से 15° अक्षांश के मध्य व्यापारिक पवनों के साथ पश्चिम की ओर चलते हैं।
  2. 15° से 30° के मध्य इनका पथ अनिश्चित होता है।
  3. 30 अक्षांश पार करने का इनका पथ पूर्व की ओर होता है।

6. रचना तथा मौसम (Structure and Weather):
इन चक्रवातों के केन्द्र में निम्न वायु दबाव होता है। इस केन्द्र को आंधी की आंख (Eye of Cyclone) कहते हैं। इसकी समभार रेखाएं अण्डाकार होती हैं। दाब प्रवणता तीव्र होने के कारण तेज़ गति से पवनें चलती हैं। इन चक्रवातों के साथ कोई प्रति चक्रवात नहीं होता। केन्द्रीय भाग में संतृप्त मूसलाधार वर्षा करती है। चक्रवात के विभिन्न भागों में मौसम इस प्रकार होता है।

  1. चक्रवात के आगम से पूर्व वायु अशांत हो जाती है तथा पक्षाभ (Cirrus Clouds) छाए रहते हैं।
  2. सूर्य व चन्द्रमा के चारों ओर प्रवाह मण्डल (Halo) उत्पन्न हो जाता है।
  3. चक्रवात के निकट आने पर तीव्र आंधी व गर्ज के साथ भारी वर्षा होती है।

वायुमंडलीय परिसंचरण तथा मौसम प्रणालियाँ JAC Class 11 Geography Notes

→ वायुमण्डलीय दाब (Atmospheric Pressure): वायुमण्डल पृथ्वी के धरातल पर पृथ्वी की गुरुत्वाकर्षण शक्ति के कारण टिका है। गुरुत्वाकर्षण शक्ति के कारण प्रत्येक वस्तु में भार होता है। वायु में भी एक घनफुट में 1.2 औंस भार होता है। इस भार के कारण पृथ्वी के धरातल पर दबाव पड़ता है। वायुमण्डलीय दाब का अर्थ है किसी भी स्थान पर वहां की हवा की उच्चतम सीमा के स्तम्भ का भार।

→ वायुमण्डलीय दाब को प्रभावित करने वाले कारक (Factors affecting Pressure): वायुमण्डलीय दाब निम्नलिखित तत्त्वों के कारण स्थान-स्थान पर तथा समय-समय पर बदलता रहता है। वे तत्त्व हैं

  • तापमान
  • ऊंचाई
  • जल वाष्प
  • दैनिक गति।

→ वायु दाब का मापक (Measurement of Pressure): वायुदाब को मापने वाले यन्त्र को बैरोमीटर कहते हैं।

→ मापने की इकाइयां (Units of Measurement): वायुमण्डलीय दाब को मापने के लिये तीन इकाइयों का प्रयोग होता है

  • इंच
  • सेंटीमीटर
  • मिलीबार।

→ समदाब रेखाएं (Isobars): Iso शब्द का अर्थ है-सभान और Bar का अर्थ है-दाब। इसलिये Isobars का अर्थ है-समदाब रेखाएं। धरातल पर समान वायु दबाव वाले स्थानों को मिलाने वाली रेखाओं को समदाब | रेखाएं कहते हैं।

→ दाब प्रवणता (Pressure Gradient): समदाब रेखाओं के अन्तर को दाब प्रवणता कहते हैं। यह दो समदाब रेखाओं पर समकोण बनाती हुई होती है। यह दाब प्रवणता वायु दिशा तथा वायु वेग को प्रदर्शित । करती है। वायुदाब

→ पेटियों का वितरण (Distribution of Pressure Belts): “धरातल पर पूर्व-पश्चिम दिशा में । अक्षांशों के समानान्तर लगभग एक ही चौड़ाई के बराबर क्षेत्रों को वायु दबाव की पेटियां कहा जाता है।” पृथ्वी के धरातल पर कुल सात वायु दबाव पेटियां हैं। चार उच्च दबाव की तथा तीन निम्न दबाव पेटियां हैं

  • भूमध्य रेखीय निम्न वायु दाब पेटी-5° उत्तर तथा 5° दक्षिण अक्षांश के मध्य।
  • उपोष्ण कटिबन्धीय उच्च वायु दाब पेटी-30°- 35° अक्षांशों के मध्य।
  • उप-ध्रुवीय निम्न वायुदाब पेटियां- 60° – 65° उत्तरी तथा दक्षिणी अक्षांशों के मध्य।
  • ध्रुवीय उच्च दाब पेटियां-उत्तरी ध्रुव तथा दक्षिणी ध्रुव के आस-पास।

JAC Class 11 Geography Solutions Chapter 9 सौर विकिरण, ऊष्मा संतुलन एवं तापमान

Jharkhand Board JAC Class 11 Geography Solutions Chapter 9 सौर विकिरण, ऊष्मा संतुलन एवं तापमान Textbook Exercise Questions and Answers.

JAC Board Class 11 Geography Solutions Chapter 9 सौर विकिरण, ऊष्मा संतुलन एवं तापमान

बहु-विकल्पी प्रश्न (Multiple Choice Questions)

प्रश्न-दिए गए चार वैकल्पिक उत्तरों में से सही उत्तर चुनिए
1. निम्नलिखित में से कौन-सी गैस वायुमण्डल में सबसे अधिक मात्रा में मौजूद है?
(A) ऑक्सीजन
(B) आर्गन
(C) नाइट्रोजन
(D) कार्बन डाइऑक्साइड।
उत्तर:
नाइट्रोजन।

2. 21 जून की दोपहर सूर्य लम्बवत् होता है
(A) विषुवत् रेखा पर
(B) 23.5°N
(C) 66.5°
(D) 66.5°N
उत्तर:
(B) 23.5°N.

JAC Class 11 Geography Solutions Chapter 9 सौर विकिरण, ऊष्मा संतुलन एवं तापमान

3. निम्न में से किन शहरों में दिन ज्यादा लम्बा होता है?
(A) त्रिवेन्द्रम
(B) हैदराबाद
(C) चण्डीगढ़
(D) नागपुर।
उत्तर:
(C) चण्डीगढ़।

4. वायुमण्डल मुख्यतः गर्म होता है
(A) लघु तरंगदैर्ध्य वाले सौर विकिरण से
(B) लम्बी तरंगदैर्ध्य वाले स्थलीय विकिरण से
(C) परावर्तित सौर विकिरण से
(D) प्रकीर्णित सौर विकिरण से।
उत्तर:
(B) लम्बी तंरगदैर्ध्य वाले स्थलीय विकिरण से।

5. पृथ्वी के विषुवत् रेखीय क्षेत्रों की अपेक्षा उत्तरी गोलार्द्ध के उपोष्ण कटिबन्धीय क्षेत्रों का तापमान अधिकतम होता है, इसका मुख्य कारण है
(A) विषुवत् रेखीय क्षेत्रों की अपेक्षा उपोष्ण कटिबन्धीय क्षेत्रों में कम बादल होते हैं।
(B) उपोष्ण कटिबन्धीय क्षेत्रों में गर्मी के दिनों की लम्बाई विषुवतीय क्षेत्रों से ज़्यादा होती है।
(C) उपोष्ण कटिबन्धीय क्षेत्रों में ‘ग्रीन हाऊस प्रभाव’ विषुवतीय क्षेत्रों की अपेक्षा ज्यादा होता है।
(D) उपोष्ण कटिबन्धीय क्षेत्र विषुवतीय क्षेत्रों की अपेक्षा महासागरीय क्षेत्र के ज्यादा करीब है।
उत्तर:
(A) विषुवत् रेखीय क्षेत्रों की अपेक्षा उपोष्ण कटिबन्धीय क्षेत्रों में कम बादल होते हैं।

लघु उत्तरीय प्रश्न (Short Answer Type Questions)

निम्नलिखित प्रश्नों के उत्तर लगभग 30 शब्दों में दीजिए।
प्रश्न 1.
पृथ्वी पर तापमान का असमान वितरण किस प्रकार जलवायु और मौसम को प्रभावित करता है?
उत्तर:
तापमान का असमान वितरण पाया जाता है। सूर्यातप की भिन्नता के परिणामस्वरूप पृथ्वी पर जलवायु तथा मौसम में भिन्नता पाई जाती है। उष्ण कटिबन्ध की अपेक्षा विषुवत् रेखा पर कम मात्रा में तापमान पाया जाता है।

JAC Class 11 Geography Solutions Chapter 9 सौर विकिरण, ऊष्मा संतुलन एवं तापमान

प्रश्न 2.
वे कौन-से कारक हैं जो पृथ्वी पर तापमान के वितरण को प्रभावित करते हैं?
उत्तर:
किसी भी स्थान पर वायु का तापमान निम्नलिखित कारकों द्वारा प्रभावित होता है

  1. उस स्थान का अक्षांश
  2. समुद्र तल से ऊंचाई
  3. समुद्र से दूरी
  4. वायुसंहति का परिसंचरण
  5. कोष्ण तथा ठण्डी महासागरीय धाराएं
  6. स्थानीय कारक।

प्रश्न 3.
भारत में मई में तापमान सर्वाधिक होता है लेकिन अत्यन्त ग्रीष्म के बाद तापमान अधिकतम नहीं होता। क्यों?
उत्तर:
भारत में मई मास सबसे अधिक गर्म होता है। सूर्य की किरणें कर्क रेखा पर लम्ब पड़ती हैं जो कि भारत को दो भागों में बांटती हैं। इसके पश्चात् वर्षा ऋतु होने के कारण तापमान का औसत रूप 6’7°C गिर जाता है।

JAC Class 11 Geography Solutions Chapter 9 सौर विकिरण, ऊष्मा संतुलन एवं तापमान

प्रश्न 4.
साइबेरिया के मैदान में वार्षिक तापान्तर सर्वाधिक होता है। क्यों?
उत्तर:
साइबेरिया की स्थिति आर्कटिक वृत्त के उत्तर में है, यहां सर्वाधिक तापान्तर महाद्वीपीयता (Continentality) के कारण है। यहां ग्रीष्म ऋतु में 10°C तापमान होता है जबकि शीत ऋतु में-18°C से – 48°C तक तापमान गिर जाता है। इस प्रकार यहां वार्षिक तापान्तर 60°C रहता है।

निबन्धात्मक प्रश्न (Essay Type Questions)

निम्नलिखित प्रश्नों के उत्तर लगभग 150 शब्दों में दीजिए।
प्रश्न 1.
अक्षांश और पृथ्वी के अक्ष का झुकाव किस प्रकार पृथ्वी की सतह पर प्राप्त होने वाली विकिरण की मात्रा को प्रभावित करते हैं?
उत्तर:
धरातल पर प्राप्त होने वाले सौर विकिरण को सूर्यातप कहते हैं। सूर्यातप की मात्रा सूर्य की किरणों के आपतन कोण पर निर्भर करती है।

  1. लम्ब किरणें तिरछी किरणों की अपेक्षा कम स्थान घेरती हैं। इस प्रकार प्रति इकाई क्षेत्र प्राप्त ताप अधिक होता है।
  2. लम्ब किरणों को तिरछी किरणों की अपेक्षा वायुमण्डल का थोड़ा भाग पार करना पड़ता है। इसलिए वायुमण्डल में मिली गैसें, जलवाष्प द्वारा अवशोषण, परावर्तन तथा बिखराव से सूर्यातप की मात्रा कम नष्ट होती है।

भूमध्य रेखा से ध्रुवों की ओर जाते हुए तापमान लगातार कम होता जाता है। किसी भी अक्षांश पर तापमान सूर्य की किरणों के कोण पर निर्भर है। लम्ब किरणें तिरछी किरणों की अपेक्षा थोड़े स्थान को घेरती हैं। अतः प्रति इकाई क्षेत्र प्राप्त ऊष्मा अधिक होती है। तिरछी किरणें वायुमण्डल में अधिक दूरी तय करती हैं तथा इनकी बहुत-सी गर्मी जलवाष्प अथवा धूलकणों द्वारा सोख ली जाती है। भूमध्य रेखा पर सारा वर्ष सूर्य की किरणें लम्बवत् पड़ती हैं तथा इन प्रदेशों में उच्च तापमान पाए जाते हैं। ध्रुवों की ओर तिरछी किरणों के कारण कम तापमान पाए जाते हैं।

JAC Class 11 Geography Solutions Chapter 9 सौर विकिरण, ऊष्मा संतुलन एवं तापमान

प्रश्न 2.
पृथ्वी और वायुमण्डल किस प्रकार ताप को सन्तुलित करते हैं? इसकी व्याख्या करें।
उत्तर:
पृथ्वी तथा वायुमण्डल ताप का सन्तुलन रखते हैं जिसे ताप बजट कहते हैं।
ताप बजट (Heat Budget):
ताप बजट से अभिप्राय ताप सन्तुलन से भी है। पृथ्वी पर जितनी मात्रा में सूर्य विकिरण से ताप प्राप्त होता है, उतनी ही मात्रा में ताप स्थलीय विकिरण द्वारा अन्तरिक्ष में वापस चला जाता है। इस प्रकार पृथ्वी तथा वायुमण्डल के ताप में एक सन्तुलन स्थापित हो जाता है।

मान लो कि वायुमण्डल की ऊपरी सतह पर प्राप्त होने वाला ताप 100 इकाई है। इसमें से केवल 51 इकाई ताप ही पृथ्वी पर पहुंचता है। इसमें से 49 इकाई ताप वायुमण्डल तथा अन्तरिक्ष में लौट जाता है। शून्य में परावर्तन हुई इस ऊष्मा को एल्बेडो (Albedo of the earth) कहते हैं।
JAC Class 11 Geography Solutions Chapter 9 सौर विकिरण, ऊष्मा संतुलन एवं तापमान 1
इस प्रकार सूर्य से प्राप्त 51 इकाई ताप पुनः वायुमण्डल को गर्म करने में लग जाता है। इसे पृथ्वी का ताप सन्तुलन या बजट कहते हैं।
JAC Class 11 Geography Solutions Chapter 9 सौर विकिरण, ऊष्मा संतुलन एवं तापमान 2

प्रश्न 3.
जनवरी में पृथ्वी के उत्तरी और दक्षिणी गोलार्द्ध के बीच तापमान के विश्व व्यापी वितरण की तुलना करें।
उत्तर:
तापमान का विश्व वितरण-(जुलाई तथा जनवरी)-तापमान का वितरण मानचित्रों पर समताप रेखाओं द्वारा दिखाया जाता है। यह वितरण वार्षिक होता है। ग्रीष्म ऋतु का तापमान जुलाई तथा शीत ऋतु का तापमान जनवरी के महीनों के मानचित्रों द्वारा प्रकट किया जाता है।

1. जनवरी में तापमान वितरण के लक्षण:
जनवरी में अधिकतम तापमान महाद्वीपों पर पाया जाता है। दक्षिण अमेरिका, (अफ्रीका) तथा ऑस्ट्रेलिया के स्थल खण्डों पर तापमान 30°C से अधिक होता है। उच्चतम ताप मकर रेखा के साथ-साथ पाया जाता है। जनवरी मास में न्यूनतम तापमान उत्तर पूर्वी एशिया में पाया जाता है। साइबेरिया में वोयांस्क में-50°C तक निम्नतम तापमान पाया गया है। उत्तरी गोलार्द्ध में समताप रेखाएं महासागरों पर ध्रुवों की ओर तथा महाद्वीपों पर भूमध्य रेखा की ओर झुकी होती हैं।
JAC Class 11 Geography Solutions Chapter 9 सौर विकिरण, ऊष्मा संतुलन एवं तापमान 3
दक्षिणी गोलार्द्ध में समताप रेखाएं महासागरों पर भूमध्य रेखा की ओर तथा महाद्वीपों पर ध्रुवों की ओर झुकी होती हैं। इस प्रकार समताप रेखाएं सूर्य की स्थिति के साथ-साथ खिसक जाती हैं। दक्षिणी गोलार्द्ध में समताप रेखाएं नियमित तथा पूर्व-पश्चिम दिशा में अक्षांशों के समानान्तर पाई जाती हैं क्योंकि यहां जल की अधिकता है, परन्तु उत्तरी गोलार्द्ध में ये रेखाएं अनियमित होती हैं।

2. जुलाई की समताप रेखाएं (July Iso-therms):
जुलाई उत्तरी गोलार्द्ध का अत्यन्त उष्ण तथा दक्षिणी गोलार्द्ध का शीतल मास होता है। इस समय सूर्य उत्तरी गोलार्द्ध में कर्क रेखा पर लगभग लम्बवत पडता है, जिससे वहां ग्रीष्म ऋतु तथा दक्षिणी गोलार्द्ध में शीत ऋतु होती है। इस समय उत्तरी गोलार्द्ध के एक अत्यधिक विस्तृत भाग में 30° सेल्सियस से भी अधिक तापमान रहता है।
JAC Class 11 Geography Solutions Chapter 9 सौर विकिरण, ऊष्मा संतुलन एवं तापमान 3

उत्तरी अमेरिका, अफ्रीका तथा एशिया के मरुस्थलों एवं अर्द्धशुष्क भूमियों में भी भीषण गर्मी पड़ती है। पाकिस्तान में जेकोबाबाद (Jacobabad) तथा लीबिया में ऐल अजीजिया (EL Azizia) संसार के सबसे अधिक गर्म स्थान हैं। ऐल अज़ीज़िया में तो तापमान 58° सेल्सियस तक इंगित किया गया है। निम्नतम तापमान भी ग्रीनलैण्ड के मध्य भाग में मिलता है।

सौर विकिरण, ऊष्मा संतुलन एवं तापमान JAC Class 11 Geography Notes

→ सूर्यातप-सूर्य वायुमण्डल के ताप का महत्त्वपूर्ण स्रोत है। इस प्रकार धरातल पर प्राप्त होने वाले सौर्य विकिरण को सूर्यातप कहते हैं। सूर्यातप तीन शब्दों के जोड़ से बना है।

→ सूर्यातप (Insolation) = In + sol + ation
In = Incoming
Sol = Solar
ation = Radiation

→ तापमान (Temperature): तापमान किसी पदार्थ में ताप की मात्रा का सू चक है। किसी स्थान पर छाया में वायु की मापी हुई गर्मी को तापमान कहते हैं।

→ तापमान का माप (Measurement of Temperature): स्क्सि के उच्चतम तथा न्यूनतम थर्मामीटर द्वारा तापमान मापा जाता है। ताप मापने की दो इकाइयां हैं
(i) सेंटीग्रेड मापक
(ii) फॉरनहाइट।

→ दैनिक तापान्तर (Daily Range of Temperature): किसी स्थान पर उस दिन के उच्चतम तथा न्यूनतम तापमान के अन्तर को उस स्थान का दैनिक तापान्तर कहते हैं।

→ वार्षिक तापान्तर (Annual Range of Temperature): किसी वर्ष के सबसे गर्म तथा सबसे ठण्डे महीनों के औसत मासिक तापमान के अन्तर को वार्षिक तापान्तर कहते हैं।

→ समताप रेखाएं (Isotherms): यह दो शब्दों के योग से बना है। Iso शब्द का अर्थ है-समान और therm शब्द का अर्थ है-तापमान। इसलिए समताप रेखाएं का अर्थ है-धरातल पर समान तापमान वाले स्थानों को जोड़ने वाली रेखाएं।

→ ताप कटिबन्ध (Temperature Zone): यूनानी विद्वानों ने तापमान तथा अक्षांशों के आधार पर पृथ्वी को तीन ताप कटिबन्धों में बांटा है

  • उष्ण कटिबन्ध (Torrid Zone): \(23 \frac{1}{2}\)° उत्तर से दक्षिण तक।
  • शीतोष्ण कटिबन्ध (Temperature Zone): दोनों गोलार्द्ध में \(23 \frac{1}{2}^{\circ} \) तथा \(66 \frac{1}{2}^{\circ}\) अक्षांश के मध्य।
  • शीत कटिबन्ध (Frigid Zone): ध्रुवों तथा के मध्य अक्षांश।

→ वायुमण्डल कैसे गर्म होता है? (Heating of Atmosphere): वायुमण्डल को गर्म करने में निम्नलिखित प्रक्रियाएं मुख्य भूमिका निभाती हैं

  • संचालन
  • संवहन क्रिया
  • विकिरण।

JAC Class 11 Geography Solutions Chapter 9 सौर विकिरण, ऊष्मा संतुलन एवं तापमान

→ वायुमण्डल का ग्रीन हाऊस प्रभाव (Green House Effect of Atmosphere): वायुमण्डल भूतल द्वारा सौर विकिरण से गर्म होता है। इसकी तुलना एक शीशे के घर या ग्रीन हाऊस से की जाती है जिसमें ध्रुवीय प्रदेशों में फूल आदि उगाए जाते हैं। इन शीशे के घरों में बाहर से सौर विकिरण अन्दर प्रवेश कर
सकता है परन्तु यह ताप विकिरण बाहर नहीं जा सकता।

→ विश्व के तापमान में वृद्धि (Global warming):  ईंधनों के जलने से (कोयला, गैस, पेट्रोलियम), भूमि की कृषि, औद्योगीकरण, तीव्र गति वाले परिवहन के साधन तथा वनों की कटाई से पिछले 100 वर्षों में पृथ्वी का औसत तापमान 0.5°C बढ़ गया है।

→ तापमान प्रतिलोम (Inversion of Temperature): ऊँचाई के बढ़ने के साथ-साथ 1°C प्रति 165 मीटर की दर से तापमान कम होता जाता है, परन्तु कई बार अस्थाई रूप से ऊंचाई के साथ-साथ तापमान । में वृद्धि होती है। ऐसी स्थिति में जब ठण्डी वायु धरातल के निकट और गर्म वायु इसके ऊपर हो तो इसे तापमान प्रतिलोम कहते हैं।

→ तापमान को प्रभावित करने वाले कारक (Factors affecting Temperature):  स्थान-स्थान पर तथा विभिन्न अक्षांशों पर तापमान विभिन्नता निम्नलिखित कारकों पर निर्भर करती है

  • अक्षांश
  • ऊंचाई
  • समुद्र से दूरी
  • प्रचलित पवनें
  • समुद्री धाराएं
  • पर्वतों की दिशा
  • भूमि की ढलान
  • मेघ तथा वर्षा
  • वनस्पति तथा मिट्टियां।

JAC Class 11 Geography Solutions Chapter 8 वायुमंडल का संघटन तथा संरचना

Jharkhand Board JAC Class 11 Geography Solutions Chapter 8 वायुमंडल का संघटन तथा संरचना Textbook Exercise Questions and Answers.

JAC Board Class 11 Geography Solutions Chapter 8 वायुमंडल का संघटन तथा संरचना

बहु-विकल्पी प्रश्न (Multiple Choice Questions)

प्रश्न-दिए गए चार वैकल्पिक उत्तरों में से सही उत्तर चुनिए
1. निम्नलिखित में से कौन-सी गैस वायुमण्डल में सबसे अधिक मात्रा में मौजूद है?
(A) ऑक्सीजन
(B) आर्गन
(C) नाइट्रोजन
(D) कार्बन डाइऑक्साइड।
उत्तर:
नाइट्रोजन।

2. वह वायुमण्डलीय परत जो मानव जीवन के लिए महत्त्वपूर्ण है
(A) समतापमण्डल
(B) क्षोभमण्डल
(C) मध्यमण्डल
(D) आयनमण्डल।
उत्तर:
क्षोभमण्डल।

3. समुद्री नमक, पराग, राख, धुएं की कालिमा, महीन मिट्टी-ये किससे सम्बन्धित हैं?
(A) गैस
(B) जलवाष्प
(C) धूलकण
(D) उल्कापात।
उत्तर:
धूलकण।

JAC Class 11 Geography Solutions Chapter 8 वायुमंडल का संघटन तथा संरचना

4. निम्नलिखित में से कितनी ऊँचाई पर ऑक्सीजन की मात्रा नगनय हो जाती है?
(A) 90 कि० मी०
(B) 100 कि० मी०
(C) 120 कि० मी०
(D) 150 कि० मी०।
उत्तर:
(C) 120 कि० मी०।

5. निम्नलिखित में से कौन-सी गैस सौर विकिरण के लिए पारदर्शी है तथा पार्थिव विकिरण के लिए अपारदर्शी है?
(A) ऑक्सीजन
(B) नाइट्रोजन
(C) हीलियम
(D) कार्बन डाइऑक्साइड।
उत्तर:
(D) कार्बन डाइऑक्साइड।

लघु उत्तरीय प्रश्न (Short Answer Type Questions)

प्रश्न 1.
वायुमण्डल से आप क्या समझते हैं?
उत्तर:
पृथ्वी को चारों ओर से घेरे हुए वायु के आवरण को वायुमण्डल कहते हैं। पृथ्वी की गुरुत्वाकर्षण शक्ति के कारण वायुमण्डल सदा पृथ्वी के साथ सटा रहता है तथा पृथ्वी का एक अभिन्न अंग है। वायुमण्डल के कारण ही पृथ्वी पर जीवन है तथा पृथ्वी एक महत्त्वपूर्ण ग्रह है।

JAC Class 11 Geography Solutions Chapter 8 वायुमंडल का संघटन तथा संरचना

प्रश्न 2.
मौसम एवं जलवायु के प्रमुख तत्त्व बताओ।
उत्तर:
वायुमण्डल दशाएं जो मौसम की रचना करती हैं मौसम के तत्व कहलाती हैं। जैसे:

  1. तापमान
  2. समुद्र तल से ऊंचाई
  3. पवनें
  4. धूप
  5. आर्द्रता
  6. मेघावरण
  7. वर्षा
  8. धुन्ध तथा कोहरा।

प्रश्न 3.
वायुमण्डल की संरचना के बारे में लिखें।
उत्तर:
वायुमण्डल में ऑक्सीजन (21%) तथा नाईट्रोजन प्रमुख गैसें हैं। शेष गैसें कार्बन डाइऑक्साइड, मिथैन, ओज़ोन, हाईड्रोजन हैं।

प्रश्न 4.
वायुमण्डल के सभी संस्तरों में क्षोभ मण्डल सबसे अधिक महत्त्वपूर्ण क्यों है?
उत्तर:
क्षोभमण्डल वायुमण्डल की सबसे निचली परत है जो कई कारणों से महत्त्वपूर्ण है

  1. पृथ्वी के धरातल पर जलवायु स्थितियों का निर्माण करने वाली महत्त्वपूर्ण क्रियाएं इसी परत में होती हैं।
  2. इस परत में गैसों, धूल-कण तथा जलवाष्प की मात्रा अधिक पाई जाती है। इसलिए मेघ, वर्षा, कोहरा आदि क्रियाएं इसी परत में होती हैं।
  3. इस अस्थिर भाग में संवाहिक धाराएं चलती हैं जो ताप और आर्द्रता को ऊंचाई तक ले जाती हैं।
  4. इस भाग में संचालन क्रिया द्वारा वायुमण्डल की विभिन्न परतें गर्म होती हैं। ऊंचाई के साथ-साथ तापमान कम होता है। तापमान कम होने की दर 1°C प्रति 165 मीटर है।
  5. इस भाग में अस्थिर वायु के कारण वायु विक्षोभ तथा आंधी तूफान चलते हैं। वायु परिवर्तन के कारण मौसम परिवर्तन होता रहता है।
  6. क्षोभमण्डल के मध्य अक्षांशीय क्षेत्र में ही चक्रवात उत्पन्न होते हैं। उपरोक्त कारणों से स्पष्ट है कि क्षोभमण्डल वायुमण्डल की सबसे महत्त्वपूर्ण परत है जो मानवीय  या-कलापों पर प्रभाव डालती है।

वायुमण्डल में मौजूद विभिन्न गैसों का महत्त्व निम्नलिखित है:

  1. नाइट्रोजन-नाइट्रोजन एक अक्रियाशील गैस है। यह मानव तथा जीव-जन्तुओं की सुरक्षा करती है। जानवरों तथा पौधों के लिए यह एक आवश्यक गैस है।
  2. ऑक्सीजन-शुष्क वायु में ऑक्सीजन की मात्रा 20.95% होती है। ऑक्सीजन का महत्त्व मानव जीवन के लिए महत्त्वपूर्ण है। इसके बिना जीवन सम्भव नहीं।
  3. कार्बन डाइऑक्साइड-यह गैस सौर विकिरण को सोख लेती हैं। यह एक ग्रीन हाऊस गैस है। इसकी वृद्धि के कारण ग्लोबल वार्मिंग में वृद्धि हो रही है।
  4. ओज़ोन-यह गैस वायुमण्डल के ऊपरी भाग में महत्त्वपूर्ण है। यह गैस सूर्य की परा बैंगनी किरणों को सोख कर पृथ्वी को इसके प्रभाव से बचाती है।

निबन्धात्मक प्रश्न (Essay Type Questions)

प्रश्न 1.
वायुमण्डल के संगठन (रचना) की व्याख्या करो।
उत्तर:
वायुमण्डल अनेक गैसों, जलवाष्प तथा धूल-कणों के मिश्रण से बना हुआ है।

1. गैसें (Gases) वायुमण्डल में ऑक्सीजन तथा नाइट्रोजन प्रमुख गैसें हैं । नाइट्रोजन की मात्रा 78% तथा ऑक्सीजन की मात्रा 21% है। शेष 1% में अन्य गैसें कार्बन डाइऑक्साइड, मिथेन, ओज़ोन, आर्गन, हाइड्रोजन, हीलियम आदि शामिल हैं। इन गैसों की मात्रा कम व अधिक होती रहती है। भारी गैसें वायुमण्डल की निचली परतों में तथा हल्की गैसें ऊपरी परतों में पाई जाती हैं।

ऑक्सीजन, नाइट्रोजन तथा कार्बन डाइऑक्साइड जीव-जन्तुओं तथा पौधों का आधार हैं। 120 कि० मी० की ऊंचाई पर ऑक्सीजन की मात्रा नगण्य हो जाती है। इसी प्रकार कार्बन डाइऑक्साइड एवं जलवाष्प पृथ्वी की सतह से 90 कि० मी० की ऊंचाई तक ही पाए जाते हैं। विभिन्न गैसों की मात्रा इस प्रकार है

JAC Class 11 Geography Solutions Chapter 8 वायुमंडल का संघटन तथा संरचना 1

JAC Class 11 Geography Solutions Chapter 8 वायुमंडल का संघटन तथा संरचना

प्रश्न 2.
वायुमण्डल का चित्र खींचो और इसकी संरचना की व्याख्या करो।
उत्तर:
वायुमण्डल की संरचना अथवा परतें (Structure or Layers of the Atmosphere):
ऊंचाई के साथ वायु दाब तथा ऊष्णता कम होती जाती है । अत: इसी को आधार मान कर वायुमण्डल को निम्नलिखित परतों में विभक्त करते हैं

(i) क्षोभ मण्डल तथा अधोमण्डल (Troposphere): वायुमण्डल की क्षैतिज परतों में से क्षोभमण्डल सबसे निचली परत है। भूतल से इसकी रेडियो तरंगें मध्यमान ऊंचाई 13 किलोमीटर है। इसमें जलवाष्प,धूल-कण तथा भारी गैसें मिलती हैं। इस मण्डल का मानव के लिए महत्त्व अत्यधिक है। क्योंकि ऋतु परिवर्तन तथा ऋतु से सम्बन्धित अन्य कार्य इसी परतमें होते हैं।

इसमें निरन्तर पवनें तथा संवहनीय धाराएं (Con-vectional Currents) चलती हैं। क्षोभ मण्डल समताप मण्डल में ऊंचाई के अनुसार तापमान निम्न हो जाता है। प्रति 165 मीटर की ऊंचाई पर 1° सेण्टीग्रेड तापमान कम क्षोभ सीमा होता जाता है। इस मण्डल में परिवर्तन होते रहते हैं।

JAC Class 11 Geography Solutions Chapter 8 वायुमंडल का संघटन तथा संरचना 2

(ii) मध्यस्तरअथवा क्षोभसीमा (Tropopause):
तापमान (फ) यह वायुमण्डल की वह परत है जहां क्षोभमण्डल समाप्त हो जाता है। एक अन्य नवीन परत समताप प्रारम्भ हो जाती है। मध्यस्तर की चौड़ाई लगभग [latex}1 \frac{1}{2} [/latex] किलोमीटर है। इस परत में प्रविष्ट होते ही क्षोभमण्डल की पवन तथा संवहनीय धाराएं समाप्त हो जाती हैं। यहां तापमान स्थिर रहता है। वायु तापमान भूमध्य रेखा पर -80° C तथा ध्रुवों पर -45° C रहता है।

JAC Class 11 Geography Solutions Chapter 8 वायुमंडल का संघटन तथा संरचना 3

(iii) समतापमण्डल (Stratosphere):
मध्यस्तर के ऊपर समताप मण्डल है। इसकी ऊंचाई भूमध्य रेखा पर अधिक तथा ध्रुवों पर कम है। इसी प्रकार ग्रीष्मकाल में इसकी ऊंचाई शीतकाल की अपेक्षा अधिक होती है। स्पूतनिकों (Sputniks) द्वारा किए अन्वेषणों से इसकी ऊंचाई 16 से 80 किलोमीटर तक आंकी गई है। इस मण्डल में ऊंचाई के अनुसार तापमान में वृद्धि नहीं होती अपितु यह समान रहता है। इसमें विकिरण (Radiation) द्वारा ताप क्षोभसीमा. का ग्रहण ताप की मुक्ति के समान होता है। फलत: इसे क्षोभमण्डल समताप मण्डल कहते हैं। ध्रुवों के ऊपर -45° C तथा भूमध्यरेखा पर -80° तापमान रहता है।

(iv) ओज़ोन मण्डल (Ozonesphere):
इस मण्डल में ओजोन गैस की प्रधानता के कारण इसे ओज़ोन मण्डल कहते हैं। ओजोन गैस सूर्य से निकलने वाली अत्यन्त ऊष्ण पराबैंगनी किरणों को सोख लेती है। वायुमण्डल में यदि यह परत न होती तो पृथ्वी पर विद्यमान प्राणी जीवन को अपार हानि होती । पराबैंगनी किरणों से मनुष्य अन्धे हो जाते हैं तथा उनका शरीर झुलस जाता है। इस मण्डल की ऊंचाई 20 से 80 किलोमीटर तथा प्रति किलोमीटर ऊंचाई पर 16° सैण्टीग्रेड तापमान बढ़ जाता है अतः यह मण्डल अत्यन्त गर्म है।

(v) मध्य मण्डल (Merosphere):
यह समताप मण्डल से ऊपर 80 कि० मी० की ऊंचाई तक इसका विस्तार है। ऊंचाई के साथ तापमान बढ़ने लगता है। 80 कि० मी० की ऊंचाई तक तापमान-100°C होता है। ऊपरी सीमा को मध्य सीमा कहते हैं।

(vi) आयन मण्डल (Ionosphere):
यह मण्डल 80 से 400 किलोमीटर की ऊंचाई पर विद्यमान है। इसमें तापमान का वितरण असमान एवं अनिश्चित है। इस मण्डल में बड़ी ही विस्मयकारी विद्युतकीय घटनाएं दृष्टिगोचर होती हैं। इस परिमण्डल में विद्युत् से चार्ज कण ईओन मिलते हैं। पृथ्वी से ऊपर जाने वाली रेडियो तरंगें पृथ्वी पर पुनः लौट आती हैं।

(vi) बाह्य मण्डल (Exosphere):
वायुमण्डल की यह सर्वोच्च परत है जिसकी ऊंचाई 400 किलोमीटर से अधिक है। अधिकांशतः इसमें हाइड्रोजन तथा हीलियम गैसें विद्यमान हैं। यहां वायु विरल है। यहां का तापमान 6,000° सैण्टीग्रेड होने का अनुमान है।

 वायुमंडल का संघटन तथा संरचना JAC Class 11 Geography Notes

→ मौसम (Weather)-मौसम शब्द का अर्थ है “किसी स्थान पर किसी विशेष या निश्चित समय में वायुमण्डल की दशाओं, तापक्रम, दबाव, हवाओं, नमी, मेघ और वर्षा के कुल जोड़ का अध्ययन करना।” जलवायु (Climate)-किसी स्थान की जलवायु उस स्थान पर एक लम्बे समय की वायुमण्डल की दशाओं के कुल जोड़ का अध्ययन होती है। यह एक लम्बे समय का औसत मौसम होती है।

→ मौसम तथा जलवायु के तत्त्व (Elements of Weather and Climate): मौसम के तत्त्व हैं

  • तापमान
  • दबाव
  • हवाएं
  • धूप
  • मेघावरण
  • आर्द्रता
  • वर्षा
  • धुन्ध तथा कोहरा।

जलवायु के तत्त्व हैं

  • अक्षांश
  • समुद्र तल से ऊंचाई
  • स्थल तथा जल का वितरण
  • वायुदाब का वितरण
  • प्रचलित पवनें
  • महासागरीय धाराएं
  • पर्वतीय अवरोध।

→ वायुमण्डल (Atmosphere): पृथ्वी को चारों ओर से घेरे हुए वायु के आवरण को वायुमण्डल कहते हैं। पृथ्वी की गुरुत्वाकर्षण शक्ति के कारण वायुमण्डल सदा पृथ्वी के साथ सटा रहता है तथा पृथ्वी का एक अभिन्न । अंग है। वायुमण्डल के कारण ही पृथ्वी पर जीवन है तथा पृथ्वी एक महत्त्वपूर्ण ग्रह है। यहां जीवनदायिनी गैस ऑक्सीजन आदि मिलती है।

JAC Class 11 Geography Solutions Chapter 8 वायुमंडल का संघटन तथा संरचना

→ वायुमण्डल का संघटन (Composition of Atmosphere): वायुमण्डल की संरचना मुख्य तीन तत्त्वों द्वारा होती है(1) गैसें (Gases) नाइट्रोजन (78%), ऑक्सीजन (21%) तथा अन्य (1%)।

→ जल वाष्प (Water Vapour): वायुमण्डल में 2% जल वाष्प पाया जाता है।

→ धूल कण (Dust Particles): ये सूर्यताप को जज़ब करके बादल तथा धुन्ध की रचना करते हैं।

→ वायुमण्डल की परतें (Layers of Atmosphere): तापमान तथा घनत्व के आधार पर वायुमण्डल को निम्नलिखित पांच परतों में बाँटा जा सकता ह

  • क्षोभमण्डल या अधोमण्डल (Troposphere): सबसे निचली परत।
  • समताप मण्डल (Stratosphere): क्षोभ मण्डल से ऊपरी परत।
  • ओज़ोन मण्डल (Ozonesphere): ओज़ोन गैस क्षेत्र।
  • आयन मण्डल (lonosphere): आयन गैस क्षेत्र।
  • बाह्य मण्डल (Exosphere): सबसे बाहरी परत।